X



トップページ数学
1002コメント333KB
大学学部レベル質問スレ 22単位目
レス数が1000を超えています。これ以上書き込みはできません。
0001132人目の素数さん
垢版 |
2023/05/09(火) 18:03:26.26ID:mAuYyNSK
大学で習う数学に関する質問を扱うスレ

・質問する前に教科書や参考書を読むなりググるなりして
・ただの計算は
http://wolframalpha.com
・数式の表記法は
http://mathmathmath.dote ra.net
・質問のマルチポストは非推奨
・煽り、荒らしはスルー

※前スレ
大学学部レベル質問スレ 21単位目
https://rio2016.5ch.net/test/read.cgi/math/1675998924/
大学学部レベル質問スレ 20単位目
https://rio2016.5ch.net/test/read.cgi/math/1669086920/
大学学部レベル質問スレ 19単位目
https://rio2016.5ch.net/test/read.cgi/math/1659623368/
0002132人目の素数さん
垢版 |
2023/05/09(火) 18:11:29.76ID:juBMpiOq
>>996
双曲線と反比例グラフに関わることで
前のスレッドで質問したものです。
ありがとうございました!!
指摘していただくまで、ずっと勘違いしたまま困っていました!
0004132人目の素数さん
垢版 |
2023/05/09(火) 21:04:25.32ID:qHc8HPKW
>>2
そういう勘違いから新たな数学が生まれ無いとも限らない
AIにはむ〜り〜
0005132人目の素数さん
垢版 |
2023/05/09(火) 21:31:53.54ID:3BFK4cOJ
松坂和夫 集合位相入門や
Sheldon Axler Measure, Integration & Real Analysisをどこまで読んだとか良い悪いとか、万年初心者の馬鹿の感想は要らない。大体この馬鹿の書き込みでスレが埋まる。
こういう書き込みをすると連レスしてくるのが常。
0007132人目の素数さん
垢版 |
2023/05/10(水) 15:19:41.09ID:FX6BTpli
勉強するだけならな
自分の能力を棚に上げて著者を罵倒するのと、自分の感性が世界で一番優れてると思い込んでるところがクソなんだわ
0009132人目の素数さん
垢版 |
2023/05/11(木) 19:23:39.37ID:x9nwPekj
R^2では非コンパクトで凸な領域A上で計量を変化して(R^2\Aでは計量は変えずに)
完備で非正曲率のまま変形する事が出来ることを示せ

という問いが分かりません。
3次元以上ではこのような事は成り立たないようなのですが
2次元でこれがどのように可能なのか分かる人いたら教えて欲しいです。
0011132人目の素数さん
垢版 |
2023/05/11(木) 19:57:22.03ID:YRtuxm2Z
>>9
変な質問ばかりしてないできちんと勉強した方が良いよ。
他人のために良問を投下してやってるとか思い上がってるなら100年早いよ。
…と言われてやめるようならそもそもやってないだろうけど。
0012132人目の素数さん
垢版 |
2023/05/11(木) 20:10:11.06ID:x9nwPekj
>>11
Gromov他のmanifolds of nonpositive curvatureという本の中で
地の文の中でexerciseとして書かれてる内容(70ページ)なので
きちんとした主張がよくわかりにくいですが

例えばR^2の普通の計量からx<0の範囲では変化させずに
x≧0の領域だけで非正曲率(で少なくとも一点では負曲率)になるように計量を変化できる
というような内容を主張しているのだと思います
0013132人目の素数さん
垢版 |
2023/05/11(木) 20:16:45.06ID:x9nwPekj
Mostowの剛性定理とかあたりが目標の本なようなので
剛性が3次元では成り立つけど2次元では成り立たず変形できるというような
内容の前提になる話なんじゃないかと思うのですが
>>12の具体的な問題の形でもいいので分かる方いたら教えて下さい。
0014132人目の素数さん
垢版 |
2023/05/11(木) 21:13:36.14ID:YXKswMfi
ともかくその本の内容の流れがわかる人間にはそれで伝わるかもしれないけど本の内容の知識ゼロの人間にそれで何か伝わるはずがない
そんな問題解くよりまずそのレベルの常識すらないのではダメ
0016132人目の素数さん
垢版 |
2023/05/12(金) 06:14:45.14ID:FTkrnJtI
内容ゼロな説教のための説教をハゲ散らかすハゲをハゲ呼ばわりするのはもっともな話
0017132人目の素数さん
垢版 |
2023/05/12(金) 09:34:11.53ID:HWSG+U5+
5chで質問しながら数学の本を読み進めようという気がしれない
結局挫折して別の本をまた斜め読みしてまた質問そして挫折してまた別の本笑
0018132人目の素数さん
垢版 |
2023/05/13(土) 19:47:51.90ID:wmHV5vNl
物_1, …, 物_p がそれぞれ2個ずつある。
物_{p+1}, …, 物_{p+q} がそれぞれ1個ずつある。

これらの物たちの中から、 r 個の物を選ぶ選び方は何通りあるか?
0020132人目の素数さん
垢版 |
2023/05/13(土) 21:08:32.92ID:wmHV5vNl
Σ を使ってもOKです。
0022132人目の素数さん
垢版 |
2023/05/13(土) 21:51:28.08ID:wmHV5vNl
不正解です。
0023132人目の素数さん
垢版 |
2023/05/13(土) 21:55:17.99ID:zw4BWsOA
不正解ってこんなのΣ使ったら表示なんかいくらでもあるんじゃないの?
自分の用意した答えと見た目に違っても不正解とは限らないってわかってる?
0025132人目の素数さん
垢版 |
2023/05/13(土) 22:50:23.39ID:nmCoUxz7
あんまり大学学部レベルっぽく無いなあ
順列組合せの無意味な問題は
せいぜい入試数学まででは無いのかな
0026132人目の素数さん
垢版 |
2023/05/13(土) 22:52:26.25ID:nmCoUxz7
大学学部レベル数学を学ぶと
新たな次元の広がりを感じるんだけど
同じテーマでどんどん複雑になるだけだと
つまんないんだよね
0027132人目の素数さん
垢版 |
2023/05/13(土) 22:53:23.81ID:nmCoUxz7
いくら小学校で算数が上手でも
中学校でははあそうですかってなるみたいな
0029132人目の素数さん
垢版 |
2023/05/13(土) 23:29:13.86ID:QSZzaVbc
>>22
そもそもここは出題スレではないので、あなたの知能検査をした方がいいです
0030132人目の素数さん
垢版 |
2023/05/14(日) 03:42:28.09ID:TNg6ZeUb
>>28

ヒント: r - 2*i < 0 となってしまう可能性がありますよね。
0032132人目の素数さん
垢版 |
2023/05/14(日) 10:05:51.50ID:bPyyzHlR
そもそもこんな出鱈目に数値設定して正しい答え出してるコード目の前にしてヒントとかアホじゃないの?
0034132人目の素数さん
垢版 |
2023/05/14(日) 10:32:28.98ID:TNg6ZeUb
Binomialの通常の定義は何でしょうか?
0035132人目の素数さん
垢版 |
2023/05/14(日) 10:33:18.39ID:TNg6ZeUb
>>31

結果は正しいので、コードを読まなくても、ゼロにしているのは分かっていました。
0036132人目の素数さん
垢版 |
2023/05/14(日) 10:33:49.89ID:TNg6ZeUb
>>34

訂正します:

Binomialの通常の定義域は何でしょうか?
0037132人目の素数さん
垢版 |
2023/05/14(日) 10:46:50.10ID:bPyyzHlR
だからお前のそういうところがダメなんだよ
そんな事今の問題で重要な事か?
そういうクズみたいな重箱突きがやめられない、しかもコード読めば正しい認識してて記述のうるささを避けるためにあえて省略してるのわかるやろ?
そしてその重箱突きがやめられないのがお前が学問に向いてない理由なんだよ
お前まさか
自分が学問に向いてて自分の学力が十分なスピードで向上してると思ってないよな?
普通の人間なら5年も数学勉強したら修士論文に取り掛かってるハズの頃だよ
お前永遠に受験数学〜般教から逃れられてないやん?

 そ の 現 実 を 直 視 せ よ
0038132人目の素数さん
垢版 |
2023/05/14(日) 10:49:40.85ID:t3e83vEE
頭が悪いのがはっきりするよな
馬鹿の独学は実を結ばない
何やってもこいつは駄目
0039132人目の素数さん
垢版 |
2023/05/14(日) 10:53:16.08ID:bPyyzHlR
馬鹿が馬鹿であるのは当然理由がある
運が悪いとかなんとかではない
本人に人格上の問題がある
その事実に気づき、向き合う覚悟ができない限り何も始まらない
そしてそれが始められる人と始められる人がいる
それはおそらく20歳ぐらいまでで決まる
そこまでのチャンスを逃したらもう多分一生チャンスは来ない
0040132人目の素数さん
垢版 |
2023/05/14(日) 10:59:36.23ID:t3e83vEE
厳密な理解と称して大学1年生の5月時点のレベルから一歩も前進しない馬鹿
要点を理解し本質を掴んで前に進むことが出来ずひたすら教科書の記述にケチをつけて○十年
馬鹿が馬鹿のまま固まった悲惨な実例がこいつ
0041132人目の素数さん
垢版 |
2023/05/14(日) 12:31:12.81ID:EoOtgrg0
>>40
でも、ええやん
こんなアホでも数学書は結構買ってるみたいだから、業界的にはいいカモやで
0042132人目の素数さん
垢版 |
2023/05/14(日) 16:27:54.89ID:b6bnXRmB
学部における分野ごとの学ぶ順番についての質問なのですが、
微分幾何学と多様体論はどちらから学ぶべきですか?

ネットで様々な大学のカリキュラムを調べても微分幾何を先に学んでいるところと多様体論を先に学んでいるところと両方あって困ってます
「大学では〇〇を先に学んだ」や「〇〇を先に学ぶともう一方が理解しやすい」などの意見を伺いたいです
0043132人目の素数さん
垢版 |
2023/05/14(日) 16:32:18.73ID:dPfWqPpF
どちらも何も微分幾何≒(可微分)多様体論だが
0044132人目の素数さん
垢版 |
2023/05/14(日) 16:44:08.22ID:b6bnXRmB
>>43
そうなんですか!?
図形の曲率とかを考えるのが微分幾何学で、球面から平面を切り取ってその性質について考えるのが多様体論だと思ってました
本質的には同じものなんですね……
0045132人目の素数さん
垢版 |
2023/05/14(日) 16:44:42.85ID:dPfWqPpF
とりあえず曲線と曲面を先に適当な本でやってから多様体でいいんじゃね
ただ、その様子だと数学科の学生ではないようなので位相空間を知らないと仮定して、その間か多様体と並行して位相空間をやるべき

といっても深くやる必要はなくて「開集合・閉集合、近傍、第二可算性とハウスドルフ空間」の定義と「連結性と(局所)コンパクト性のもつ簡単な性質」くらいを抑えればいい(正則・正規空間やら完備距離空間やらはひとまず後回しでok)
0046132人目の素数さん
垢版 |
2023/05/14(日) 16:52:27.30ID:dPfWqPpF
>>44
同じというより曲線と曲面を一般化したものが多様体
曲線曲面はそれぞれ1,2次元の多様体
0047132人目の素数さん
垢版 |
2023/05/14(日) 17:21:14.47ID:b6bnXRmB
>>45
詳しくありがとうございます!
位相空間論は一通り目を通したのですが曖昧な部分も多いので、挙げて頂いた部分を重点的に復習してから曲線と曲面、そして多様体論へと学習を進めていきたいと思います
教えて頂きありがとうございました!
0050132人目の素数さん
垢版 |
2023/05/14(日) 20:54:41.43ID:pEuyRQrx
>>49
2×6×6
0051132人目の素数さん
垢版 |
2023/05/14(日) 21:57:40.58ID:RKFJuss+
数式(x^3-2)^(1/2)の整数解は、
一つしか存在しないことを証明してくれ
0055132人目の素数さん
垢版 |
2023/05/14(日) 22:54:32.37ID:EoOtgrg0
ブール代数Bがc.c.c.ならばBのストーン空間もc.c.c.であることの証明はどこに載ってるか教えてくれ。
0056132人目の素数さん
垢版 |
2023/05/15(月) 00:05:38.22ID:5F6s6RCL
>>41
>業界的には
業界?w
スンゲ違和感ある用語ね
0057132人目の素数さん
垢版 |
2023/05/15(月) 00:07:20.81ID:5F6s6RCL
本を書くってのは
大学で講義するのと同じ
ただの余暇手慰みみたいなもん
0060132人目の素数さん
垢版 |
2023/05/16(火) 09:14:10.55ID:hzv96Gma
c.c.c.って何だっけと思って日本語のwikiみたらヒドいな
反鎖を未定義のまま書いてるから意味不明
ググってもそのwikiの不完全な文ばかりが引用されてて全くちゃんとした定義に辿り着けない
(もちろん英語で調べたらすぐ分かったが)
0061132人目の素数さん
垢版 |
2023/05/16(火) 10:04:42.04ID:psdXXL9a
まぁどのみち証明知りたいんじゃなくて証明載ってる文献知りたいわけだから証明考えても意味ないやろな
ともかく質問する時に「どういう概念はエクスキューズなしに使っていいか」すらわかってないアホ質問多い
数学本体より2年も3年も数学の世界で勉強してまだその程度の事がわからない方が重大
0062132人目の素数さん
垢版 |
2023/05/16(火) 13:44:56.22ID:cboqZUoS
原啓介著『測度の考え方』

最初のほうは分かりやすくて、証明も書いてあっていい本だと思います。
ですが、証明が書かれていないE. Hopfの拡張定理あたりからは読む意味がないですね。


著者はACCESSで働いていたとのことですが、何をやっていたんですかね?
0063132人目の素数さん
垢版 |
2023/05/16(火) 13:48:57.63ID:cboqZUoS
盛田健彦著『実解析と測度論の基礎』を借りてきて少し見ていますが、親切な本ではないですね。

その文章から一癖ありそうな人という印象です。
0064132人目の素数さん
垢版 |
2023/05/16(火) 13:50:43.58ID:cboqZUoS
盛田さんの本だったら、吉田伸生さんの本のほうがマシだと思います。

盛田さんの本は絶版ですが、吉田さんの本は新装版が出版されました。
0065132人目の素数さん
垢版 |
2023/05/16(火) 13:51:44.50ID:cboqZUoS
そして、やはり一番分かりやすいのは、Axlerさんの本です。

Egorovの定理の証明を先程、読みました。
0066132人目の素数さん
垢版 |
2023/05/16(火) 14:08:25.35ID:9rmuiIMM
ID:cboqZUoS
死ね
0067132人目の素数さん
垢版 |
2023/05/16(火) 14:20:42.78ID:aKfc+dzN
そもそも自分の数学力が一つも向上していないという事実にまるで向き合えていない
0069132人目の素数さん
垢版 |
2023/05/16(火) 16:07:08.99ID:cboqZUoS
Springerの多くの数学の本が2270円でいま売っていますが、みなさんは何を買いますか?

Foundations of Differentiable Manifolds and Lie Groups (Graduate Texts in Mathematics) (Graduate Texts in Mathematics, 94) ペーパーバック ? 2010/12/4
英語版 Frank W. Warner (著)

ってどうですか?
0070132人目の素数さん
垢版 |
2023/05/16(火) 16:09:53.75ID:cboqZUoS
Springerの本について質問です。

この前買ったBourbakiの英訳本はスキャンしたような感じの本でした。

一方、例えば、Axlerさんの本のように、綺麗に印刷された本もあります。

見分ける方法はありますか?
0071132人目の素数さん
垢版 |
2023/05/16(火) 16:24:13.58ID:cboqZUoS
安いから何でも買いたくなりますが、床から天井までの7段×4列の作り付けの本棚が既に一杯なので、どうすればいいのか悩んでいます。
0072132人目の素数さん
垢版 |
2023/05/16(火) 16:38:16.89ID:cboqZUoS
積読になる確率が非常に高いので、必要最小限の本のみ買うことにします。
0073132人目の素数さん
垢版 |
2023/05/16(火) 16:50:57.65ID:cboqZUoS
とりあえず、今、全く理解できないような高度な本はすべてやめて、今読んでも参考になるような本から選んで買おうと思います。
0075132人目の素数さん
垢版 |
2023/05/16(火) 17:36:34.46ID:cboqZUoS
>>74

まだ読む可能性のある積読の本が大半なので、売りません。
売るとしたら、不要になった日本語の微分積分と線形代数の本です。
0076132人目の素数さん
垢版 |
2023/05/16(火) 17:43:39.75ID:cboqZUoS
>>73

結局、6冊注文しました。
ブルバキのソフトカバーの例があるので、印刷品質が心配です。

Warner Foundations of Differentiable Manifolds and Lie Groups
Bott & Tuの有名な本
Lang Complex Analysis
Forster Lectures on Riemann Surfaces
Apostol Modular Functions and Dirichlet Series in Number Theory
Bondy & Murty Graph Theory

Langとグラフ理論の本は今でも読めると思います。
ApostolはIntroduction to Analytic Number Theoryという本を持っているので、何となく買ってしまいました。
0077132人目の素数さん
垢版 |
2023/05/16(火) 17:44:54.73ID:cboqZUoS
なんかもっとほしいような気がするので、追加で注文するかもしれません。
0078132人目の素数さん
垢版 |
2023/05/16(火) 17:47:11.88ID:HPV3yfo9
>>75
積読の本は電子化しろ
5~6万あれば電子化の環境は整う
で裁断した本を安く売れ
0079132人目の素数さん
垢版 |
2023/05/16(火) 17:48:09.97ID:cboqZUoS
Springerの有名な本だとメルカリとかに出品しても2000円くらいで売れそうですよね。

ただ、印刷品質が心配です。
0080132人目の素数さん
垢版 |
2023/05/16(火) 17:51:02.47ID:cboqZUoS
>>78

それがどうも電子化したファイルだと読む気がしないんですよね。
Axlerさんのルベーグ積分の本も無料で公開されていますが、結局ハードカバーの本を買いました。
電子化したファイルに、うまく適応できていないだけなのかとも思ったのですが、やはり紙の本のほうが利用しやすいです。
0082132人目の素数さん
垢版 |
2023/05/16(火) 18:00:11.30ID:cboqZUoS
>>80

大きなタブレットPCだと読みやすいのかなという気もしていますが、持っていないので、試していません。
0083132人目の素数さん
垢版 |
2023/05/17(水) 04:53:32.44ID:vzoA5eMR
確認

普通、何かの代数的な集合Xについて、X上の極大とか、超とか、素とかの冠詞がついたフィルターって言葉は
X自身は除外するのが常識っていうか普通だよな?

X自身もそれらに該当すると定義してる書籍ってまず無いよな?
0084132人目の素数さん
垢版 |
2023/05/17(水) 05:19:16.07ID:BQNz1g/c
>>83
フィルターにはX入るに決まってる
0085132人目の素数さん
垢版 |
2023/05/17(水) 08:46:38.52ID:3Fs5dMUy
そういうことちゃう
環R自身が極大イデアルとは言わないよな?ってことでしょ

そりゃX自身を極大○○に含めるならそれただ一つの最大のものになるから意味なくすわけで
0086132人目の素数さん
垢版 |
2023/05/17(水) 13:01:57.79ID:qsDmF3QC
>>85
違う
フィルターは集合の集合よ
0087132人目の素数さん
垢版 |
2023/05/17(水) 19:48:07.65ID:vzoA5eMR
順序、半順序、擬順序、の定義がマジで著書によってバラバラに分かれてるから、
マジで言葉の統一してほしいわ
0088132人目の素数さん
垢版 |
2023/05/18(木) 17:27:04.56ID:H2b7yVLZ
>>55自己解決した

年を追うごとに自分の数学的思考力が堕ちてきてるのがマジで痛感させられる
0089132人目の素数さん
垢版 |
2023/05/18(木) 19:50:22.50ID:5MGEdJSR
大学以降の数学の用語を取り決めるような機関はない
工学とか医学ならそういう言葉の違いの不統一が招くリスクを避けるために用語の業界標準を決める機関があるけど数学にはない
そもそもどういう定義が優れているのかは文脈によって変わる
その教科書、論文で議論している内容にそぐわない用語を強制的に使わされたらいらない不自然さでかえって読みにくくなってしまう
そういうメリット、デメリットを鑑み、相互的に判断して、数学の世界では用語を統一する事をせず、著者の設定を読み抜かない練習をしとかなければならん
もしかしたら数学本体そのものよりそういう数学の文献との付き合い方を理解できるようになることの方が大切だったりする
実際このスレでもいるやろ?どういう文脈での質問なのか何も知らせないで話題振ってくるアホ
何年数学の世界で勉強しとんじゃと思う
0090132人目の素数さん
垢版 |
2023/05/18(木) 20:19:19.65ID:H2b7yVLZ
0は自然数に含めるべきやろ
0を自然数に含めた時の不利益ってΣ[n;自然数]a_n/nみたいな表記をする時に分母に0が来てしまうってことぐらいやろ
0091132人目の素数さん
垢版 |
2023/05/18(木) 22:40:54.82ID:xBdxpoyX
自然数に0を含めたら、今度は「1以上の整数」に新しく名前を付けたくなるね
そういえば自然数と非負整数って同じじゃん、これは無意味
じゃあ「1以上の整数」を自然数と名付けよう
0092132人目の素数さん
垢版 |
2023/05/18(木) 22:57:35.53ID:NngCi4QT
>>91
非正整数={0,−1,−2,−3,……}
非非正整数={1,2,3,4,……}
0093132人目の素数さん
垢版 |
2023/05/18(木) 23:08:03.79ID:H2b7yVLZ
それと、もう1つ大切なことは、否定の接頭辞は極力排除した方がいい
紛らわしさを減らすため
0095132人目の素数さん
垢版 |
2023/05/19(金) 09:39:16.61ID:LHim7DL2
[定理]
平方数と立方数にはさまれた
唯一の数は26である

[証明]
k,xは自然数,kx≠0とする

x^3-(x+k)^2=2 から
∴整数解は、k=2,x=3
0096132人目の素数さん
垢版 |
2023/05/19(金) 09:47:15.33ID:Cvv9AI1y
A, B を R のボレル部分集合とします。

A ∩ B = 空集合とします。

|A ∪ B| = |A| + |B| が成り立ちます。

|A ∪ B| ≦ |A| + |B| は、 A, B がどんな R の部分集合であっても成り立ちます。

この状況で、

|A ∪ B| を評価する際、

|A ∪ B| ≦ |A| + |B| が成り立つという情報だけで十分であるとします。

そのとき、

|A ∪ B| ≦ |A| + |B|

と書くのと、

|A ∪ B| = |A| + |B|

と書くのではどちらが良いでしょうか?
0098132人目の素数さん
垢版 |
2023/05/19(金) 10:30:08.87ID:Cvv9AI1y
>>97

外測度です。
0099132人目の素数さん
垢版 |
2023/05/19(金) 11:37:14.66ID:Cvv9AI1y
ルベーグ可測集合って重要ですか?

Barry Simonという人が

"Passing from Borel to Lebesgue measurable functions is the work of the devil. Don't even consider it!"

と書いているそうですね。
0101132人目の素数さん
垢版 |
2023/05/19(金) 18:32:26.74ID:KyOAo8/s
あまりに素晴らしすぎるから“悪魔の技”なのでは?
0102132人目の素数さん
垢版 |
2023/05/19(金) 19:40:15.70ID:Cvv9AI1y
Sheldon Axler著『Measure, Integration & Real Analysis』

やっと、ルージンの定理の証明を読み終わり、積分の章に入りました。

問題は最初はすべて解いていましたが、途中からすべてサボることにしました。
0103132人目の素数さん
垢版 |
2023/05/19(金) 19:52:18.48ID:ES6dylen
>>100
0104132人目の素数さん
垢版 |
2023/05/19(金) 22:23:34.46ID:WTe3zdcz
非交和位相∐{i∈I}XiでIは無限集合の時。この開集合は{∪Ui|i有限個}という形で表される。これは正しいでしょうか。
0105132人目の素数さん
垢版 |
2023/05/20(土) 12:06:42.30ID:w5R1vM0f
リーマン多様体Mでのisometryφ:M→Mの誘導する接空間での準同型φ_*と
リーマン幾何のある測地線cに沿った平行移動Pとは可換でしょうか?
つまりxでの接空間をφ_*でφxの接空間にうつしてからφcに沿った平行移動Pで移したものと
cに沿ってxからPで平行移動してからφ_*を当てたものとが一致するかが知りたいです

(欲しいのは一般の場合ではなくMがアダマール多様体の時の結果なので,必要ならそう制限して下さい
この場合は任意の2点をつなぐ測地線がただ一本である事などが使えます)
分かる方いたらよろしくおねがいします
0106132人目の素数さん
垢版 |
2023/05/20(土) 12:26:56.40ID:/j81lLf1
Sheldon Axler著『Measure, Integration & Real Analysis』

積分の章を読んでいますが、

∞ × 0 = 0 × ∞ = 0

という約束が書いてあります。

この約束を使うのがどこであるかが分かりません。

どこで使われるのか注意深く読んでいこうと思います。
0107132人目の素数さん
垢版 |
2023/05/20(土) 12:29:50.26ID:/j81lLf1
あ、分かりました。

∫ χ_E dμ = μ(E)

の証明で、既に使われていました。
0108132人目の素数さん
垢版 |
2023/05/20(土) 14:06:52.02ID:/j81lLf1
この約束ですが、

μ(A) = ∞ で inf_{A} f = 0 のときに、

μ(A) * inf_{A} f = 0 にしたいからそう約束したんですかね?
0109132人目の素数さん
垢版 |
2023/05/21(日) 00:08:03.42ID:A5n454pj
質問よろしいでしょうか?
任意のiでAがB_iの稠密な部分集合ならば、
(A)^nはΠB_iで稠密だと示せるでしょうか?
0110132人目の素数さん
垢版 |
2023/05/21(日) 00:39:34.75ID:PZvBgu+N
直積の閉包は閉包の直積なのでそうですね
0111132人目の素数さん
垢版 |
2023/05/21(日) 02:41:03.25ID:A5n454pj
ありがとうございます。
0112132人目の素数さん
垢版 |
2023/05/21(日) 17:42:05.24ID:+aSCXQBZ
>>105
一般にM上のベクトル束Eに計量が与えられたときEの計量から自然にレヴィ・チヴィタ接続なる接続があれは一意に定まるそうです
変換φがisometricならM本体の接束Tとそのφによる引き戻しのベクトル束T'それぞれに計量が定まってその間に自然な変換φ_.が誘導されてる状況だと思います
この状況ならφはTの接続∇とT'の接続∇'と可換になると思います
当然∇で記述される平行移動の方程式もφと可換になると思います
0113132人目の素数さん
垢版 |
2023/05/22(月) 09:35:40.03ID:Dpb8PkHI
>>112
ありがとうございます
isometryが∇と可換である事が言えればいいというのが盲点でした
これなら地道に定義に戻って確認できそうです感謝です
0114132人目の素数さん
垢版 |
2023/05/24(水) 11:08:22.60ID:pUzXi7qd
>>76

注文した本がさきほど届きました。

埼玉県久喜市の大日本印刷からすべて送られてきました。

印刷のクオリティですが、出版されたのが比較的新しいと思われるグラフ理論の本だけ綺麗でした。
後は、ブルバキの英訳本ほど酷くはないですが、よく見るとあらが目立ちます。
岩波のオンデマンドよりは良いです。

PODでおそらく工場から直送なので、外観はパーフェクトです。

ドイツから送られてこなくて良かったです。
というのも、埼玉県久喜市の大日本印刷から発送の場合は、一応、ビニールのクッションにくるまれて段ボール箱に入れて送られてきます。
雨に多少濡れても大丈夫だと思います。

ですが、ドイツから送られてくる場合には、ダンボール箱に裸の本がそのまま入っています。
ひどい雨が降った場合には、本が水に濡れてしまいます。
クレームを入れれば、濡れた本はそのままもらえて、交換品を送ってくれますが、大量に購入した場合にはこちらは悪くないのに、Springerに損をさせたような気になりますよね。

セール期間中に大量に購入することをおすすめします。
0115132人目の素数さん
垢版 |
2023/05/24(水) 11:10:37.82ID:pUzXi7qd
Forster Lectures on Riemann Surfaces

ですが、アルコールで表面を拭いているときに、手が滑って机の上に落として、少し凹みを作ってしまいました。

少しショックです。
0116132人目の素数さん
垢版 |
2023/05/24(水) 11:49:40.69ID:pUzXi7qd
今日Springerで何を注文するか考えて、明日、まとめて注文しようと思います。

絶対に必要でほしいと思っていた本を注文するのではないので、選ぶのって結構苦痛なんですよね。
その苦痛のコストがかかっていると考えると、それほど安いようにも思えなくなってきます。
0119132人目の素数さん
垢版 |
2023/05/25(木) 08:03:51.30ID:VlcFMeSd
>>117

中身は誤植が訂正されている以外は、1981年のハードカバーのものと同じみたいです。

表紙の裏に以下の情報が書かれています(他に情報はありません):

1981年のハードカバー第1版の、ソフトカバーでのリプリントと書いてあります。
他に、1999年の訂正された第4刷とも書いてあります。
0120132人目の素数さん
垢版 |
2023/05/25(木) 12:02:36.34ID:VlcFMeSd
追加でSpringerから購入する本を検討していたら、20冊になってしまいました。
0121132人目の素数さん
垢版 |
2023/05/25(木) 13:26:18.46ID:VlcFMeSd
松坂和夫著『現代数学序説』

この本の母関数についての説明が分かりやすいです。
松坂さんが母関数について参考にした本は何ですか?
0122132人目の素数さん
垢版 |
2023/05/25(木) 13:35:50.97ID:VlcFMeSd
形式的ベキ級数 p(x) で p(x)^q = 1 + x となるようなものが存在することを、全く代数的に証明できるんですね。

今、松坂和夫著『現代数学序説』を図書館で借りて見ているのですが、買おうと思います。
0123132人目の素数さん
垢版 |
2023/05/25(木) 19:51:57.69ID:VlcFMeSd
>>121

おかしなところを見つけてしまいました。

(1 + x)^(1/2) = 1 + a_1*x + a_2*x^2 + a_3*x^3 + …

という形式的ベキ級数の x に -x を「代入」して

(1 - x)^(1/2) = 1 - a_1*x + a_2*x^2 - a_3*x^3 + …

という計算をしているところがあります。

ですが、形式的ベキ級数の x に代入することの定義が書いてありません。
0124132人目の素数さん
垢版 |
2023/05/25(木) 20:04:28.00ID:VlcFMeSd
(a_0 + a_1*x + a_2*x^2 + a_3*x^3 + …) * (a_0 + a_1*x + a_2*x^2 + a_3*x^3 + …) = c_0 + c_1*x + c_2*x^2 + c_3*x^3 + …



(a_0 - a_1*x + a_2*x^2 - a_3*x^3 + …) * (a_0 - a_1*x + a_2*x^2 - a_3*x^3 + …) = d_0 + d_1*x + d_2*x^2 + d_3*x^3 + …

とすると、

c_0 = d_0
c_1 = -d_1
c_2 = d_2
c_3 = -d_3
c_4 = d_4



となっています。

(1 + x)^(1/2) = 1 + a_1*x + a_2*x^2 + a_3*x^3 + …

とすると、

(1 + a_1*x + a_2*x^2 + a_3*x^3 + …) * (1 + a_1*x + a_2*x^2 + a_3*x^3 + …) = 1 + x + 0*x^2 + 0*x^3 + …

なので、

(1 - a_1*x + a_2*x^2 - a_3*x^3 + …) * (1 - a_1*x + a_2*x^2 - a_3*x^3 + …) = 1 - x + 0*x^2 - 0*x^3 + … = 1 - x

となることがわかります。
0125132人目の素数さん
垢版 |
2023/05/26(金) 08:47:44.39ID:MYtUGoiY
今回のSpringerのセールで結局、合計28冊も買うことになってしまいました。
0126132人目の素数さん
垢版 |
2023/05/26(金) 08:55:17.97ID:MYtUGoiY
作り付けの本棚にスペースを確保するために、本を作り付けでない本棚に移動しなければならなくなりました。

もう当分、本を買うのはやめようと思います。
0127132人目の素数さん
垢版 |
2023/05/26(金) 11:58:15.12ID:Ms8IQrru
2変数関数のsupもしくはinfの取る順番によって結果が変わる例ってあります?
0128132人目の素数さん
垢版 |
2023/05/27(土) 01:02:30.06ID:c7M6RGHY
x,yを一次独立な2次の実列ベクトルとし、A=xy^tで定める。(ただしA≠A^t)
B=ab^t (a,bは2次の非ゼロの実列ベクトル)としたとき B-cA(c∈ℝ)が対称行列となるcが存在し、
c=±√(det(B-B^t)/det(A-A^t))
であることを示せ
0129132人目の素数さん
垢版 |
2023/05/27(土) 01:02:37.09ID:c7M6RGHY
お願いします
0130132人目の素数さん
垢版 |
2023/05/27(土) 01:30:45.44ID:7ZR1eNKE
>>128
まず表記的にc=±√なんちゃらと書いてるけど、それは微妙
たかが2×2行列の話だから成分で書き下して、対称行列の条件である非対角成分が等しいとおけばcは単なる1次式を解くだけで決まる
具体的にはx=(x1,x2)等と書くことにすればc=(a1b2-a2b1)/(x1y2-x2y1)
そこからdet(B-B^t)=(a1b2-a2b1)^2等を使って問題の表記に至る
あえてA,Bで書くことで±表記が必要になっていて良くない表示だと思うし、この形で一般化があるわけでもなさそう
0131132人目の素数さん
垢版 |
2023/05/27(土) 09:36:46.32ID:oxQLkCIC
まぁなんとか教科書レベルの問題解けるレベルくらいには至ってるんやろうけど、他の人にキチンと伝わる問題文作れるレベルにはまだ到達できてないんやろな
0132132人目の素数さん
垢版 |
2023/05/27(土) 10:09:46.11ID:bjEdbi8+
B - cA = (B - cA)^t を書き換えると B - B^t = c(A - A^t).
A - A^t, B - B^t は反対称で2次反対称行列は1次元分しかない(スカラー倍を除いて一意)が、今 A - A^t は0でないのでこの式を満たすcが存在する。
両辺の行列式をとると det(B - B^t) = c^2 det(A - A^t) となる。

極力成分表示を使わないならこんな感じ。A, Bが特別な形である必要はない。
0133132人目の素数さん
垢版 |
2023/05/27(土) 15:52:09.50ID:c7M6RGHY
なるほどありがとうございます
0134132人目の素数さん
垢版 |
2023/05/30(火) 07:23:23.90ID:+3awVIQA
x,y,zの3方向を持つ立方体の
xが実数,yが虚数とすると、
zは何数ですか?
0135132人目の素数さん
垢版 |
2023/05/31(水) 11:45:54.75ID:mBq3P+p7
3元数
0136132人目の素数さん
垢版 |
2023/05/31(水) 14:43:41.89ID:a/hvs5y5
https://imgur.com/6QI0Hal.jpg

↑Springerから注文していた本が送られてきました。

まだ届いていないのは、Courant & Johnの『Introduction to Calculus and Analysis II/2』だけです。
この本だけなぜかドイツから来るようです。
0137132人目の素数さん
垢版 |
2023/05/31(水) 14:46:36.06ID:a/hvs5y5
Tuさんの多様体の本ですが、以前、ソフトカバーのものを購入済みで、今回2冊目を買ったのですが、
なぜか、今回買ったもののほうが安っぽいです。

価格に応じてクオリティーを変えているなんてことは考えにくいですよね。

セールだからということではなく、改悪されたのだと思います。
0138132人目の素数さん
垢版 |
2023/06/01(木) 09:19:11.60ID:NY6dElCV
>>136

なぜか、アポストルの解析的整数論の本のサイズが異常に大きかったです。
0139132人目の素数さん
垢版 |
2023/06/01(木) 16:11:47.42ID:NY6dElCV
>>138

アポストルの解析的整数論の本はハードカバーの中古本を既に持っていたのですが、安かったので
新品のソフトカバーを買いました。

ハードカバーのほうは、普通のサイズの本です。

Springerは本のサイズをどうやって決定しているんですかね?

ハードカバーのPughのReal Mathematical Analysisも普通のサイズよりも大きいです。
0142132人目の素数さん
垢版 |
2023/06/01(木) 19:36:56.75ID:NY6dElCV
今読んでいる本に、

---

X が空集合であるときに、 X 上の空間系 ~ は同値関係になる。
空集合には元が存在しないので、商集合 X / ~ は空集合となる。

---

と書かれています。また、

---

集合 X 上に同値関係 ~ があるとき、 X の部分集合

[x] := {y ∈ X : y ~ x}

を x の属する同値類という。

---

とも書かれています。

X が空集合のときに、 {y ∈ X : y ~ x} 内で使われている X の元 x についてはどう考えればいいのでしょうか?
0143132人目の素数さん
垢版 |
2023/06/01(木) 19:46:13.21ID:NY6dElCV
X を空集合とする。
x ∈ X とする。

Aさんは次のように考えました。
X は空集合なのだから、 y ∈ X をみたす y は存在しないから、
{y ∈ X : y ~ x} = 空集合
である。

Aさんの主張はどこが間違っているのでしょうか?
0144132人目の素数さん
垢版 |
2023/06/01(木) 20:11:32.11ID:/6ahWlsh
>>142
空でしょ
0145132人目の素数さん
垢版 |
2023/06/01(木) 20:12:02.80ID:/6ahWlsh
てか空の元でしょ
0146132人目の素数さん
垢版 |
2023/06/01(木) 20:13:16.90ID:/6ahWlsh
>>143
>Aさんの主張はどこが間違っているのでしょうか?
間違ってないのでは?
0147132人目の素数さん
垢版 |
2023/06/01(木) 20:14:47.72ID:/6ahWlsh
あそうかxが存在しないから{y∈X|x〜y}も存在しないか
0148132人目の素数さん
垢版 |
2023/06/01(木) 20:50:06.08ID:wmAZtPxY
>>142
関係の集合による定式化を明記していないのが曖昧模糊になっている理由
それを書けばよい
0149132人目の素数さん
垢版 |
2023/06/01(木) 20:57:04.45ID:NY6dElCV
X = 空集合とする。
x ∈ X とする。
{y ∈ X : (y, x) ∈ 空集合} = 空集合?
0150132人目の素数さん
垢版 |
2023/06/01(木) 21:07:15.84ID:mJ7dqO4e
Xを空集合とは限らない集合としたときの、X上の関係の定義を書いてみよ。
そこをはしょるから訳がわからなくなる。
0151132人目の素数さん
垢版 |
2023/06/01(木) 21:07:46.19ID:8QCDEEbr
きっちり束縛記号書かんからわからんのやろ
人には書き方がいい加減だなんだガタガタいうくせに自分がなんか考えてる時にはこのグダグダ
アホですか
0152132人目の素数さん
垢版 |
2023/06/01(木) 21:24:40.45ID:NY6dElCV
>>150

R を X × X の部分集合とする。
(x, y) ∈ R であるとき、 x ~ y と書く。
~ を R が定める関係という。
0153132人目の素数さん
垢版 |
2023/06/01(木) 21:32:48.58ID:NY6dElCV
[x] := {y ∈ X : y ~ x} for x ∈ X
X / ~ := {[x] : x ∈ X}

X = 空集合であるとき、

X / ~ = 空集合 or X / ~ = {空集合} ?
0154132人目の素数さん
垢版 |
2023/06/01(木) 21:47:06.70ID:TIdtkjYJ
>>152
それで?Xが空のときRはどうなるんだ?

これすらわからんのに測度だの多様体だのアホかよ
そんなもんやるくらいなら高校数学の論理でも復習した方が遥かに有益
0155132人目の素数さん
垢版 |
2023/06/01(木) 22:03:12.22ID:ZM1NCp9d
>>152
関係と同値関係の混同が見られる。
0156132人目の素数さん
垢版 |
2023/06/01(木) 22:12:02.40ID:0Qz+63PP
まぁこういうのがわからんというあのは初学者あるあるなんだが、このレベルであるにも関わらず「自分は頭がいい」と思ってるのが信じられん
0157132人目の素数さん
垢版 |
2023/06/02(金) 01:23:41.03ID:VzTmQA35
質問:以下の理解でおk?

Xを位相空間とする。A⊆Xとする。
Aが可分 ⇔ ∃可算B⊆A (AにおけるBの閉包)=A
Aにおける閉包はXにおける閉包cl(B)をAに制限したものだから、結局、∃可算B⊆A A⊆cl(B)

↑これでおk?
0158132人目の素数さん
垢版 |
2023/06/02(金) 02:03:43.47ID:QSFC3j/y
>>155

意味不明です。混同などしていないと思います。

X が空集合のときには、その唯一の部分集合である R も空集合になります。
そして、 R が定める空な関係 ~ は同値関係になっています。
0159132人目の素数さん
垢版 |
2023/06/02(金) 02:04:27.26ID:QSFC3j/y
訂正します:

>>155

意味不明です。混同などしていないと思います。

X が空集合のときには、 X × X の唯一の部分集合である R も空集合になります。
そして、 R が定める空な関係 ~ は同値関係になっています。
0160132人目の素数さん
垢版 |
2023/06/02(金) 02:06:16.14ID:QSFC3j/y
>>154

X が空集合であるときには、 X × X は空集合、そして、その唯一の部分集合 R も空集合です。

何が言いたいのでしょうか?
0161132人目の素数さん
垢版 |
2023/06/02(金) 05:30:01.72ID:t8mgSWhB
>>160
それがわかっていれば>>153のような疑問が湧く筈がない
0163132人目の素数さん
垢版 |
2023/06/02(金) 06:37:39.68ID:550N8k2S
>>153
>X / 〜 = 空集合
こっち
0165132人目の素数さん
垢版 |
2023/06/02(金) 07:28:29.40ID:t8mgSWhB
>>162
普通可分ってX=Aのときぐらいしか使わないけど、Aが真部分集合の場合ってどのようなシチュエーションで使われるの?
0166132人目の素数さん
垢版 |
2023/06/02(金) 08:50:43.49ID:QSFC3j/y
>>161
>>163

あ、勘違いしていました。

X が空集合であるときに、 {[x] : x ∈ X} が空集合であるのは、、
X が空集合であるときに、 {x : x ∈ X} が空集合であるのと同じ理由からですね。
0167132人目の素数さん
垢版 |
2023/06/02(金) 10:50:50.86ID:x6jbsKU6
散々教科書の誤植レベルのミスをあげつらって著者を馬鹿にしてたのに、自分の(誤植レベルではない)ミスはただの勘違言って矮小化したいんですね
0168132人目の素数さん
垢版 |
2023/06/02(金) 10:51:33.23ID:x6jbsKU6
>ただの勘違言って
ただの勘違いと言って
0171132人目の素数さん
垢版 |
2023/06/08(木) 02:13:28.37ID:ahNfjg5L
(X,<)を全順序、c.c.c.,可分とする。(←これが必要か分からん)
x,y∈Xに対して、x~y ⇔ x=y or x<y&(x,y)可分 or x>y&(y,x)可分 と定義する。
x~yが同値関係となることを示したい。

x~yが推移律を満たすことを示したいが分からん
0174132人目の素数さん
垢版 |
2023/06/08(木) 14:59:07.95ID:/tocbAQA
Cを有界な閉凸錐としたときCの端点全てからなる集合Tに対して
conv(T)=Cとなる理由が分かりません。
明らかにT⊆Cより conv(T)⊆conv(C)=Cなのは分かるのですが、逆にconv(T)⊇Cであることはどのように示したら良いですか??
0175132人目の素数さん
垢版 |
2023/06/08(木) 15:00:04.28ID:/tocbAQA
conv(A)はAの凸包です
0176132人目の素数さん
垢版 |
2023/06/08(木) 15:01:55.98ID:/tocbAQA
すいません、Cを有界な閉凸錐と書いてしまったのですが、閉凸集合が正しいです。
連投になってしまい申し訳ありません。
0177132人目の素数さん
垢版 |
2023/06/08(木) 16:01:18.62ID:/ym0G46B
https://math.mit.edu/~yyao1/pdf/2023_regular_season.pdf
これの(13)を解いてほしいです。
0178132人目の素数さん
垢版 |
2023/06/08(木) 16:12:13.10ID:duDZ3Hrr
bee用にしては難しいな
0179132人目の素数さん
垢版 |
2023/06/08(木) 16:45:39.24ID:PxL+2R8T
x∈Cを自由にとってxを通る直線lを任意にとる
l ∩ C はlの線分でその両端点はTの元
0180132人目の素数さん
垢版 |
2023/06/08(木) 16:52:20.66ID:DUgTTha8
>>174
クレインミルマンで検索
選択公理を使う
0181132人目の素数さん
垢版 |
2023/06/08(木) 17:53:57.59ID:Z79Wx+KW
梅原雅顕、一木俊助著『これからの集合と位相』に、

集合 A, B, C, D が、 |A| = |C| と |B| ≦ |D| をみたすと

|B^A| ≦ |D^C|

が成り立つと書いてあります。

B = 空集合
A = 空集合
D ≠ 空集合
C = 空集合

であるとき、

B^A = {空写像}
D^C = {}

なので、

B^A から D^C への写像は存在しません。

したがって、

B^A から D^C への単射も存在しません。

したがって、

|B^A| ≦ |D^C|

は成り立ちませんよね?
0182132人目の素数さん
垢版 |
2023/06/08(木) 18:00:48.22ID:Z79Wx+KW
あ、

B^A = {空写像}
D^C = {空写像}

なので、成り立ちますね。
0183132人目の素数さん
垢版 |
2023/06/08(木) 18:15:56.29ID:Z79Wx+KW
梅原雅顕、一木俊助著『これからの集合と位相』に、

集合 A, B, C, D が、 |A| = |C| と |B| ≦ |D| をみたし、 B が空集合であるときに、

|B^A| ≦ |D^C| が成り立ち、等号は D が空集合のときに限る

と書いてあります。

B = 空集合
A = 空集合
D ≠ 空集合
C = 空集合

であるとき、

B^A = {空写像}
D^C = {空写像}

なので、

|B^A| = |D^C|

が成り立ちますよね。

これは、間違った記述ですね?
0185132人目の素数さん
垢版 |
2023/06/08(木) 18:25:05.92ID:Z79Wx+KW
梅原雅顕、一木俊助著『これからの集合と位相』に、

集合 A, B, C, D が、 |A| = |C| と |B| ≦ |D| をみたすと

|A^B| ≦ |C^D|

が成り立つと書いてあります。

A = C = 空集合
B = 空集合
D ≠ 空集合

であるとき、

|A^B| = 1 > 0 = |C^D|

なので、一般には成り立ちませんよね。

間違いをまた発見してしまいました。
0186132人目の素数さん
垢版 |
2023/06/08(木) 19:29:45.84ID:uO73FZXG
お前が発見しなければならないのは間違い探しがやめられない自分の魂の卑しさ
しかし永遠にみつけられんやろ
0188132人目の素数さん
垢版 |
2023/06/08(木) 20:17:02.02ID:yLj3VaX1
>>177
この書き方どうなの?
sinx
sin(x+sinx)
sin(x+sin(x+sinx))
の極限のことよね
s[n+1](x)=sin(x+s[n](x))
なんでしょ?
・・・を使うんなら
・・・+sin(x+sin(x+sinx))・・・
じゃないの?
0189132人目の素数さん
垢版 |
2023/06/08(木) 20:26:26.35ID:yLj3VaX1
マイナスか
0190132人目の素数さん
垢版 |
2023/06/09(金) 01:12:11.95ID:6LW2TQ9E
Σ[k=0,m] (-1)^k *C[k+2n-1,k]*C[n,m-k]

wolframによるとこの和は -C[2n-1,n-1]*(m-n)*C[n,m]/(m+n) となるのですが
これはどのように計算すると得られますか
0192132人目の素数さん
垢版 |
2023/06/09(金) 06:42:11.40ID:LIzUf2XD
これは₂F₁なのでガウスの超幾何定理だけで済むタイプ
別スレで出てた₃F₂が出てくるとSaalschützの定理とかDixonの公式とか使わないといけなくなる
0193132人目の素数さん
垢版 |
2023/06/09(金) 08:11:51.60ID:DKKH2JOZ
>>184
|x-y|≦π/2はどう示すの?
0195132人目の素数さん
垢版 |
2023/06/09(金) 08:46:32.08ID:/7/cJ7Xz
0<x<π/2 においてy = sin(x - sin(x-..)..) は方程式
y = sin( x-y ) の解で一意に決まる
x-yは方程式x-(x-y) = sin(x-y)の解で
x - 0 > sin(0), x -π/2 < sin(π/2)
だから0<x-y<π/2
0196132人目の素数さん
垢版 |
2023/06/09(金) 08:46:38.00ID:DKKH2JOZ
>>194
0<x<π/2+1ね
0197132人目の素数さん
垢版 |
2023/06/09(金) 08:48:43.87ID:DKKH2JOZ
>>195
π/2<x<π/2+1の部分は?
0199132人目の素数さん
垢版 |
2023/06/09(金) 09:03:04.49ID:DKKH2JOZ
>>198
[0,π/2+1]だよ
0200132人目の素数さん
垢版 |
2023/06/09(金) 09:03:15.87ID:/7/cJ7Xz
積分区間0<x<π/2+1か
でも一緒

x-yは方程式x-(x-y) = sin(x-y)の解で
x - 0 > sin(0), x -π/2< sin(π/2+1)
だから0<x-y<π/2

tu平面で
u = x - t、u = sin(t)
の交点と考えれば0<t<π/2とすぐわかる
0201132人目の素数さん
垢版 |
2023/06/09(金) 09:06:43.28ID:/7/cJ7Xz
つまりはこの直線とsinカーブがちゃんと0<t<π/2で交点持つようにxの範囲決めてるんやな
超えてもできるだろうけど煩雑なしょうもない作業増えるだけ
逆にちゃんと理屈わかってないと-π/2<x-y<π/2に収まっててasinを噛ませられる事の論述で引っかかるようにしてあるんやな
0202132人目の素数さん
垢版 |
2023/06/09(金) 09:28:56.41ID:DKKH2JOZ
>>201
なるほど
ここから先も交点は1つだけど
戻ってくるからx=t+π-arcsintとかになるわけね
0203132人目の素数さん
垢版 |
2023/06/09(金) 12:55:54.25ID:5S65eY71
y = sin(x-sin(x-sin(x-...)))
y = sin(x-y)
dy/dx = cos(x-y)/(1+cos(x-y))
dx = 1+1/cos(x-y) dy
また x = y + Arcsiny

∫[π/2+1,0] y dx = ∫[1,0] y dy +∫[1,0] y/cos(x-y) dy
= 1/2 +∫[1,0] tan(x-y) dy = 1/2 + ∫[1,0] tan(Arcsiny) dy = 1/2 +∫[1,0] y/√(1-y^2)dy
= 1/2 +∫[π/2,0] sint dt = 3/2
0204132人目の素数さん
垢版 |
2023/06/09(金) 13:00:27.79ID:cTDQGYUG
x = y + acos(y)だから
∫[0,π/2+1]ydx = ∫[0,1] y(1+asin'(y))dy
でxには早々に退場いただく方が好き
0205190
垢版 |
2023/06/09(金) 14:09:21.02ID:6LW2TQ9E
まちがえました
>>190の式は

誤 Σ[k=0,m] (-1)^k *C[k+2n-1,k]*C[n,m-k]
正 Σ[k=0,m] (-1)^k *C[k+2n-1,k+n]*C[n,m-k]

でした。しみません。宜しくお願いします。
0206132人目の素数さん
垢版 |
2023/06/09(金) 15:08:47.18ID:tjuh6K7O
>>204
当然こっち
0207132人目の素数さん
垢版 |
2023/06/09(金) 15:09:35.88ID:tjuh6K7O
x=y+arcsinyね
0208132人目の素数さん
垢版 |
2023/06/09(金) 15:12:32.35ID:tjuh6K7O
あと極限の存在は別途述べねばなるまいか
0209132人目の素数さん
垢版 |
2023/06/09(金) 16:20:53.19ID:cgm7XP53
@各項を(a+k)!,(a-k)!で表示
A(a+k)! = (a+1)ₖa!
 (a-k)! = a!/( a〜a-k+1) = (-1)ᵏa!/(-a)ₖ
で各項を(a)ₖで表示
B = ₂F₁(◯,△;□;1)なり=₃F₂(◯,△,□;☆,*;1)なり
C超幾何定理なり、Dixonの公式なり
0210132人目の素数さん
垢版 |
2023/06/09(金) 17:08:12.82ID:6LW2TQ9E
C[2n-1+k,n+k]=(2n-1+k)!/((n+k)!(n-1)!) で、
 (2n-1+k)!=(2n-1)!*(2n)_k ,  (n+k)!=n!*(n+1)_k

C[n,m-k]=n!/((m-k)!(n-m+k)!) で、
 (m-k)! = (-1)^k*m!/(-m)_k ,  (n-m+k)! =(n-m)!*(n-m+1)_k

で、(与式)=C[2n-1,n]*C[n,m]*sum( ((2n)_k*(-m)_k)/((n+1)_k*(n-m+1)_k )

まで進んだのですがここまであってますか。あと、このあと超幾何に持ち込ぬには
0211132人目の素数さん
垢版 |
2023/06/09(金) 17:17:49.62ID:t1aLUJTR
Σ (2n)ₖ(-m)ₖ(1)ₖ/( (n-m+1)ₖ(n+1)ₖ )
=₃F₂( 2n, -m, 1; n-m+1,n+1; 1 )

にPfaff-Saalschütz Theorem

₃F₂(a,b,-n; c,1+a+b-c-n;1)
= (c-a)ₙ(c-b)ₙ /( (c)ₙ(c-a-b)ₙ )

を適用
0212132人目の素数さん
垢版 |
2023/06/09(金) 17:25:57.91ID:t1aLUJTR
分母k!忘れた
エスパーして
0213132人目の素数さん
垢版 |
2023/06/09(金) 18:39:31.52ID:6LW2TQ9E
答えデタ━━━━(゚∀゚)━━━━!!

こんなにみずかしい問題だったのか
0214132人目の素数さん
垢版 |
2023/06/09(金) 18:44:32.13ID:6LW2TQ9E
となみに、上では3F2に持ち込みましたが
>>209 では 2F1に持ち込むことができるらしそうな書き込みでしたが
どんなマジックな手法を使うのでしょうか
0215132人目の素数さん
垢版 |
2023/06/09(金) 18:53:52.62ID:t1aLUJTR
あなたが間違って書いた問題なら₂F₁に持ち込めます
訂正後のやつは無理
0216132人目の素数さん
垢版 |
2023/06/09(金) 18:56:50.88ID:6LW2TQ9E
そうでしたか。ありがとうございました。
0217132人目の素数さん
垢版 |
2023/06/09(金) 23:52:43.15ID:6LW2TQ9E
答え出たと思ったけど
ぷふぁふ-ざーるしゅっつとかの定理の証明をせんとあかんのね
0218132人目の素数さん
垢版 |
2023/06/10(土) 00:12:54.73ID:V3/1Yzvu
可分 と 順序位相 に関係した命題を確認させてくれ
どこに載ってる?
0220132人目の素数さん
垢版 |
2023/06/10(土) 07:30:09.68ID:NiZs46bd
リーマン積分可能な関数列 (f_n) が f に区間 [a, b] で一様収束するならば、

∫ f(x) dx = lim ∫f_n(x) dx

が成り立つ。

ルベーグ積分で考えるとこの「一様収束」という条件を「一様有界」という条件に置き換えられるということですか?
0221132人目の素数さん
垢版 |
2023/06/10(土) 07:40:59.58ID:ZsbUuNGT
超幾何定理系の公式は今も盛んに研究されてて話によるとwolframのまとめサイトには10000個くらいの公式が載ってるそうな
流石の専門家も全部知ってるとかありえない
とはいえ代表的なやつは勉強しといた方がいいかもな、この方面目指すならば絶対
超幾何定理は₂F₁(a,bc;1)がいつでも計算可能と言ってる全ての基本、コレは絶対不可避
Pfaff-Saalschütz Theoremは₃F₂(a,b,c:d;e;1)の5次元の内自由度4(上1個は負の整数でないとダメだから3.5次元くらいの感じか?)だからかなり強力
この辺までは抑えといた方がいいんかもしれん
しかしもはや全部抑えるのは無理なのでどこまで勉強しといた方がいいのかは流石にこの方面の専門家のいる大学の先生にお話聞かんとわからんやろな
0222132人目の素数さん
垢版 |
2023/06/10(土) 10:23:20.75ID:b7eIpwQE
n≧1のとき、
 sum_[k=0,n](-1)^k*C[n,k]*(ak+b)^(n-1)
が 0 になるのは明らかなんでしょうか。
0223132人目の素数さん
垢版 |
2023/06/10(土) 11:42:37.13ID:gSSl5z9j
>>221
> 10000個くらいの公式
計算問題だね
研究業績として認めてはいけまい
0224132人目の素数さん
垢版 |
2023/06/10(土) 17:45:08.93ID:21TL+xKy
>>222
読み手による
般教向けの教科書、般教の試験の解答なら“明らか”は通用しない
けどそれ以降なら“明らか”はともかく“容易”で済まされて文句言えない
0225132人目の素数さん
垢版 |
2023/06/10(土) 19:40:40.42ID:b7eIpwQE
え〜
そんなに容易いんですか
0226132人目の素数さん
垢版 |
2023/06/10(土) 20:04:55.06ID:ZsbUuNGT
般教の数学、つまり数学の専門家を目指してるわけではない人間ならともかく、専門課程まで進んで数学の専門家を名乗るつもりならこんなのに手こずってる場合じゃないやろ
実質 主張は

0≦m<nのとき
ΣₙCₖ (-1)ᵏkᵐ = 0

からすぐ出るしそれは

(1-x)ⁿ = Σ(-x)ᵏₙCₖ

の両辺0〜n-1階微分してx=1代入して終わり
ほとんど定石の範囲内
0227132人目の素数さん
垢版 |
2023/06/10(土) 20:20:21.93ID:DmhdgHjA
詳しく教えて
0228132人目の素数さん
垢版 |
2023/06/10(土) 21:58:43.97ID:b7eIpwQE
経済学部生なので許してください
0229132人目の素数さん
垢版 |
2023/06/11(日) 00:22:22.82ID:4QyNAw4Z
~なので
~だから

~と仮定する。
~を仮定する。


ん~、文章書いてて、自分でも無意識に混在させてることに気付いた。
後から文章を訂正するにしても、どういう基準でどっちに統一させるべきかで一々無駄に悩むww
0230132人目の素数さん
垢版 |
2023/06/11(日) 09:12:08.65ID:sCgEWZUA
北斗無双とベルセルク無双のあたったときのあたり回数の違いは?計算できる?
0231132人目の素数さん
垢版 |
2023/06/11(日) 09:17:11.46ID:sCgEWZUA
>>230
1回当たったら、75%で次も当たるパチンコがあるとします。

1回当たったら、80%で次も当たるパチンコがあるとします。

この2台で、それぞれ5回以上、10回以上、15回以上あたる確率はどの程度違うか
計算できるものでしょうか?

また、それぞれの平均当たり回数が計算できますか?
0233132人目の素数さん
垢版 |
2023/06/11(日) 09:31:29.89ID:sCgEWZUA
こんな問題も計算て
0234132人目の素数さん
垢版 |
2023/06/11(日) 09:35:13.05ID:sCgEWZUA
こんな問題は数学板なら、ササッと計算できるかと思ったか
確率はスレタイと違う専門外だからわからんのか
意外とパチ板とレベルかわらんのかもな。
返信ありがとう
0236132人目の素数さん
垢版 |
2023/06/11(日) 15:25:08.56ID:PzX5aj63
固有ベクトルを求めるとき
「任意の実数」と「任意の定数」の違いがよくわかりません
「kは任意の実数」とか「cは任意の定数」とか、
問題によってあるいは本によって違いがあってその違いを教え江下さい
0237132人目の素数さん
垢版 |
2023/06/11(日) 20:14:11.77ID:FkJ8tWlg
違いは特にないと思います
0238132人目の素数さん
垢版 |
2023/06/11(日) 20:27:38.72ID:rpXMqp+2
xの多項式f_n(x),g_n(x)が
f_1(x)=x
g_1(x)=-1
・(n+1)(f_{n+1}(x)-f_n(x))=x(g_n(x)+f'_n(x))
・(n+1)(g_{n+1}(x)-g_n(x))=x(f_n(x)-g'_n(x))

をみたすときlim[n→∞]f_n(x)を求めよ

すいません、わからないので教えていただきたいです。お願いします
0239132人目の素数さん
垢版 |
2023/06/11(日) 21:02:54.93ID:rpXMqp+2
>>238

xの多項式f_n(x),g_n(x)が
f_1(x)=x
g_1(x)=-1
・(n+1)(f_{n+1}(x)-f_n(x))=-x(g_n(x)+f'_n(x))
・(n+1)(g_{n+1}(x)-g_n(x))=x(f_n(x)-g'_n(x))

をみたすときlim[n→∞]f_n(x)を求めよ

すいません、見直したら第3式の-が抜けてました…
0241132人目の素数さん
垢版 |
2023/06/11(日) 23:28:29.58ID:rpXMqp+2
>>240
何項か計算してエスパーしましたか??
ぱっと見でわかったりできるもんなんですかね…
0242132人目の素数さん
垢版 |
2023/06/11(日) 23:32:33.20ID:UAI4W1RB
何項か計算
天才はパッと見た目でわかるかもしれんが天才ではないのでわからない
なれっこないものの話しても仕方ない
0243132人目の素数さん
垢版 |
2023/06/12(月) 03:08:26.85ID:QMvufrvq
clを位相空間の閉包作用素とする

cl(∪Ai)=∪cl(Ai)って成り立たないよな?
0244132人目の素数さん
垢版 |
2023/06/12(月) 03:17:41.61ID:Fi60UATX
I = ℚ、Aᵢ = { i } ( i∈I = ℚ )
cl(Aᵢ) = cl( { i } ) = { i }
∪cl( Ai ) = ∪{ i } = ℚ
cl( ∪Aᵢ ) = cl( ∪{ i } ) = cl(ℚ) = ℝ
0245132人目の素数さん
垢版 |
2023/06/14(水) 21:58:33.37ID:Y/m3mJyV
微分方程式解くとき
唐突にf(x,t)=g(x)h(t)みたいな変数で分離し始めることあるけど
そうしていいとする理由ってどう考えればいいんですか?
f(x,t)=(xt+1)sinxtみたいになってたらどうするんだろうっていつも考えちゃいます
0246132人目の素数さん
垢版 |
2023/06/14(水) 23:51:51.27ID:OiKgtsVw
>>245
はぁ
テイラー展開でChatGPT
0247132人目の素数さん
垢版 |
2023/06/15(木) 16:49:43.77ID:5O56xtXP
関数空間でない空間でのコンパクト収束位相とはどういう意味なのでしょうか

既に位相が入っている空間X(距離空間でもある)のある種の部分集合の属に対して
the topology of uniform convergence on compact sets
を入れるという記述が読んでいる本の中にありました
(具体的にはリーマン多様体の中の平坦な次元最大の部分多様体全体に対してこれで位相を入れると書かれています)
調べたら関数空間の場合はこの概念の定義があり、コンパクト開位相と同じものだという記述があったのですが
関数空間でない上のような部分集合族ではどう定義するのかが分かりません
知っている方いたら教えて下さい
0249132人目の素数さん
垢版 |
2023/06/15(木) 17:21:43.19ID:5O56xtXP
>>248
Gromov他のmanifolds of nonpositive curvatureという本のp.158やp.254ですが
いきなり出てきたのでたぶん見てもこれ以上の情報はないかと思います
0250132人目の素数さん
垢版 |
2023/06/15(木) 17:54:15.23ID:X4YLW5N3
凸包coと閉包clについてS∈ℝ^nでco(cl(S))⊆cl(co(S))なのはわかるのですがSが閉包ならばco(cl(S))⊇cl(co(S))も言える理由を考えてるのですが分かりそうでわかりません…
多分、有界閉集合Tに関してco(T)も閉集合になるのだろうとは思ったのですがどうのように示せば良いかが分かってません…
0251132人目の素数さん
垢版 |
2023/06/15(木) 19:25:24.23ID:GxY+7p2f
http://math.caltech.edu/Convexity.html
自著の紹介だがサンプルとして無料アップされている章にあなたの知りたいことが書いてあると思われる
0252132人目の素数さん
垢版 |
2023/06/15(木) 19:28:07.92ID:rWswYJ1v
x ∈ cl(co(T)) なら x の 1/n 近傍に x_n ∈ co(T) がある
x_n ∈ co(T) なら x_n ∈ T か x_n1, x_n2 ∈ T で x_n ∈ co({x_n1, x_n2})
有界閉集合なら x_n, x_n1, x_n2 に集積点がある
てな具合でどう?
0253132人目の素数さん
垢版 |
2023/06/15(木) 19:52:47.69ID:IU2J5P7Q
>>249
一ヶ月前も質問を投下して何も解決しなかったのにそれから90頁進んだことにしてまた質問投下とか馬鹿だな。
早くしね。
0255132人目の素数さん
垢版 |
2023/06/15(木) 20:06:58.89ID:q8hLLOvo
ζ_n:1のn乗根、Z(ζ_n)⊂Q(ζ_n):円分体の整数環としたとき、
Z(ζ_n)がUFDとなるnがどのようなものかわかりますか。
また、具体的にn=3,5,6の場合はUFDになりますか。
0256132人目の素数さん
垢版 |
2023/06/15(木) 20:19:51.90ID:5O56xtXP
なんでこんなに怒ってるのかと思ったけど>>15を言ったのが自分だと勘違いしてるのか
自分じゃないので悪しからず
0257132人目の素数さん
垢版 |
2023/06/15(木) 20:29:20.97ID:2RBfZAkR
>>255
とりあえず二次体の場合はググれは虚二次体の類数は山ほどでてくる
ℚ(√(-3))はUFD、ℚ(√(-5))はダメ
0259132人目の素数さん
垢版 |
2023/06/15(木) 20:49:03.75ID:X4YLW5N3
>>252
なるほど!なんとなく理解しました!ありがとうございます!
0260132人目の素数さん
垢版 |
2023/06/15(木) 20:56:50.19ID:IxORTEb3
>>250
そもそも有限次元とか仮定入れないと無理やろ
可算無限次元の集合
p⁺ₙ=(第0成分が1/n第n成分が1/n,残りはゼロ)
p⁻ₙ=(第0成分が-1/n第n成分が1/n,残りはゼロ)
として閉集合
F = { p⁺ₙ, p⁻ₙ ; n∈ℕ }
を考える
Fの異なる2元間の距離は√2以上離れてるので閉集合
でもその凸包は第0成分のみ1/nである元を含むけど原点は含まれない
0261132人目の素数さん
垢版 |
2023/06/15(木) 21:20:01.78ID:5MB0FltT
イヤ、普通に反例あるやん
F = { (x,y) ∈ ℝ² | x>0, |y|≧x }
は閉集合、しかし凸包は{ x > 0 }で閉集合ではない
0262132人目の素数さん
垢版 |
2023/06/15(木) 21:23:39.79ID:7vmPC8ZN
>>261
閉?
0264132人目の素数さん
垢版 |
2023/06/15(木) 21:49:31.17ID:7vmPC8ZN
原点は?
0266132人目の素数さん
垢版 |
2023/06/15(木) 22:18:58.48ID:7vmPC8ZN
>>265
なるほどそれなら
0267132人目の素数さん
垢版 |
2023/06/15(木) 22:27:52.59ID:X4YLW5N3
すいません、

凸包coと閉包clについてS∈ℝ^nでco(cl(S))⊆cl(co(S))なのはわかるのですがSが閉包ならばco(cl(S))⊇cl(co(S))も言える理由を考えてるのですが分かりそうでわかりません…
多分、有界閉集合Tに関してco(T)も閉集合になるのだろうとは思ったのですがどうのように示せば良いかが分かってません…

の「Sが閉包ならば〜」とあるのですが、もともと「Sが有界ならば〜」でした。自分が写し間違えてました。
また、S⊆ℝ^nです。
色々書いてないことや間違いが多くてすいません。
0268132人目の素数さん
垢版 |
2023/06/15(木) 22:38:53.71ID:+OR7RzLI
空間が無限次元だと有界でもダメ
前にあげた無限次元の反例は有界、完備、でもコンパクトでないから反例になる
0269132人目の素数さん
垢版 |
2023/06/15(木) 22:45:30.08ID:+OR7RzLI
しかし有限次元ならaₙがコンパクト集合Kの凸包の集積点なら
aₙ=(1-tₙ)bₙ+tₙcₙ、tₙ∈[0,1], bₙ,cₙ∈K, Kがコンパクト、lim aₙ=a
ととれる
必要なら部分列を取り直してlim tₙ=t, lim bₙ = b, lim cₙ = cとしてよい
でa = (1-t)b+tcはKの凸包に入る
よってKの凸包は閉集合
0270132人目の素数さん
垢版 |
2023/06/15(木) 22:46:48.39ID:+OR7RzLI
書き方変やな
要するに次元が有限ならコンパクトの凸包は閉集合
有限次元なら有界閉集合はコンパクト
0271132人目の素数さん
垢版 |
2023/06/16(金) 02:30:01.18ID:qCmxl1VH
p∈co(S) ⇔ 有限個のSの点 a_1,…, a_k と和が1となる非負の数 t_1,…, t_k があって p = t_1a_1 +…+ t_ka_k と書ける、はOK?
さらにちょっと頑張ると、S⊆ℝ^n ならこのkがn+1で取れるということがわかる。
よってコンパクト集合の凸包はコンパクト。
0272132人目の素数さん
垢版 |
2023/06/16(金) 02:41:50.41ID:nloTYWwW
>>271
ようはカラテオドリの定理ですよね?
0273132人目の素数さん
垢版 |
2023/06/16(金) 10:07:52.91ID:D/alNrkF
f(x)は0≦x≦1で連続のとき
 | f(c) |>| ∫[0→c] f(x)dx |
をみたすc (0<c≦1)がある。

これ成り立ちますか。
0274132人目の素数さん
垢版 |
2023/06/16(金) 10:34:13.75ID:/tHff/4v
聞く前に10秒くらいは考えようよ
0275271
垢版 |
2023/06/16(金) 11:00:06.23ID:qCmxl1VH
>>272
名前付いてる定理だったのか。ありがとう
0276132人目の素数さん
垢版 |
2023/06/16(金) 15:08:30.77ID:mXUqYXsa
>>275
こちらこそ、わかりやすい解説ありがとうございました!
0277132人目の素数さん
垢版 |
2023/06/17(土) 02:18:00.63ID:B91T9P5h
医師になるのは、めちゃくちゃ簡単だよ。
どんな馬鹿医大でも国家試験の合格率7割以上はあるし、自治医大以上ならほぼ100%。

弁護士の場合は難関ロースクールを卒業しても、国家試験を通るのは10%程度。

医師になるには金と時間がかかるが、試験自体は簡単。
うちは従兄弟三人医師になったが、英検二級すら落ちるレベルの頭だからね。

医師国家試験の合格率ランキング見てみ。
一番低い杏林大学ですら、79.4%。

奈良県立大以上の偏差値の25校は95.0%超え。

これのどこが難関試験なの?
医学部に学費を支払える財力のハードルが高いだけで、医師にはバカでもなれる。

弁護士、司法書士、会計士、英検1級あたりは、バカには絶対に無理。

まとめると
医師国家試験→バカでも受かる。しかし、医学部6年間で1,000万以上かかる学費のハードルが高い。
司法試験→ロースクール卒業しても、合格できるのはごく一部。非常に難関な試験。
司法書士→ロースクールに行かなくても受験できるが、難易度は司法試験並み。
英検1級→英語がずば抜けて優秀でないと合格できない。英語の偏差値100必要。(実際にはそんな偏差値はないが)
会計士→おそらく、最難関試験か。会計大学院修了者の合格率は7.6%しかない。
不動産鑑定士→鑑定理論が地獄。単体の科目としては最難関の一つ。経済学などは公務員試験より簡単か。
0278132人目の素数さん
垢版 |
2023/06/17(土) 02:24:57.84ID:a9tSpCK8
医師試験はセンター試験の7割程度の勉強で行けるっていう噂は聞いたことある
0279132人目の素数さん
垢版 |
2023/06/17(土) 04:59:38.49ID:a9tSpCK8
ω_1:=アレフ(1)とする。
γ<ω_1が極限順序数ならば、γは単調増加な順序数列の極限となることを示せ
0281132人目の素数さん
垢版 |
2023/06/17(土) 05:06:07.71ID:a9tSpCK8
記号で表すならば、
∀γ<ω_1∃(α_n;順序数)_{n∈N} ∪α_n=γ
0282132人目の素数さん
垢版 |
2023/06/17(土) 05:11:38.77ID:a9tSpCK8
訂正

記号で表すならば、
∀γ<ω_1∃(α_n;順序数)_{n∈N} [ (α_n)は狭義単調増加 かつ ∪α_n=γ]
0284132人目の素数さん
垢版 |
2023/06/17(土) 08:38:43.44ID:WMYydSo2
ハゲてきたんだが
ハゲを数学的に定義することは
不可能じゃないかと思う。

髪の毛が全くなければハゲ、というならば
簡単かもしれないが、
髪の毛が多少残っていると―・・


あ!
新しい数学の分野を見つけたオレ!
0285132人目の素数さん
垢版 |
2023/06/17(土) 08:52:01.09ID:gjObeNYb
今までに経験した衝撃的な発見
・小学生の時、ハゲた先生の頭さわって、ツルツルかと思ったらウブ毛が生えててザラザラだった
・中学生の時、腹の出た先生の腹をたたいて、ポーンといい音がすると思ったら全然違った
0286132人目の素数さん
垢版 |
2023/06/19(月) 21:26:50.40ID:fqVR7hqN
【質問】
exp(-x^2) や sin(x^2) の不定積分が初等関数で表せられないというけど、
どうやって証明するんですか?

解析の本を調べましたが、証明が書かれてる本が見つかりませんでした。
証明が載っている本があれば教えて下さい。
0287132人目の素数さん
垢版 |
2023/06/19(月) 23:19:29.17ID:FXTV91oV
>>286

>>exp(-x^2) や sin(x^2) の不定積分が初等関数で表せられないというけど、
>>どうやって証明するんですか?

その主張はどの本に書いてありましたか?
0288132人目の素数さん
垢版 |
2023/06/19(月) 23:26:53.95ID:fqVR7hqN
>>287
事実としては微積分の色んな本に書いてあります。
例えば、杉浦解析入門を始め、工学系の教養の教科書にも書いてあります。
0289132人目の素数さん
垢版 |
2023/06/19(月) 23:30:56.73ID:FXTV91oV
>>288

>>初等関数で表せられない

この内容を正確に書いてありますか?
例えば
「初等関数の関数としては書けない」
という意味だとしたらウソになるでしょう。
0290132人目の素数さん
垢版 |
2023/06/19(月) 23:47:43.98ID:fqVR7hqN
>>289
それはどのように書けるのですか?
0291132人目の素数さん
垢版 |
2023/06/20(火) 00:26:14.40ID:op+5Lzbw
微分ガロアだね、和書だとあまりないけど微分体の理論に載ってる(ただしこれで微分ガロアに入門するのは色々キツいと思う)
「初等関数」の正確な定義を知らなくても初等関数の四則演算くらいは初等関数だと認められるだろうし、ガロア的な感じに議論できるだろうことは想像できるっしょ
0293132人目の素数さん
垢版 |
2023/06/20(火) 08:30:49.86ID:ERFLpFzC
すばらしい
0294132人目の素数さん
垢版 |
2023/06/20(火) 10:47:08.28ID:8XL/KZy2
>>291
そんな難しい理論を使わなくても、リュービルの定理だけで十分なので、その証明を読めば良い
だが、和書で書かれているのは知らない
0295132人目の素数さん
垢版 |
2023/06/20(火) 11:59:35.30ID:ERFLpFzC
>>294
偽物臭いぞ
0296132人目の素数さん
垢版 |
2023/06/20(火) 15:36:37.82ID:qzw1B6m7
リュービルによる証明は
位数が最小になる表現から出発して
それが1下げられることを示して矛盾を導くものであると
金子本には書かれている。
「できない」を「不可能」にバージョンアップするというコメントは
秀逸
0297132人目の素数さん
垢版 |
2023/06/20(火) 21:33:33.31ID:8XL/KZy2
>>295
偽物とはどういう意味か?

>>296
>「できない」を「不可能」にバージョンアップするというコメントは秀逸

「できない」と「不可能」の違いとは?
どちらも同じ意味では無いのか?
0298132人目の素数さん
垢版 |
2023/06/20(火) 21:39:32.10ID:ERFLpFzC
>>297
>>偽物とはどういう意味か?

リュービルというだけでは
たいていは関数論のリュービルの定理のことだと思われる

>>「できない」と「不可能」の違いとは?

「初等関数では表せない」というややあいまいな表現と
「このような仕方で構成できる関数の範囲には含まれない」
という形の明確な主張の違い
0299132人目の素数さん
垢版 |
2023/06/20(火) 21:40:21.05ID:7DcjvU1N
>>297
できないは能力が足りないだけ
不可能はどんな能力があっても無理
フェルマーの定理もつい最近までは
できないだけであったが今では不可能
様相論理で定義されてなかったっけ
0300132人目の素数さん
垢版 |
2023/06/20(火) 22:13:53.47ID:ERFLpFzC
>>299
>>フェルマーの定理もつい最近までは
>>できないだけであったが今では不可能

こういう雑な表現で満足できる者は
数学者を友人に持てない
0301132人目の素数さん
垢版 |
2023/06/20(火) 22:24:09.71ID:8XL/KZy2
>>298
この文脈でリュービルの定理を関数論の定理と思う方がどうかしている
不定積分の表示や微分ガロア理論とか出ているに


>>299
> できないは能力が足りないだけ

これは>>300が言うように、表現が雑
0302132人目の素数さん
垢版 |
2023/06/20(火) 22:41:16.77ID:ERFLpFzC
>>301
リュービルの証明のことを知らない者の質問に
「リュービルの定理」と言ってそのまま通じると思うか
0303132人目の素数さん
垢版 |
2023/06/20(火) 23:42:53.18ID:7DcjvU1N
>>300,301
雑で別に構わんが?
できないと不可能の違いは
不可能性の証明に尽きる
単にできないというのは
その能力が足りないだけ
0304132人目の素数さん
垢版 |
2023/06/20(火) 23:45:31.48ID:7DcjvU1N
>>300
>数学者を友人に持てない
友人には不足してないので
ご心配なきよう
0305132人目の素数さん
垢版 |
2023/06/21(水) 00:15:06.86ID:Va/Wfx5S
この赤線の部分が0ではなく1になるのはなぜですか?
0だと色々不都合が起きるのは分かるのですが…
https://i.imgur.com/6D5gzxt.jpg
0306132人目の素数さん
垢版 |
2023/06/21(水) 02:09:25.09ID:wn/367VJ
機体に穴があき酸欠状態で
僅か10分しかなく、必死で家族が待つ地球へ戻ろうとする様を描いています。
想像してみてください。//youtu.be/oWs3yvVADVg
0307132人目の素数さん
垢版 |
2023/06/21(水) 06:24:40.01ID:9RRcHEaJ
>>304
おまえは敵だと言われているのが分かる?
0308132人目の素数さん
垢版 |
2023/06/21(水) 07:39:37.61ID:NlNcSfmx
>>307
別に敵でもいいけど?
わけ分からんこと書く人だなあ
0309132人目の素数さん
垢版 |
2023/06/21(水) 07:48:40.46ID:9dCOJVga
「f(x)=|x|はx=0で微分できない」は嘘なんですか?頑張れば微分できようになるんすか?
0310132人目の素数さん
垢版 |
2023/06/21(水) 07:51:25.51ID:lXNutise
>>309
頑張ればできるかもしれないのと頑張ればできるのを混同するような奴は数学向いてない
0313132人目の素数さん
垢版 |
2023/06/21(水) 08:04:13.81ID:9RRcHEaJ
>>308
わからないんですね
0314132人目の素数さん
垢版 |
2023/06/21(水) 08:05:52.57ID:9RRcHEaJ
>>311
弱微分はヘルマンダーの本の第一章
0315132人目の素数さん
垢版 |
2023/06/21(水) 08:09:48.92ID:9RRcHEaJ
小松彦三郎でもよい
0316132人目の素数さん
垢版 |
2023/06/21(水) 08:18:30.06ID:vmdjnR+V
この流れ、学部1年のとき「~~とおく」「~~とする」の違いについて熱弁してた人思い出したわwwww
国語的な揚げ足取りばかりで数学的理解はボロボロだったな……
0317132人目の素数さん
垢版 |
2023/06/21(水) 08:32:45.99ID:9RRcHEaJ
>>316
それは掛け算の順序にうるさい人と似ている
0318132人目の素数さん
垢版 |
2023/06/21(水) 08:43:05.14ID:gLy3aLvI
掛け算の順序は別にあっても問題ないと思いますけどね
0319132人目の素数さん
垢版 |
2023/06/21(水) 09:02:56.38ID:WzZ+fxQf
>>316
自分で証明書いてると、「と置く」、「とする」が乱立してるのに気づいたら見たらめっちゃ気持ち悪いってことに自覚する
自分の中で整理できるレベルにまでたどり着け
0320132人目の素数さん
垢版 |
2023/06/21(水) 09:42:26.84ID:lOTfKEZq
 ゜ 。           。゜
             ゜。
  とおく とおく はなれていても

   すうがくが わかるように

力一Π 輝ける日を すうがくで迎えたい

  大事なのは 変わっていくこと

    変わらずに いる こと
゜。
   ○゜
   どんなに高いタワーからも

    見えない 僕の ふ る さ と 

  無くしちゃだめなことを いつでも

    胸に抱きしめているから

    ぼくの夢を かなえる場所は

     すうがくときめたから

   大事なのは わからずにいること

       わかっていくこと 
          
    。゜      ゜。○゜           ゜○゜
  
0321132人目の素数さん
垢版 |
2023/06/21(水) 09:42:37.22ID:9RRcHEaJ
>>319
そういうのが気持ち悪くなった経験は
誰にでもある
そういう枝葉末節にこだわらないことを
学ぶのは
案外一人だけでやっていると難しい
0322132人目の素数さん
垢版 |
2023/06/21(水) 09:48:04.52ID:lOTfKEZq
ホモろぅじぃゎ詩人、ホモろぅじぃゎ🌷ぽぇ夢
遠く悠く、うん、とおくだね!
   とする‥とする‥ ←だめだね!

とおく…とおく…,  ∞夢幻∞ Qed.

パパッと修了、ぉ仕舞ぃ!
0323132人目の素数さん
垢版 |
2023/06/21(水) 09:54:31.89ID:lOTfKEZq
とおく∨とする←ゎ、ぽぇ夢の領域だから
こ↑こ↓ろでみなくちゃ、わからなぃんだね!?
スゥゥ…楽人ッチャマたちが苦手な領域だね!?
🌈ホモろぅじぃ♂のポぇまーッチャマ🌠に習ぅと…
 ァルルェ~!?なんだかちょっぴり素敵だね!?
0324132人目の素数さん
垢版 |
2023/06/21(水) 10:03:22.09ID:lOTfKEZq
>>320
↑おっ、間違ぇたゾ↓

  力一Π輝ける日を このみちで迎えたい

            ↑だたゾ。

    とおくとおく 離れていても

     すうがくが わかるように

  力一Π輝ける日を このみちで迎えたい

だたゾ…
0325132人目の素数さん
垢版 |
2023/06/21(水) 10:07:56.96ID:lOTfKEZq
゜。    。
   ○゜
僕の夢をかなえる場所は このみちと きめたから


スゥゥ…楽やめたら 手が震ぇちゃぅからね、
しかたなぃね!
0326132人目の素数さん
垢版 |
2023/06/21(水) 10:13:11.73ID:lOTfKEZq
夢に全身全霊、命を投じてイノチの火を燃焼し尽くすんだよ!
 🌠アントキノイノチ🔥ファイヤー❗🔥
    🌈GOだよ!GO太!!🌈
光陰の矢の如く、光速F1で時と一体化して疾走するだよ!
0327132人目の素数さん
垢版 |
2023/06/21(水) 10:18:56.57ID:lOTfKEZq
GO太のイノチのスゥゥ…楽ビッグバン、もう始まってる!

って感じでぇ…
‥んまぁ、その‥にゃぴ‥、とおく∨とする
ぽぇ夢だからね、(適当)でイイネ!
0328132人目の素数さん
垢版 |
2023/06/21(水) 15:29:46.27ID:JHe2aSzv
>>312
多分このあとで微分方程式解く時積分でlog出てくるんですけどlogf(x)-logf(0)=e^x-1
で解答だと1じゃないとf(0)=0となって不都合が起きてる感じです
0330132人目の素数さん
垢版 |
2023/06/21(水) 15:47:42.47ID:JHe2aSzv
>>329
はい、自分もそうだと思うんですが確かに両辺n→∞とすると
f(x)=∫[0→x]e^yf(y)dy
f(0)=0
f'(x)=e^xf(x)
f'(x)/f(x)=e^x
logf(x)-logf(0)=e^x
↑ここで確かにf(0)=0だと嬉しくないので問題文に間違いがあるのか自分の考えのどこかにか間違いがあるのかわからないです…
0331132人目の素数さん
垢版 |
2023/06/21(水) 15:49:13.63ID:klKeRi4W
初歩的な質問ですいません

位相空間論の話なのですが、
開集合は、なぜ”開”集合と名付けられたのですか?
もっとふさわしい名称があると思うのですが。(例えば”近点”集合とか。近傍と紛らわしいという問題はありますが)
開集合は何に関して、開いているのかがよく分かりません。(わかりにくい質問ですいません)
0332132人目の素数さん
垢版 |
2023/06/21(水) 15:49:40.77ID:JHe2aSzv
ただ明らかにf'(x)=e^xf(x)はe^(e^x)というのは分かるので、自分の微分方程式の解き方に間違いがありそうだなとは思っています
0333132人目の素数さん
垢版 |
2023/06/21(水) 16:03:50.83ID:hlJcd1YI
>>330
じゃあ収束すると仮定したところか、極限交換したところがおかしいかのどっちか
漸化式から極限が存在すればf(0)=0
0334132人目の素数さん
垢版 |
2023/06/21(水) 16:06:42.34ID:hlJcd1YI
x=0で収束するならばね
x=0だけで収束しない可能性はもちろんあるけど
0335132人目の素数さん
垢版 |
2023/06/21(水) 16:18:26.81ID:hZXZNqQI
>>331
開区間の一般化だからだろ
0336132人目の素数さん
垢版 |
2023/06/21(水) 16:33:13.46ID:JHe2aSzv
>>333
そうですね
解答見た感じ与えられた漸化式をとくとn→∞でfn(x)=0となるので交換ができないのでやはりこの問題がおかしいと思いました。(院試の改題なので改題の仕方に問題があるぽいです)
0337132人目の素数さん
垢版 |
2023/06/21(水) 16:36:13.97ID:hZXZNqQI
>>305
(a)
fn’(x)=e^xfn-1(x) fn(0)=1/n!
f’(x)=e^xf(x) f(0)=0
f(x)=0
(b)
fn(x)=e^nx/n!
(c)
f(x)=0
0338132人目の素数さん
垢版 |
2023/06/21(水) 16:38:13.35ID:hZXZNqQI
>>336
別におかしくは無い
f’(x)=e^xf(x)をf’(x)/f(x)=e^xにできるとは限らないというだけ
0339132人目の素数さん
垢版 |
2023/06/21(水) 16:42:08.80ID:hZXZNqQI
>>332
>ただ明らかにf'(x)=e^xf(x)はe^(e^x)というのは分かる
f(x)=Ae^e^x
0340132人目の素数さん
垢版 |
2023/06/21(水) 16:51:00.44ID:JHe2aSzv
>>339
すいません、これは自分の書き方が悪く明らかにそういう形になるというだけでした
0341132人目の素数さん
垢版 |
2023/06/21(水) 17:12:06.57ID:E4opifdK
>>331
歴史は知らんけど先に閉集合が名付けられたと思えばいいんじゃないの
ユークリッド空間上で閉集合上の点列はその極限が外に出ることはできない、だから閉じてる
開集合はその逆でどの境界点にも出ることができる、だから開いてる
0342132人目の素数さん
垢版 |
2023/06/21(水) 19:41:24.40ID:Z/SjfIXF
「非真性的ではない特異点を真性特異点と呼ぶ」みたいな文をどこかで見たがこれもその例か
0343132人目の素数さん
垢版 |
2023/06/21(水) 20:29:51.79ID:daZZz0VB
>>331
1次元ユークリッド空間Rで、開区間 (a, b) が開と呼ばれていることによる
0345132人目の素数さん
垢版 |
2023/06/21(水) 20:50:30.54ID:CpQ5G3fh
境界がない=閉じてない=開いてる、くらいでいいんじゃね
0346132人目の素数さん
垢版 |
2023/06/21(水) 21:12:04.47ID:klKeRi4W
>>341
なるほど。なんか凄い納得できました。
ありがとうございます。

回答くださった方、ありがとうございます。
0347132人目の素数さん
垢版 |
2023/06/21(水) 21:17:27.09ID:E4opifdK
たぶん位相空間の開集合の定義だけを見て>>331のような疑問を持ったんだと思うけど、
ユークリッド空間でその定義と同値な言い換えを知っていれば開や閉という名称に違和感はないと思うね
0348132人目の素数さん
垢版 |
2023/06/22(木) 09:28:18.89ID:bA5uzkgG
>>331
開区間と閉区間には違和感はないのですか?
0350132人目の素数さん
垢版 |
2023/06/22(木) 12:20:30.26ID:vnKEIeYr
陰関数の定理を用いて証明される階数定理って何の役に立つんですか?
0351132人目の素数さん
垢版 |
2023/06/22(木) 12:30:39.29ID:uug7bkV1
数学の線型代数学の分野における階数・退化次数の定理(かいすう・たいかじすうのていり、英: rank–nullity theorem)とは、最も簡単な場合、ある行列の階数 (rank) と退化次数 (nullity) の和は、その行列の列の数に等しいということを述べた定理である。次元定理[1]とも呼ばれる。
0352132人目の素数さん
垢版 |
2023/06/22(木) 12:57:59.16ID:mbeQFSX5
見るからに基本的でそんな疑問が浮かぶような定理じゃないんだが
埋め込みとか沈め込みがなんで重要かくらい理解してから考えてみたら
0353132人目の素数さん
垢版 |
2023/06/22(木) 18:36:31.29ID:FEph2zbY
>>292
>>286で不定積分が初等関数で表せられないことの証明について質問した者です

その後スレを見る暇が無くて、先ほど見返しましたらとっくに過ぎてました
リュービルの定理や微分ガロア理論というのは、初めて知りました。

ただ微分ガロア理論は私には難しすぎるので、>>292さんがご教示いただいた
本で証明の概略を見てみます。
どうもありがとうございました。
0354132人目の素数さん
垢版 |
2023/06/22(木) 22:08:16.86ID:r7ZfVRUp
有理関数の積分についての疑問です

微積分の入門書には全ての有理関数は分子の次数が分母の次数より低くなるように割ったあと
残った有理関数を部分分数分解したら全て積分できるような事が書いてある

しかし分母の多項式が代数的数を零点に持たなければそもそも因数分解できないで机上の空論になるのではないですか?
0355132人目の素数さん
垢版 |
2023/06/22(木) 22:22:41.27ID:Px5gaTKK
回転行列ってこうですよね
https://i.imgur.com/8pBOiRj.jpg
これを(1,0)と(0,1)というベクトルに左からかけるとそれぞれこうなりますよね?
https://i.imgur.com/FV3J4vj.jpg
こうしてできたベクトルと元のベクトルをこのように表すとして
https://i.imgur.com/IKU9RFM.jpg
それらの関係を表せばこうですよね
https://i.imgur.com/AvYYx8T.jpg
それを行列のように表せばこうなるはずです
https://i.imgur.com/B4EzHDX.jpg
これはこの図のようにベクトルの組を同時に回転しているような演算だと思うんですが
https://i.imgur.com/KxTCcyp.jpg
ベクトルの組を回転させる時にかけてるこの行列が回転行列の転置みたいになってるのが不思議に思います
https://i.imgur.com/WjtkmaM.jpg
これはテンソルというものなのでしょうか?
0356132人目の素数さん
垢版 |
2023/06/22(木) 22:27:59.67ID:IMGX8Hu2
>>354
実多項式は必ず1次または2次の多項式に実因数分解されます
0357132人目の素数さん
垢版 |
2023/06/22(木) 22:37:50.47ID:x/7y91LC
指数関数対数関数三角関数逆三角関数無理関数は初等関数だけど
複素関数にしたら三角関数も逆三角関数も指数関数対数関数で表せるし
べき乗根も指数関数対数関数で表せるから
指数関数対数関数だけで考えたら良い
C(x):複素有理関数体から指数関数対数関数で拡張していくことのできるすべての関数が初等関数
具体的にはf(x)が初等関数とは
C(x)=K0⊂K1⊂K2⊂…⊂Kn⊂……
という拡大列で
Kn+1=Kn(exp(f),log(f)|f∈Kn)
として
K=∪Kn
の元のこととする
ここで
exp(f)'/exp(f)=f'
log(f)'=f'/f
なので
D(f)=f', E(f)=f'/f
と定義してやれば
Kn+1=Kn(g|D(g)∈E(Kn) or E(g)∈D(Kn))
見たいに書ける
0358132人目の素数さん
垢版 |
2023/06/22(木) 22:43:13.09ID:x/7y91LC
>>354
>しかし分母の多項式が代数的数を零点に持たなければそもそも因数分解できない
代数的数って多項式の根のこと
零点が無ければ(0以外の)定数関数だよ
0359132人目の素数さん
垢版 |
2023/06/22(木) 22:56:06.19ID:r7ZfVRUp
>>356
それは理論上の話であって具体的には因数分解出来ない物があるのではないですか?
てないと全ての5次以上の方程式が冪乗根とかを使って具体的に解けてしまいます

>>358
済みません、言葉を間違えていました
「代数的数」は「四則演算と冪根だけで表せれる数」でした
0360132人目の素数さん
垢版 |
2023/06/22(木) 23:29:30.89ID:t7gfNjo5
例えば1/(簡単な例で

f(x) = 5x⁴-1)/(x⁵-x+3)

の分母は確かに因数分解できないけど、根を α〜εとでもおけば部分分数分解は全部同じ形になる
この場合

f(x) = 1/(x-α) + ... + 1/(x-ε)。

になる
その不定積分は

∫f(x)dx = lo|(x-α| + ... + log/|x-ε| + C
= log |(x-α| ... /|x-ε| + C
= log (x⁵-5x-3| + C

のように結局対称式なので整理すればα〜εを消去できる
0362132人目の素数さん
垢版 |
2023/06/23(金) 01:10:09.05ID:vBUhmFi+
>>359
ん?代数的数とはべき乗根を使って書けるとは限らないけど?
君は「具体的に書ける」を「べき乗根を使って書ける」と認識したいのだろうけど
普通の感覚ではそれはどうでもいい
0363132人目の素数さん
垢版 |
2023/06/23(金) 04:19:28.63ID:TJgAQDOA
>>354
代数学の基本定理「複素係数の多項式は必ず複素数内に零点を持つ」と、
実係数多項式の虚数零点は共役なものと必ず2つペアで現れることから保証されています。
0364132人目の素数さん
垢版 |
2023/06/23(金) 08:37:57.59ID:wq5h+CFA
レムニスケートの等分点の話には
なかなかつながらないな
0365132人目の素数さん
垢版 |
2023/06/23(金) 13:51:42.90ID:Dkh8vVMn
>>360
根を α~εは虚数の場合は、その議論では不十分
虚数のlogになるので、主値を選んで偏角の議論をする必要がある
0367132人目の素数さん
垢版 |
2023/06/23(金) 14:49:12.98ID:Dkh8vVMn
結果は正確でも、議論に重大な欠陥があるからテストなら0点
0368132人目の素数さん
垢版 |
2023/06/23(金) 16:02:51.51ID:hAVN4rBn
>>365
>>363の2行目を読んでないね
0369132人目の素数さん
垢版 |
2023/06/23(金) 17:43:13.73ID:Dkh8vVMn
>>368
【追試】
では、∫dx/(x^2+1) を>>360の方法で正しく解答せよ。
0372132人目の素数さん
垢版 |
2023/06/24(土) 00:08:55.84ID:ZdRBWjDr
【ライプニッツの定理】
有理関数の不定積分は初等関数で書ける。
特に、有理関数、対数関数、逆正接関数で書ける。
0373132人目の素数さん
垢版 |
2023/06/24(土) 09:04:36.86ID:lc8wXOJ3
斎藤毅著『微積分』ですが、三角関数の定義の前に、平面幾何の命題を証明しています。
これって平面幾何的に考えなくても、もちろん厳密に計算だけでも示せますよね。
厳密でない平面幾何的な証明をしているのはなぜでしょうか?
この部分が気に入りません。

「高校数学とのつながりを重視して,逆三角関数の逆関数として定義した.といっても,連続関数の不定積分の存在や広義積分の収束条件を証明するまで三角関数を使えないようでは困るので,積分を使わずに弧の長さを定義しておいた.」
0376132人目の素数さん
垢版 |
2023/06/24(土) 10:25:15.92ID:lc8wXOJ3
斎藤毅著『微積分』ですが、円の孤の長さを孤に内接する折れ線の長さ以上で、孤に外接する折れ線の長さ以下であるような唯一の実数として定義しています。

ですが、孤に内接する折れ線の長さの上限と定義しなかったのはなぜでしょうか?
0377132人目の素数さん
垢版 |
2023/06/24(土) 10:41:11.31ID:8guI7e4t
>>376
「原論」に合わせたのであろう
0378132人目の素数さん
垢版 |
2023/06/24(土) 10:42:38.55ID:8guI7e4t
>>370
伊勢幹夫先生はそのような問題を
「単なる労働に過ぎない」
と言ってスキップされた
0379132人目の素数さん
垢版 |
2023/06/24(土) 10:47:07.38ID:Dm0QrQGF
>>359
正規行列は対角化可能、とかも5次以上の行列で固有値を具体的に求められない場合があるから机上の空論なの?
0380132人目の素数さん
垢版 |
2023/06/24(土) 12:10:53.73ID:3EWbRRim
>>370
x^5-x+3は重根が無いから
a1〜a5を根とすると
1/(x^5-x~3)=A1/(x-a1)+…+A5/(x-a5)
A1=1/(a1-a2)(a1-a3)(a1-a4)(a1-a5)=5a1^4-1=4-15/a1

A5=4-15/a5
∫dx/(x^5-x+3)=(4-15/a1)log|x-a1|+…+(4-15/a5)log|x-a5|
まあここまでかな
対称式だけど多項式じゃ無いし
0381132人目の素数さん
垢版 |
2023/06/24(土) 12:12:07.79ID:3EWbRRim
>>378
確かに>>380は単なる労働
0382132人目の素数さん
垢版 |
2023/06/24(土) 12:20:57.11ID:3EWbRRim
前に書いたときあるけど
3次方程式の解の公式ですら
複素数の3乗根を実部と虚部の四則とべき乗根で表せないだよな
存在して特定できるんだから具体的に表せなくても
元の係数の関数て考えるのが普通
0383132人目の素数さん
垢版 |
2023/06/24(土) 13:27:08.24ID:lc8wXOJ3
Real and Functional Analysis (Graduate Texts in Mathematics, 142) ペーパーバック ? 2013/10/4
英語版 Serge Lang (著)

現在、Amazon.co.jpでペーパーバックが1668円ですね。
この前のSpringerのセールのときには、2270円だったと思います。
そのときには買わなかったのですが、思わず注文しました。
新品ですが、どんなコンディションのものが送られてくるのか不安です。
0384132人目の素数さん
垢版 |
2023/06/24(土) 13:49:15.72ID:lc8wXOJ3
現在1200円です。

Differential Analysis on Complex Manifolds (Graduate Texts in Mathematics) ペーパーバック ? 2010/11/23
英語版 Raymond O. O. Wells (著)
0385132人目の素数さん
垢版 |
2023/06/24(土) 13:49:48.03ID:lc8wXOJ3
現在979円です。

Lectures on Partial Differential Equations (Universitext) ペーパーバック ? イラスト付き, 2008/10/10
英語版 Vladimir I. Arnold (著)
0386132人目の素数さん
垢版 |
2023/06/24(土) 13:50:22.30ID:lc8wXOJ3
>>384-385

両方とも注文してしまいました。
0387132人目の素数さん
垢版 |
2023/06/24(土) 18:44:44.27ID:QBFsXmBC
(3)が分かりません、位相空間に強い方お願いします
https://i.imgur.com/HmGEkkS.jpg
0388132人目の素数さん
垢版 |
2023/06/24(土) 19:04:55.37ID:QBFsXmBC
>>387
自己解決しました
(0,1)(1,0)が分離不能でした
0390132人目の素数さん
垢版 |
2023/06/24(土) 22:42:37.34ID:pLJVeXu1
>>389
方針はホボホボ一瞬?
詳細を詰めるのが面倒か?
そうでもないか
0391132人目の素数さん
垢版 |
2023/06/24(土) 23:16:49.23ID:DZPnYw6n
すいません、確率についての質問なのですが、

1~100の数字から1つ自分で決める。その後同様に確からしい確率で1~100の数字のどれかが選ばれるとする。外れたらまた同じことをする。

これを何回も繰り返していってできるだけ少ない回数で自分の言った数字を当てる作戦を考えたいのですが、こういうのってどういう作戦が「最適」となるのでしょうか??
0392132人目の素数さん
垢版 |
2023/06/24(土) 23:18:45.53ID:DZPnYw6n
たとえば自分が思ったのはまだ当たっていない数字をどんどん言っていくほうがより当たりやすくなると思うのですがどうなのでしょうか?
またその作戦が最適解である、というのはどのようにして言えるのでしょうか?
0394132人目の素数さん
垢版 |
2023/06/25(日) 08:57:09.04ID:yL+dRGxy
ラッセルのパラドックスってのがよくわかりません
集合はあくまで集合であって自身の要素としては扱えないってことですか?
0395132人目の素数さん
垢版 |
2023/06/25(日) 09:16:14.15ID:pPR54CPq
AがAに属さないということを
集合論の公理に含めないとしたら
集合Aのとり方によっては
それが成り立つ場合と成り立たない場合があることになるので
そのような集合全体とそうでない集合全体が生じるが
それらを集合の一種とみなすと
論理的な破綻が生じるので
集合と呼べるものの範囲は何らかの仕方で
もっと限定しないといけない
0396132人目の素数さん
垢版 |
2023/06/25(日) 09:21:46.60ID:w6zc8jwM
AはAに属さない(要素としては扱えない)
ということを公理にするための話なんですね
ほんの少し掴めそうな気がしてきました
0397132人目の素数さん
垢版 |
2023/06/25(日) 09:39:03.23ID:INDvmnrD
>>394
ZFC|-¬∃x∀y(y∈x)
である。
証明は背理法。∃x∀y(y∈x)を仮定すると、x∈xを得る。
正則性公理から、矛盾を得る。
0398132人目の素数さん
垢版 |
2023/06/25(日) 09:41:48.97ID:INDvmnrD
もっというと、
ZFC|-¬∃x1,…,xn(x1∈…∈xn∈x1)
証明は背理法。
∃x1,…,xn(x1∈…∈xn∈x1)を仮定する。
正則性公理により、{x1,…,xn}は∈-極小元yを持つ。
yがx1,…,xnのどれであったとしても矛盾を得る。
0399132人目の素数さん
垢版 |
2023/06/25(日) 09:46:14.67ID:INDvmnrD
ZFCでは{x|¬(x∈x)}は集合ではない。
証明は背理法
ZFCでは{x|¬(x∈x)}は集合であるとする。
つまり、ZFC|-∃y∀x(x∈y⇔¬(x∈x))である。
ここから直ちに矛盾を得る
0400132人目の素数さん
垢版 |
2023/06/25(日) 10:17:03.15ID:stfJqzZm
>>396
公理としては無限降下列は存在しないていう基礎の公理ね
0401132人目の素数さん
垢版 |
2023/06/25(日) 19:44:02.69ID:6r/txfgr
>>387-388
A_(1,0) = x軸の正の部分,
A_(0,1) = y軸の正の部分,

これらを2次元平面の開集合で分離しようとしても、原点の近くで必ず交わってしまうから
0402132人目の素数さん
垢版 |
2023/06/25(日) 19:57:37.61ID:stfJqzZm
>>401
>これらを2次元平面の開集合で分離しようとしても、原点の近くで必ず交わってしまうから
ほんのちょっと違う
平面の部分集合を平面の開集合で分離するのではなくて
使える開集合が限定されているから交わる
双曲線の一方だけが点つまり双曲線は2点
それで平面を類別したのとx,y軸を原点以外の半直線4つに分けたそれぞれが点
原点を除外したのは入れると簡単すぎるからだな
0403132人目の素数さん
垢版 |
2023/06/25(日) 22:15:46.81ID:6r/txfgr
>>402
>>401だが、書き方が悪かったかな

商位相は引き戻して割る前の空間の開集合だから、>>401の話は全部割る前の空間
R^2 -{(0,0)} での話です

もちろん、A_(1,0), A_(0,1)は商空間では単なる2点で(もちろん異なる)、その引き戻しが
(x軸の正の部分)と(y軸の正の部分)になり、それが R^2 -{(0,0)} の開集合で分離出来
ないことによる。
0404132人目の素数さん
垢版 |
2023/06/25(日) 22:37:09.89ID:6r/txfgr
>>403
すまん、分離が出来ない理由が不十分だった。
訂正

A_(1,0)の引き戻し = (x軸の正の部分) と A_(0,1)= (y軸の正の部分) に、
幾らでも近い第1象限の双曲線が存在するから、分離出来ない。
0405132人目の素数さん
垢版 |
2023/06/25(日) 23:13:18.43ID:w6zc8jwM
https://i.imgur.com/u52EGbU.jpg
読んでる本がわからないです
なんでXが集合ならYはXの部分集合ってことになるんですか?
Yはこの時点だと集まりとしか言ってなくて集合とは言ってないからYがXに含まれるかはわからなくないですか?
0406132人目の素数さん
垢版 |
2023/06/25(日) 23:46:16.28ID:INDvmnrD
>>405
部分集合公理:集合Xに対して、その部分クラスは集合である。

別な言い方をすると、集合Xと論理式φに対して、{x∈X|φ(x)}は集合である。
0407132人目の素数さん
垢版 |
2023/06/25(日) 23:52:26.70ID:w6zc8jwM
>>406
つまりこれは「Xが集合であるなら明らかにそれに含まれるYも集合だ」ってのを言ってる文?
0409132人目の素数さん
垢版 |
2023/06/25(日) 23:59:14.51ID:INDvmnrD
>>405
>>Yはこの時点だと集まりとしか言ってなくて集合とは言ってないからYがXに含まれるかはわからなくないですか?
YがXに含まれている、つまり、Y⊆Xであることは定義によって明らか。
で、Y⊆XとXが集合であることから、Yも集合である。という理屈。
0410132人目の素数さん
垢版 |
2023/06/26(月) 00:33:53.22ID:FD+feVvW
あとラッセルのパラドックスについて検索すると
「自身を要素に持たない集合の集まりは集合とは言えない」
と出てきますがこれと「集合全体の集まりは集合とは言えない」というのは同じことなんでしょうか?
後者はこの世全ての集合というのがなんだか限定的な状況で
前者はA={1}、B={2}、Y={{1}、{2}}みたいな具体例がいくつも考えられるような何となく広い意味に感じます
0412132人目の素数さん
垢版 |
2023/06/26(月) 04:10:31.53ID:bzxo1iVd
>>403
>それが R^2 -{(0,0)} の開集合で分離出来
>ないことによる。
ほんのちょっと違う
R^2-原点の開集合では分離できるが
この空間の開集合の引き戻しでは分離できない
0413132人目の素数さん
垢版 |
2023/06/26(月) 04:18:18.46ID:bzxo1iVd
>>404
>A_(1,0)の引き戻し = (x軸の正の部分) と A_(0,1)= (y軸の正の部分) に、
>幾らでも近い第1象限の双曲線が存在するから、分離出来ない。
これなら問題ない
0414132人目の素数さん
垢版 |
2023/06/26(月) 06:37:20.52ID:vfjbFyrG
ラッセルのパラドックス

出典: フリー百科事典『ウィキペディア(Wikipedia)』

ラッセルのパラドックス(英: Russell's paradox)とは、素朴集合論において、自身を要素として持たない集合全体からなる集合の存在を認めると矛盾が導かれるというパラドックス。バートランド・ラッセルからゴットロープ・フレーゲへの1902年6月16日付けの書簡においてフレーゲの『算術の基本法則』における矛盾を指摘する記述に現れ、1903年出版のフレーゲの『算術の基本法則』第II巻(独: Grundgesetze der Arithmetik II)の後書きに収録された。なお、ラッセルに先立ってツェルメロも同じパラドックスを発見しており、ヒルベルトやフッサールなどゲッティンゲン大学の同僚に伝えた記録が残っている。

ラッセルの型理論(階型理論)の目的のひとつは、このパラドックスを解消することにあった
0415132人目の素数さん
垢版 |
2023/06/26(月) 07:39:29.50ID:bzxo1iVd
>>414
>ラッセルの型理論(階型理論)
ZFCより優れてるんじゃ無いかな
素朴だしプログラミングや自然言語研究に使われる一般性もある
0416132人目の素数さん
垢版 |
2023/06/26(月) 21:07:21.27ID:erZv0rYT
集合についてですが自身を要素とする集合は許されないみたいですが
自身以外の集合を要素として集合を名乗ることは許されますか?
A={1,{2}}という集合はありですか?
0419132人目の素数さん
垢版 |
2023/06/26(月) 22:44:35.23ID:DVaB91mE
>>376
曲線の長さなら、内接の上限として定義しても問題ないが、
2次元の曲面の場合は、内接の上限では∞となってしまう例がある(3次元以上も同様)。
そのため、内接と外接のサンドウィッチで定義する。
おそらく、次元に依らない定義にした方がよいと判断されたのであろう。
0420132人目の素数さん
垢版 |
2023/06/26(月) 22:46:52.34ID:DVaB91mE
ルベーグ可測性も内測度と外測度が一致すると定義しているのもそのため
0421132人目の素数さん
垢版 |
2023/06/26(月) 22:51:29.72ID:vfjbFyrG
>>416
>>A={1,{2}}という集合はありですか?

0=φ,1={0}, 2={0,1}, 3={0,1,2}, ...

毎年1年生の微積分の授業でこれをやった。
もちろんラッセルのパラドックスも。

別のパラドックスをやった人もいた。
抜き打ち試験のパラドックスとか。

この話だけで30分はもつ。
0422132人目の素数さん
垢版 |
2023/06/27(火) 07:07:41.94ID:1CbeElLS
>>419
>2次元の曲面の場合は、内接の上限では∞となってしまう例がある(3次元以上も同様)。
なら
内接多面体<S<概説多面体てなる数値Sは存在しないのでは?
0423132人目の素数さん
垢版 |
2023/06/27(火) 10:34:22.38ID:rXhkQ5AD
目次【本記事の内容】

実数の連続性公理について質問です

例えば以下の6個は全て同値ですがアルキメデスの原理を内包している場合とそうでない場合の違いがピンと来ません
感覚的な説明で結構ですので宜しくお願いします

1.Dedekindの切断による実数の定義

2.Weierstrassの公理

3.有界な単調数列の収束

4.区間縮小法+アルキメデスの原理

5.Bolzano–Weierstrassの定理

6.Cauchyの収束条件+アルキメデスの原理
0424132人目の素数さん
垢版 |
2023/06/27(火) 12:19:32.67ID:6uguPgXG
>>423
さらに、次の主張も同値です。

7. 中間値の定理

8. 最大値・最小値の定理

9. Rolleの定理

10. Lagrangeの平均値の定理

11. Cauchyの平均値の定理


他にも実数の公理と同値な命題は沢山あります。
0425132人目の素数さん
垢版 |
2023/06/27(火) 12:26:30.56ID:6uguPgXG
>>423
アルキメデスの原理は順序に関する主張。

例えば、Cauchy列の収束は実数の完備性しか主張しておらず、
完備化が順序の公理と適合していることを保証するのがアルキメデスの原理かと。
0426132人目の素数さん
垢版 |
2023/06/27(火) 16:45:32.32ID:SWjZGkg0
>>424
マジ?
その証明はどんな本に書いてありますか?
0427132人目の素数さん
垢版 |
2023/06/27(火) 16:52:53.11ID:iCbZv4nE
>>426

実数論講義 (微分積分学)
赤攝也

はどうでしょうか?

アマゾンのレビューの一つに、


特に「連続の公理」について詳しく
デーデキント(1831-1916)の
「切断」を公理1と置き
それと同値な22コの命題を証明したのが
たいへんユニークな記述となっています。


と書かれています。
0428132人目の素数さん
垢版 |
2023/06/27(火) 17:21:24.59ID:U26pCXHd
>>427
>赤攝也
よくやるなあ
0429132人目の素数さん
垢版 |
2023/06/27(火) 17:32:21.74ID:SWjZGkg0
>>427
おお、ありがとうございます!
昔の本って今と違って味がありますね
0430132人目の素数さん
垢版 |
2023/06/27(火) 17:49:08.65ID:CRAZAxJn
>>419
>2次元の曲面の場合は、内接の上限では∞となってしまう例がある(3次元以上も同様)。

この例を知りたい
0431132人目の素数さん
垢版 |
2023/06/27(火) 18:04:06.42ID:4t3olQvB
>>430
アリストテレスのジャックオーランタンとかいうやつ
0432132人目の素数さん
垢版 |
2023/06/27(火) 19:21:28.41ID:6uguPgXG
>>430
シュワルツのランタン

円柱の表面積を内部から多面体のように近似していく(ランタンの形になる)
近似の三角形をうまく取っていくと、円柱の表面積を超えて、最終的に無限大に出来る
0434132人目の素数さん
垢版 |
2023/06/27(火) 19:50:50.00ID:p7Y7Bc2m
直線を同じ点を通らないように並行移動させて
作る面を球内に制限すると
いくらでも面積の大きい内接面ができる
0435132人目の素数さん
垢版 |
2023/06/27(火) 20:15:29.60ID:YIp4OMv2
>>425
アルキメデスの原理を満たさない順序体もありますよね
コーシー列の収束と順序の公理を満たすだけで実数の公理と同値になりますか?
0436132人目の素数さん
垢版 |
2023/06/27(火) 20:58:06.27ID:FoYK0gzJ
>>435
自分で考えてないのが丸わかりのききかた
0438132人目の素数さん
垢版 |
2023/06/29(木) 05:01:07.95ID:ePu6JB1l
雪片曲線みたいなやつか
0439132人目の素数さん
垢版 |
2023/06/29(木) 09:24:25.65ID:OnKsqz/N
>>437
その場合は長さ無限大なのでは?
曲面咳の状況とは違うと思うけど
0440132人目の素数さん
垢版 |
2023/06/29(木) 09:31:25.39ID:OdxUbeBI
せやね
曲線の長さの定義のひとつで
lim Σ | Pᵢ-Pᵢ₊₁|
とする場合があるけど、三次元の中の2次元局面ではそれがうまくいかないと言う例がシュバルツの提灯
曲線の場合は何次元でも上の定義でうまくいくんじゃない?
知らんけど
0441132人目の素数さん
垢版 |
2023/06/29(木) 09:45:11.46ID:esUePVuH
正の面積を持つジョルダン曲線の例を挙げよ
0442132人目の素数さん
垢版 |
2023/06/29(木) 18:17:45.32ID:DjbhytWn
>>440
> シュバルツの提灯

提灯っていつの時代やねんw
ランタンやろ
そもそもシュワルツはドイツ人やし、ちょうちんなんて知らん
0443132人目の素数さん
垢版 |
2023/06/29(木) 18:48:35.35ID:nfa1vGBW
吉田洋一「零の発見」
p.174
および見開きの写真
0445132人目の素数さん
垢版 |
2023/06/29(木) 19:38:46.46ID:nfa1vGBW
誰かがSchwarzを極小曲面の専門家と呼んでいた
0446132人目の素数さん
垢版 |
2023/06/29(木) 20:25:39.79ID:HnX+7Cab
f:ℝ^n→ℝが微分可能な時(f(x+λy)+f(x))/λ→<∇f(x),y> (λ→+0)となるのは何故ですか?
0448132人目の素数さん
垢版 |
2023/06/29(木) 22:42:23.58ID:OnKsqz/N
>>447
はぁ
0449132人目の素数さん
垢版 |
2023/06/29(木) 23:14:58.44ID:5mRhSz4N
正項級数についてのガウスの判定法って定理として書くほどの有用性はありますか?

定理というのは使う機会が多い結果を書いたものだと思います。

使う機会がないものを定理として書く意味はありません。

稀に必要なことがあるならば、ガウスの判定法の証明と同じ議論をしてその場で証明すればいいだけのことではないでしょうか?
0452132人目の素数さん
垢版 |
2023/06/30(金) 05:57:55.16ID:DpdNMEXg
>>451
これは正論w

>>449
あなたの狭い数学では必要なくても、必要な分野があるんですよ
解析数論、ゼータ関数やディリクレ級数論には不可欠な定理(考え方)ですけどね
0453132人目の素数さん
垢版 |
2023/06/30(金) 07:38:50.76ID:3fVz2KZ7
っつーか、しょっちゅう使うなら、補題っていうよな

ごく一部の著者なら、補題/系といった語の使い分けを一切せずに、全部定理として言いくるめることもあるけど、これは少数な例
0454132人目の素数さん
垢版 |
2023/06/30(金) 08:31:06.66ID:Zk3dxBIo
シュワルツの補題は
シュワルツの定理というべきだという意見を
7年前の学会で聞いた
0455132人目の素数さん
垢版 |
2023/06/30(金) 08:36:45.04ID:PYvjil6k
米田レンマか米田の定理か
0456132人目の素数さん
垢版 |
2023/06/30(金) 09:48:09.31ID:Zk3dxBIo
5lemmaが5theoremになるとは思えない
0457132人目の素数さん
垢版 |
2023/06/30(金) 10:26:36.99ID:RlyFFQ2d
宮島静雄著『微分積分学I』

Raabeの判定法

正項級数 Σa_n に対して lim a_{n+1}/a_n = 1 であるが lim n * (1 - a_{n+1}/a_n) =: k が存在するとき、

Σa_n は k > 1 のとき収束し、 k < 1 のとき発散する。

-----------------------------------------------------------------------------------------

Gaussの判定法

正項級数 Σa_n に対して、ある δ > 0 があって

a_{n+1}/a_n = 1 - 1/n + O(1/n^{1+δ})

が成り立っていれば、 Σa_n は発散する。
0458132人目の素数さん
垢版 |
2023/06/30(金) 10:28:34.57ID:RlyFFQ2d
小平邦彦著『解析入門1』

正項級数 Σa_n において

a_n/a_{n+1} = 1 + σ/n + O(1/n^{1+δ}), δ > 0,

とする。このとき、 δ > 1 ならば Σa_n は収束し、 δ ≦ 1 ならば Σa_n は発散する。
0459132人目の素数さん
垢版 |
2023/06/30(金) 10:29:33.52ID:RlyFFQ2d
小平さんの書き方は宮島さんの書き方と比べて、精密さがありません。

なぜ、小平さんはこんな雑な定理の述べ方をしたのでしょうか?
0460132人目の素数さん
垢版 |
2023/06/30(金) 10:33:32.36ID:RlyFFQ2d
宮島さんの定理を使えば、

a_n/a_{n+1} = 1 - σ/n + O(1/n), δ < 1

さえ示せれば、発散することが分かりますし、

a_n/a_{n+1} = 1 - σ/n + O(1/n), δ > 1

さえ示せれば、収束することが分かります。
0461132人目の素数さん
垢版 |
2023/06/30(金) 10:34:01.01ID:RlyFFQ2d
>>460

訂正します:

宮島さんの定理を使えば、

a_n/a_{n+1} = 1 + σ/n + O(1/n), δ < 1

さえ示せれば、発散することが分かりますし、

a_n/a_{n+1} = 1 - σ/n + O(1/n), δ > 1

さえ示せれば、収束することが分かります。
0462132人目の素数さん
垢版 |
2023/06/30(金) 10:34:32.74ID:RlyFFQ2d
>>461

訂正します:

宮島さんの定理を使えば、

a_n/a_{n+1} = 1 + σ/n + O(1/n), δ < 1

さえ示せれば、発散することが分かりますし、

a_n/a_{n+1} = 1 + σ/n + O(1/n), δ > 1

さえ示せれば、収束することが分かります。
0463132人目の素数さん
垢版 |
2023/06/30(金) 10:37:21.15ID:RlyFFQ2d
>>462

訂正します:

宮島さんの定理を使えば、

a_n/a_{n+1} = 1 + σ/n + O(1/n), 0 < δ < 1

さえ示せれば、発散することが分かりますし、

a_n/a_{n+1} = 1 + σ/n + O(1/n), δ > 1

さえ示せれば、収束することが分かります。
0464132人目の素数さん
垢版 |
2023/06/30(金) 10:39:04.16ID:RlyFFQ2d
ところで、不等式の書き方についてなのですが、

x > 1 などと書く人がいます。

これは、 1 < x と書いたほうが良いと思います。

理由は、数直線上で、右にある点ほど大きな実数を表すからです。

「>」は使わないほうがいいということになりますよね。
0466132人目の素数さん
垢版 |
2023/06/30(金) 11:23:17.28ID:RlyFFQ2d
>>465

間違ったことは書いていないと思います。
0467132人目の素数さん
垢版 |
2023/06/30(金) 11:27:38.85ID:RlyFFQ2d
宮島静雄さんの本は妙な行間がある箇所がたくさんあります。

第2巻にオイラー・マクローリンの公式が書いてありますね。
0468132人目の素数さん
垢版 |
2023/06/30(金) 12:30:47.17ID:3fVz2KZ7
>>464
こいつって地味に俺が感じてるところと同じところを指摘するから、見ててニヤニヤすることがあるんだよなwww
0469132人目の素数さん
垢版 |
2023/06/30(金) 12:32:39.66ID:3fVz2KZ7
>>464
じゃあお前に質問なんだけど、

「任意にx<yを取る。」って言ったら、
「yより小さいxを任意に取る」 か 「xより大きいyを任意に取る」 か
どっちの意味だと思う?
0471132人目の素数さん
垢版 |
2023/06/30(金) 13:11:35.81ID:RlyFFQ2d
>>469

「任意にx<yを取る。」とだけ書かれていたら、

「x < y であるような2つの実数を取る」の意味だと思います。
0472132人目の素数さん
垢版 |
2023/06/30(金) 13:11:41.53ID:DpdNMEXg
>>464
これは全く逆

大事な物、メインを先に書く方が分かりやすし、そう書くべき。
この場合、1より xが大事だから、x>1 と書く方が良い。
0473132人目の素数さん
垢版 |
2023/06/30(金) 13:12:35.82ID:TR/aoMqt
そだな
0474132人目の素数さん
垢版 |
2023/06/30(金) 13:17:02.93ID:VDuoYfC1
プログラムのif文なんかだと可読性が大事なので不等式の左辺の方に今着目している変数を置く
そういうのが分かってない人のプログラムは全体的に汚くて読みにくい
数学の不等式も基本的にはそうではないか?ID:RlyFFQ2dのレスに出てくる不等式も全部そうだろ?
0475132人目の素数さん
垢版 |
2023/06/30(金) 13:18:59.46ID:DpdNMEXg
ε-δで、「任意のε>0」と書くのが普通。というか、こう書くべき。

「任意の0<ε」なんて書く奴は見たことないし、セミナーならツッコミが入るw
0477132人目の素数さん
垢版 |
2023/06/30(金) 13:49:42.07ID:Vi1Ems1C
まぁ高校生ならこう言う事言う奴いても不思議ないんだけどな
「x²-5x+6>0の解は x< 2または3<xである」とか書くからな

オレはどっちかと言うとこっちの方が違和感あるけどな
0478132人目の素数さん
垢版 |
2023/06/30(金) 14:36:21.89ID:VDuoYfC1
x^2 を x² みたいに書いてるのっていつも同じ人?それとも最近はそういう書き方が流行ってるの?
ちょいちょい文字化けして余計に見にくいんだが
0479132人目の素数さん
垢版 |
2023/06/30(金) 14:42:46.71ID:RzuIevd3
機種依存文字は使わないという基本的なリテラシーがないんだろうね
0480132人目の素数さん
垢版 |
2023/06/30(金) 15:20:36.02ID:hhPT0E2t
時間の止まった老害が説教をたれています
0481132人目の素数さん
垢版 |
2023/06/30(金) 15:26:18.58ID:oUBRvOBS
機種依存文字は控えるというのはSJISが使われてた頃の話で、
²もそうだがUnicodeが出来てからそんなリテラシーは消えた
文字化けする人はまず自分の文字コードの設定を見直せよ
0483132人目の素数さん
垢版 |
2023/06/30(金) 17:06:36.08ID:VDuoYfC1
どうやら化けてるわけじゃないみたいだがJane Styleでこういうnは字が潰れて読めない

ₙC₂

²も画面に近づかないと2なのか3なのか分かりにくいほど小さい
0485132人目の素数さん
垢版 |
2023/06/30(金) 17:38:02.10ID:VDuoYfC1
どうもJane Styleのデフォルトのフォントが悪いみたいだ
適当に変えとくわ
失礼した
0487132人目の素数さん
垢版 |
2023/06/30(金) 20:03:28.60ID:AAVeb+XQ
>>486
機種依存文字っていうのはSJISをWindowsとMacで独立して拡張したりしてた頃の話で、
世界中のwebサイトの90%がUnicodeの一種であるUTF-8で出来てる今の時代に、
機種依存文字は使わないとかいうリテラシーなんて完全に因習的な考え方なんだよ
0488132人目の素数さん
垢版 |
2023/06/30(金) 20:08:38.77ID:AAVeb+XQ
かつて機種依存文字(環境依存文字)は、OSなどの環境の違いにより文字化けを起こしてしまうため、Webページにおいて使ってはならない文字として扱われていました。

しかし、これは昔のはなし。
実は、WebページがUTF-8などのUnicodeを採用している場合は、機種依存文字(環境依存文字)が問題なく使えます。


https://www.asobou.co.jp/blog/web/izonmoji
0489132人目の素数さん
垢版 |
2023/06/30(金) 20:59:40.34ID:4jadW20+
まぁもはUnicodeで普通に規定されてる範囲の文字は使っていいやろ
もちろんフォントグリフは微妙に機種ごとに違いがあって見た目の違いは当然出てくるんだけど、それは上つき下つき文字とか関係なくどんな文字でもそう、それを言い出したらもはや何も始まらない
Unicodeは世界中のあらゆる文字を収録していってるみたいだから普通のPCとかスマポのブラウザに入ってない文字とかはあるかもしれんけど上つき下つき文字はええやろ
0490132人目の素数さん
垢版 |
2023/07/01(土) 01:40:56.26ID:6o0+3LBE
5chはShift_JIS
0491132人目の素数さん
垢版 |
2023/07/01(土) 19:48:52.98ID:xq7I8YbU
フーリエ級数展開には積分と総和を入れ替えられるかの問題があったって話を聞いたんですが
何かの足し算で表されるものを積分したものはそれぞれ積分したものを足し合わせものと等しいってのは当たり前のことではないんですか?
フーリエ係数はそれぞれで積分すると発散するけど足し合わせてから積分すれば収束するとかそういう可能性を否定できなかったとかそういう感じですか?
0492132人目の素数さん
垢版 |
2023/07/01(土) 21:03:28.09ID:Memg0Kv8
lim ∫[a,b] fₙ(x)dx と ∫[a,b] limfₙ(x)dx が必ずしも一致しないと言うのは解析の最初の一歩
0493132人目の素数さん
垢版 |
2023/07/02(日) 02:30:46.70ID:Ajq/PEPm
【基本的な例】
2重数列 a_{m,n} = n/(n+m) に対して、
lim_{n→∞} lim_{m →∞} a_{m,n} = 0,
lim_{m→∞} lim_{n→∞} a_{m,n} = 1
なので、2重極限は一般に交換可能ではない。
0494132人目の素数さん
垢版 |
2023/07/02(日) 21:21:55.16ID:HjuAv88H
宮島静雄著『微分積分学I』

実関数のノルムを定義のところでは、有界な関数について定義しています。
ところが、関数列 {f_n}_n がコーシー列であることの定義のところでは、
f_n に有界という条件を課していません。

確かに、 f_m - f_n が有界でさえあれば、 ||f_m - f_n|| は定義されますが、
たとえば、 f_m が有界で、 f_n が有界でない場合には、 f_m - f_n は非有界です。
一方、 f_m が非有界、 f_n も非有界であったとしても、 f_m - f_n が有界になる場合はあります。

松坂和夫さんの本では、 X を集合、 Y をノルム空間としたとき、 X から Y への有界写像全体の集合の元 f に対して、

||f|| = sup_{x∈X} |f(x)|

とそのノルムを定義しています。

宮島さんは、実関数の有界性について曖昧にしていますが、問題ないのでしょうか?
0495132人目の素数さん
垢版 |
2023/07/02(日) 21:22:44.14ID:HjuAv88H
訂正します:

宮島静雄著『微分積分学I』

実関数のノルムの定義のところでは、実関数のノルムを有界な関数について定義しています。
ところが、関数列 {f_n}_n がコーシー列であることの定義のところでは、
f_n に有界という条件を課していません。

確かに、 f_m - f_n が有界でさえあれば、 ||f_m - f_n|| は定義されますが、
たとえば、 f_m が有界で、 f_n が有界でない場合には、 f_m - f_n は非有界です。
一方、 f_m が非有界、 f_n も非有界であったとしても、 f_m - f_n が有界になる場合はあります。

松坂和夫さんの本では、 X を集合、 Y をノルム空間としたとき、 X から Y への有界写像全体の集合の元 f に対して、

||f|| = sup_{x∈X} |f(x)|

とそのノルムを定義しています。

宮島さんは、実関数の有界性について曖昧にしていますが、問題ないのでしょうか?
0497132人目の素数さん
垢版 |
2023/07/02(日) 21:27:26.18ID:HjuAv88H
宮島さんのやり方に何かメリットはあるのでしょうか?
0498132人目の素数さん
垢版 |
2023/07/02(日) 22:02:26.83ID:dbd3Sy/3
ある
よく考えればわかる
よく考えてないから分からんのだよ
単に頭悪いだけかもしれんがね
0499132人目の素数さん
垢版 |
2023/07/03(月) 07:26:05.87ID:ESvq/whE
>>495
その本は読んでませんが、Cauchy性は任意のε>0に対してm,n>N⇒||f_m-f_n||<εなるNが存在することと定義されていることと思います。
m,n>N⇒f_m-f-nは有界で||f_m-f_n||<ε
と読み替えてみてはいかかでしょうか?
ちなみにf_nが非有界でもCauchy性は定義されます。例えばf_n(x)=xなら||f_m-f_n||=0でありCauchy列と解されるべきです。
0500132人目の素数さん
垢版 |
2023/07/03(月) 17:56:08.07ID:pXtbp1Gs
しかし、実際に扱うには||f||がノルムの定義を満たしていることが要求される。
少なくとも||f||=∞では、コーシー列を考えてもナンセンス
0501132人目の素数さん
垢版 |
2023/07/03(月) 18:29:29.42ID:4KEMm4h5
似たような話だけど

f(x)=g(x) in L^2

という式において

f(x)−g(x)=0 in L^2

だけでいいのか
それとも

f(x)∈L^2、g(x)∈L^2

も必要なのかどうなのか
0502132人目の素数さん
垢版 |
2023/07/03(月) 19:55:13.47ID:VLzi6Mw7
宮島静雄著『微分積分学I』

p.211 定理6.8

[a, b] 上の C^1 級関数の列 {f_n}_n が次の条件をみたしているとする:

(1) f_n はある関数 f に各点収束している;

(2) 導関数 f'_n はある関数 g に [a, b] 上で一様収束している。

このとき、 f も C^1 旧関数となり f' = g が成り立つ。

------------------------------------------------------------------------------

ふと思ったのですが、(1)って強すぎませんか?

(a) ある点 y ∈ [a, b] に対して、 f_1(y), f_2(y), … は収束している;

x ∈ [a, b] とする。

f'_1, f'_2, … は [y, x] 上の連続関数列で、 g に [y, x] 上で一様収束する。
このとき、 g は連続関数である。
このとき、 ∫_{y}^{x} f'_1(t) dt, ∫_{y}^{x} f'_2(t) dt, … は ∫_{y}^{x} g(t) dt に収束する。

よって、 f_1(x), f_2(x), … は ∫_{y}^{x} g(t) dt + lim_{n→∞} f_n(y) に収束する。

∫_{y}^{x} g(t) dt + lim_{n→∞} f_n(y) が定理6.8(1)の f である。
0503132人目の素数さん
垢版 |
2023/07/03(月) 20:57:24.33ID:+PqxRTJO
定義は論理的に矛盾がなければ何してもいいんだからより拡張されてる方を自分が好むならその定義で進めていけばいいし
拡張されてるのが嫌いならそれで進めていけばいい
後続の命題が拡張形にも適用できるかどうかはその証明次第
0504132人目の素数さん
垢版 |
2023/07/03(月) 21:21:22.89ID:+PqxRTJO
>>502
反例:f_n(x) = n

どうでもいいけど主張(a)もその証明もひどすぎるな
不正確な記述を読み手が好意的に解釈してあげなきゃ1点もあげられない
0505132人目の素数さん
垢版 |
2023/07/03(月) 21:39:15.09ID:6EuANVN/
じゃあ数学者なんていってもナンセンスな意味ない事言ってるバカばっかりやなぁと思っておけばいい
0506132人目の素数さん
垢版 |
2023/07/03(月) 23:28:32.63ID:pXtbp1Gs
>>504
> 反例:f_n(x) = n

反例になってない
n→∞で f_n(x) = n →∞でどの点でも各点収束してない
0508132人目の素数さん
垢版 |
2023/07/03(月) 23:52:41.85ID:ZNwEPV9C
だからそれじゃ(a)を満たしてないだろうと
0509132人目の素数さん
垢版 |
2023/07/03(月) 23:54:48.93ID:pXtbp1Gs
>>507
はぁ?
でも(a)は満たしているだよ
お前こそ国語が出来無いようだなw
0510132人目の素数さん
垢版 |
2023/07/03(月) 23:55:39.55ID:+PqxRTJO
はあ?(a)はおまえの主張でその次の行から(a)の証明を書いたつもりなんだろ?
(a)の主張が間違ってるということを示す反例を挙げたまで
0511132人目の素数さん
垢版 |
2023/07/03(月) 23:56:33.23ID:ZNwEPV9C
ああ、もしかして「(2)だけから(a)が言えて元々の定理が成り立つ」と言ってると勘違いしてんのか
いくら松坂くんでもそんなこと言わんだろうwwww
0512132人目の素数さん
垢版 |
2023/07/03(月) 23:59:52.63ID:pXtbp1Gs
>>502
f(x)を最後の式で定義して、それが条件を満たしてることを示せばOK

ちなみに>>503,504のID:+PqxRTJOは、数学も国語も出来ないから、相手にしない方が良い。
0513132人目の素数さん
垢版 |
2023/07/04(火) 00:00:05.02ID:V2xfsKKo
もしかして(1)の代わりに(a)でいいだろと言いたいのか?いったいどこにそう書いてる?
0514132人目の素数さん
垢版 |
2023/07/04(火) 00:01:29.90ID:TR8qzVze
ID:+PqxRTJO

お前が数学も国語も出来ないのは分かったから、もう引っ込めや。
バカを晒すだけだぞw
0515132人目の素数さん
垢版 |
2023/07/04(火) 00:03:57.37ID:TR8qzVze
>>512
お前はホンマのアホやなあ
お前があげた例は(a)の反例にすらなってない。
f_n(x)=n のどこが収束しているのかとw
0516132人目の素数さん
垢版 |
2023/07/04(火) 00:05:33.03ID:V2xfsKKo
この定理で(1)が(a)で置き換えられるってそりゃ当たり前だろw
(1)がなければ積分定数の違いしか出ないんだから

まさかそんな当たり前のことを長々と証明をつけて主張してると誰が思うんだよw
0517132人目の素数さん
垢版 |
2023/07/04(火) 00:06:00.14ID:TR8qzVze
>>502
f(x)を最後の式で定義して、そのf(x)が条件を満たしてることを示せばOK
0518132人目の素数さん
垢版 |
2023/07/04(火) 00:07:54.44ID:TR8qzVze
>>518
でもお前のあげた反例は、どの主張の反例にもなってない。

人をバカ呼ばわりする前に、自分がバカなんだと反省しろ!
0520132人目の素数さん
垢版 |
2023/07/04(火) 00:09:48.44ID:TR8qzVze
>>502
>反例:f_n(x) = n

(1)に対しても、(a)に対しても反例になってないことの、お詫びはまだ?
0521132人目の素数さん
垢版 |
2023/07/04(火) 00:11:29.96ID:TR8qzVze
間違得るのは仕方ないが、それを棚に上げて他人をバカ呼ばわりする奴は、結局墓穴を掘るということや
0523132人目の素数さん
垢版 |
2023/07/04(火) 00:39:21.72ID:TR8qzVze
>>522
素直に間違いを認めろよ
0524132人目の素数さん
垢版 |
2023/07/04(火) 00:40:54.58ID:TR8qzVze
>>504ですが、反例になっていませんでした。済みませんでした。」と謝れば済むことなのに
0525132人目の素数さん
垢版 |
2023/07/04(火) 06:54:27.47ID:YpRPYLRJ
ID:TR8qzVzeは何か勘違いしている
0528132人目の素数さん
垢版 |
2023/07/04(火) 07:41:39.80ID:YpRPYLRJ
>>527

「in L^2」なのだからf,gのどちらもL^2の元でないといけない

f(x)−g(x)=0 in L^2はf(x)−g(x)=0 a.e xなのであまり問題にならない
0529132人目の素数さん
垢版 |
2023/07/04(火) 07:45:46.96ID:m6sWqMgg
>>501
>f(x)−g(x)=0 in L^2
>だけでいいのか
という式の意味は
∫(f(x)-g(x))^2dx=0?
0530132人目の素数さん
垢版 |
2023/07/04(火) 08:11:45.31ID:gjWt525M
1/2-[1-(1/2)]=0∈Zであっても1/2=1-(1/2) in Zとは言わないだろう
0531132人目の素数さん
垢版 |
2023/07/04(火) 08:21:39.14ID:NmlRwPoY
それはもう前後の文読んでその場で判断するしかない
意味を確定させてないのが関数解析のミソ
例えばFourier変換で超関数がどうなるか議論する時“テスト関数”の空間に何を取るかが問題になる
横軸が時間の側の有界な台を持つ滑らかな関数をテスト関数の空間にとると(それが普通)周波数の方を横軸にとる方ではテスト関数をその双対空間に取らざるを得ず2つの空間で統一なんてできない
つまり関数解析で出てくる空間では同じ話の中でもこっちの方ではコレ、こっちの方ではコレ‥なんて事はよくある話で「いっつもコレ!」なんて決め打ちするとかえって話が難しくなる
だから関数解析では「毎回話は変わる、仕方ない、その話の流れではどの空間にどのノルム使ってるのか全部読み落とさないように心がける」しかない、その作業が楽々できるように修行するしかない
0532132人目の素数さん
垢版 |
2023/07/04(火) 08:48:08.36ID:YpRPYLRJ
>>531
>>501の質問と噛み合ってないよ
0533132人目の素数さん
垢版 |
2023/07/04(火) 08:50:04.26ID:YpRPYLRJ
あと、佐藤幹夫は>>531みたいな考察はどうでもいい、という立場だったね
0534132人目の素数さん
垢版 |
2023/07/04(火) 10:40:03.11ID:keV6xgoI
ある実数 c に対して、 lim_{x→a} f(x) = c



任意の ε > 0 に対しある δ> 0 があって

0 < |x - a|, |y - a| < δ ⇒ |f(x) - f(y)| < ε

が成り立つ。

-----------------------------------------------------------------------

この命題ですが、数列(コーシー列)を使わない証明を考えて下さい。

数列を使うのは不純であるように思います。
0535132人目の素数さん
垢版 |
2023/07/04(火) 10:49:30.75ID:dNLZK6iM
>>528

f(x)=g(x) in L^2 と f(x)-g(x)=0 in L^2 では意味が異なるという事で宜しいでしょうか?
0536132人目の素数さん
垢版 |
2023/07/04(火) 11:40:44.38ID:YpRPYLRJ
>>535
普通の数学書でこんな悪い書き方しないから気にしなくていい
0537132人目の素数さん
垢版 |
2023/07/04(火) 11:42:58.90ID:YpRPYLRJ
>>534
「考えてください」ってなんだよ
質問じゃないじゃん
0539132人目の素数さん
垢版 |
2023/07/04(火) 11:50:39.41ID:gjWt525M
普通にε/2で取っておけば済む話じゃないのかと
0540132人目の素数さん
垢版 |
2023/07/04(火) 11:55:01.05ID:YpRPYLRJ
>>538
g(ε)=sup{ f(x) | 0<|x-a|<δ }とおいてεを0に飛ばすのね
0541132人目の素数さん
垢版 |
2023/07/04(火) 14:55:44.71ID:keV6xgoI
(a, b) において f(x) は連続で、 lim_{x→a+0} f(x), lim_{x→b-0} f(x) が存在しないときの

∫_{a}^{b} f(x) dx

の定義について質問です。

A := (0, (a + b)/2) × (0, (a + b)/2)

と置きます。

A から R への関数 F(x, y) を

F(x, y) := ∫_{a + x}^{b - y} f(t) dt

で定義します。

lim_{(x, y) → (0, 0)} F(x, y) のことを

∫_{a}^{b} f(x) dx

とあらわすということであっていますか?
0542132人目の素数さん
垢版 |
2023/07/04(火) 14:57:30.24ID:keV6xgoI
訂正します:

(a, b) において f(x) は連続で、 lim_{x→a+0} f(x), lim_{x→b-0} f(x) が存在しないときの

∫_{a}^{b} f(x) dx

の定義について質問です。

A := (0, (b - a)/2) × (0, (b - a)/2)

と置きます。

A から R への関数 F(x, y) を

F(x, y) := ∫_{a + x}^{b - y} f(t) dt

で定義します。

lim_{(x, y) → (0, 0)} F(x, y) のことを

∫_{a}^{b} f(x) dx

とあらわすということであっていますか?
0543132人目の素数さん
垢版 |
2023/07/04(火) 15:38:47.38ID:Yjm3sBsh
∫a->b f(t)dtは、lim_{x->0, y->0} ∫_{a+x}^{b-y}f(t)dt、同時に(0,0)に極限を取る
0544132人目の素数さん
垢版 |
2023/07/04(火) 16:12:25.11ID:keV6xgoI
「同時に(0,0)に極限を取る」

とはどういうことでしょうか?
0545132人目の素数さん
垢版 |
2023/07/04(火) 16:27:02.18ID:keV6xgoI
以下の定義では駄目ですか?

A から R への関数 F(x, y) を

F(x, y) := ∫_{a + x}^{b - y} f(t) dt

で定義します。

任意の正の実数 ε に対して、正の実数 δ で以下を満たすような実数 r が存在するとします。

A ∋ (x, y) かつ 0 < √(x^2 + y^2) < δ ⇒ |F(x, y) - r| < ε

このとき、

∫_{a}^{b} f(x) dx = r

と定義する。
0546132人目の素数さん
垢版 |
2023/07/04(火) 16:31:10.39ID:keV6xgoI
訂正します:

以下の定義では駄目ですか?

A から R への関数 F(x, y) を

F(x, y) := ∫_{a + x}^{b - y} f(t) dt

で定義します。

任意の正の実数 ε に対して、正の実数 δ と実数 r で以下を満たすようなものが存在するとします。

A ∋ (x, y) かつ 0 < √(x^2 + y^2) < δ ⇒ |F(x, y) - r| < ε

このとき、

∫_{a}^{b} f(x) dx = r

と定義する。
0547132人目の素数さん
垢版 |
2023/07/04(火) 16:32:51.57ID:keV6xgoI
訂正します:

以下の定義では駄目ですか?

A := (0, (b - a)/2) × (0, (b - a)/2)

とします。

A から R への関数 F(x, y) を

F(x, y) := ∫_{a + x}^{b - y} f(t) dt

で定義します。

r を実数とします。
任意の正の実数 ε に対して、正の実数 δ で以下を満たすようなものが存在するとします。

A ∋ (x, y) かつ 0 < √(x^2 + y^2) < δ ⇒ |F(x, y) - r| < ε

このとき、

∫_{a}^{b} f(x) dx = r

と定義する。
0548132人目の素数さん
垢版 |
2023/07/04(火) 17:08:08.93ID:vidoxCuf
>>547
ダメです
理由はa, bの極限は別々に取らないといけないから
0550132人目の素数さん
垢版 |
2023/07/04(火) 19:40:24.13ID:QXGmwLjK
Let me ask a question about group theory. As for every even order finite group, do this contains an element of order 2? If your kindness makes you answer this question, I appreciate it.
0551132人目の素数さん
垢版 |
2023/07/04(火) 20:14:11.99ID:keV6xgoI
>>549

>>547

でOKということですね。
0552132人目の素数さん
垢版 |
2023/07/04(火) 21:05:59.80ID:keV6xgoI
https://i.imgur.com/6bXzodm.jpg

↑の証明ができました。

>>547

の定義がよくある定義と一致することが分かりますので、

>>547

の定義はOKです。

というより、よくある定義よりも良い定義です。
0553132人目の素数さん
垢版 |
2023/07/04(火) 21:10:25.90ID:keV6xgoI
結局、普通の2変数関数の極限ということですね。
0554132人目の素数さん
垢版 |
2023/07/04(火) 21:51:13.60ID:TR8qzVze
>>552
2変数の極限が、1変数に制限した極限と同値なわけないやん
通常の定義より、あなたの定義の仮定の方が強すぎる。
0556132人目の素数さん
垢版 |
2023/07/04(火) 22:50:43.78ID:TR8qzVze
>>557
なら、証明を書いてくれ
0557132人目の素数さん
垢版 |
2023/07/04(火) 23:05:50.38ID:TR8qzVze
>>555
「通常の広義積分の定義」⇒「あたなたの広義積分の定義」

この証明が必要です。
0558132人目の素数さん
垢版 |
2023/07/05(水) 00:51:24.13ID:mTRnwHmQ
>>554
だな
一般に1変数の累次極限から、2変数関数の極限は出てこない

反例はf(x,y)=xy/(x^2+y^2) (x,y) ≠(0,0), f(0,0)=0.
0559132人目の素数さん
垢版 |
2023/07/05(水) 01:20:00.58ID:yufhWtUn
f(x,y)=g(x)+h(y).

lim_{x,y}(f(x,y))=lim_{x}(lim_{y}(f(x,y)))=lim_{y}(lim_{x}(f(x,y)))=lim_{x}(g(x))+lim_{y}(h(y)).
0560132人目の素数さん
垢版 |
2023/07/05(水) 14:28:20.36ID:NGt8+0sW
>>557

それは非常に簡単だと思います。

むしろその逆のほうが難しいと思います。
0561132人目の素数さん
垢版 |
2023/07/05(水) 17:56:19.43ID:y8epv1ep
最近、外食が多くなった息子
愛想もなく、何げなく言った一言ですが
「俺さぁ、最近、気が付いたんだけど、うちのご飯がいちばんおいしい」って
私も愛想なく「そうなの」なんて答えましたが
心の中で号泣でした
0562132人目の素数さん
垢版 |
2023/07/05(水) 20:50:43.14ID:NGt8+0sW
宮島静雄著『微分積分学I』

一様に広義積分可能というのが登場します。
これって重要なんですか?

一様に広義積分可能であるための十分条件について述べた定理6.15の証明におかしなところを見つけました。
0563132人目の素数さん
垢版 |
2023/07/07(金) 20:25:07.25ID:6SLsJE7B
一様に広義積分可能って初めて聞きましたけどなんなんですか?
0565132人目の素数さん
垢版 |
2023/07/08(土) 12:11:47.98ID:Fgkc7PSO
>>563

定義

[a, ∞) 上の関数 f_n (n = 1, 2, …) が n について一様に([a, ∞) 上で)広義積分可能であるとは、任意の ε > 0 に対し
ある b_0 があって

b ≧ b_0 ⇒ |∫_{a}^{∞} f_n(x) dx - ∫_{a}^{b} f_n(x) dx| < ε

がすべての n について成り立つことを言う。
0567132人目の素数さん
垢版 |
2023/07/08(土) 12:32:00.29ID:1USH14CV
広義一様絶対収束
0568132人目の素数さん
垢版 |
2023/07/12(水) 01:26:10.27ID:in6Lof0L
スレが死んでる
0569132人目の素数さん
垢版 |
2023/07/12(水) 21:10:07.08ID:CmzQyOL9
京大の院試について教えてください
S²を二次元球面とする。
S²×S²の部分集合Mを以下のように置く。
M={(x,y) s.t. xy=0 x1+y1=0}
ただし、x1,y1はそれぞれ、xの第一成分yの第一成分である。
このときMは向きつけ可能であることを示せ
という問題 何かいい方法があれば教えてください
0570132人目の素数さん
垢版 |
2023/07/12(水) 21:10:11.50ID:CmzQyOL9
京大の院試について教えてください
S²を二次元球面とする。
S²×S²の部分集合Mを以下のように置く。
M={(x,y) s.t. xy=0 x1+y1=0}
ただし、x1,y1はそれぞれ、xの第一成分yの第一成分である。
このときMは向きつけ可能であることを示せ
という問題 何かいい方法があれば教えてください
0571132人目の素数さん
垢版 |
2023/07/12(水) 21:30:37.80ID:4xCU8H7r
>>570
問題文がおかしい
S²か2次元球面と言ってるだけではそこにはxyなんて演算はない
0572132人目の素数さん
垢版 |
2023/07/12(水) 21:45:25.89ID:xocsJnB2
S^2は3次元ユークリッド空間の単位球でxyは内積らしい
Mはトーラスだから当然向き付け可能
0573132人目の素数さん
垢版 |
2023/07/12(水) 23:19:59.71ID:+OJhEhF7
トーラス?
0574132人目の素数さん
垢版 |
2023/07/13(木) 00:30:30.25ID:+gycrC74
>>571
何で?内積でしょ
0575132人目の素数さん
垢版 |
2023/07/13(木) 05:53:33.44ID:4ySSC7f5
S²とかいて
{(x₁, xさふに, x₃) ∈ℝ| x₁²+x₂²+x₃²=1 }
を意味するなどという話が院試レベルででるはずがない
0576132人目の素数さん
垢版 |
2023/07/13(木) 06:00:45.30ID:yTwiskiE
まぁS²が“ℝ³の単位球面”であると問題文に書いてあるなら問題は定義式

M : Σxₖyₖ = 0

のgrad.がS²×S²上0にならない事と

x₁+y₁=0

のgrad.がM上0にならない事が言えれば向きづけ可能くらいはすぐ言える
トーラスになるのはもっと大変だろうけど
0577132人目の素数さん
垢版 |
2023/07/13(木) 06:18:50.21ID:bU33Qji4
>>576
トーラスだと思う?
0578132人目の素数さん
垢版 |
2023/07/13(木) 06:27:27.40ID:+gycrC74
M∋((a,b,c),(d,e,f))
aa+bb+cc=1
dd+ee+ff=1
ad+be+cf=0
a+d=0
0579132人目の素数さん
垢版 |
2023/07/13(木) 06:45:53.31ID:bU33Qji4
>>578
で、何でトーラス?
0580132人目の素数さん
垢版 |
2023/07/13(木) 07:17:15.81ID:mWIB4peh
まぁ多分x₁≧0の部分がアニュラス(S¹×I)になるんやろな
でx₁=0の部分がその境界でS¹2つ、x₁≦0も同じでいわゆる“アニュラスのダブル”になるからトーラスになるとか示せるんじゃないの
知らんけど
0581132人目の素数さん
垢版 |
2023/07/13(木) 07:27:36.42ID:bU33Qji4
>>580

>>まぁ多分x₁≧0の部分がアニュラス(S¹×I)になるんやろな

そう思える理由は?
0582132人目の素数さん
垢版 |
2023/07/13(木) 07:42:22.52ID:+gycrC74
bb+cc=ee+ff=1-aa
be+cf=aa
0583132人目の素数さん
垢版 |
2023/07/13(木) 08:09:24.22ID:L1RaWTMV
>>581
ここまで書いてわからんならこのレベルの問題に挑戦する資格がない
0584132人目の素数さん
垢版 |
2023/07/13(木) 08:43:08.96ID:+gycrC74
a=0
|(b,c)|=|(e,f)|=1
(b,c)(e,f)=0
((b,c),(e,f))↔S×2
a>0(or a<0)
|(b,c)|=|(e,f)|=k<1
(b,c)(e,f)=kkcosθ=1-kk
2≧1+cosθ=1/kk>0
1>k≧1/√2
k=1/√2→θ=0→(b,c)=(e,f)
1>k>1/√2→0<θ≦π/2→(b,c)=2点for(e,f)
((b,c),(e,f))↔S×I
S×I×2/S×2=トーラス
0585132人目の素数さん
垢版 |
2023/07/13(木) 08:46:53.59ID:+gycrC74
>>584
>S×I×2/S×2=トーラス
(S×I)∪_(S×2)(S×I)=トーラス
0586132人目の素数さん
垢版 |
2023/07/13(木) 08:48:01.52ID:+gycrC74
>>585
>(S×I)∪_(S×2)(S×I)=トーラス
(S×I)∪_(S×∂I)(S×I)=トーラス
0587132人目の素数さん
垢版 |
2023/07/13(木) 09:12:30.12ID:bU33Qji4
>>583
クラインボトルでない理由は?
0588132人目の素数さん
垢版 |
2023/07/13(木) 09:34:18.21ID:iUK8n0Ck
ID:bU33Qji4
何このゴミ
0589572
垢版 |
2023/07/13(木) 09:41:51.62ID:ewq9rcfG
xとyの中点はyz平面上にあり、原点からの距離は1/√2.
この中点を固定するとxの動ける範囲は中点を中心とする半径1/√2の円上。
例えばx軸正の方向からの角度を考えてやればクラインボトルではなくトーラスであることがわかる。
試験で解くなら実際に微分同相作ってしまった方が早いかもしれないけど。
0590132人目の素数さん
垢版 |
2023/07/13(木) 09:44:49.74ID:ewq9rcfG
問題文のx,yと第一成分、第二成分の意味のx,yが混じってしまった。すまねえ
0591132人目の素数さん
垢版 |
2023/07/13(木) 11:31:17.30ID:Sx5x467y
>>589
ここでもそっちの方が早いのでは?
0592132人目の素数さん
垢版 |
2023/07/13(木) 13:11:27.64ID:MQH69Wy/
そもそもxy=0がxy≧0の境界なので向きづけ可能、x₁+y₁=0がx₁+y₁≧0の境界なので向きづけ可能なんだからクラインボトルなわけない
0593132人目の素数さん
垢版 |
2023/07/13(木) 14:25:28.44ID:Sx5x467y
一般に
向き付け可能な4次元可微分多様体上に
可微分な実関数f,gがあり、f=g=0で
2次元の部分多様体が定まれば
それは常に向き付け可能である。
(特にクラインボトルであるわけがない)
従って、それがトーラスかどうかは関係なく
向き付け可能性は結論できる。
0595132人目の素数さん
垢版 |
2023/07/13(木) 18:01:09.73ID:Sx5x467y
fをテイラー展開し、|z|=1のとき\overline{z}=1/zであることを使って
有理式の級数の線積分に直すとできそう
0596132人目の素数さん
垢版 |
2023/07/13(木) 20:10:52.72ID:SUF63AF7
「絶対収束する無限級数は順序を変えても結果が変わらない」という命題の最も一般的な述べ方って何ですか?

全単射によって順序交換する場合のみならず
Nを直和分解して元の級数を「無限級数の無限級数」として表すこともできる
さらに無限級数の無限級数の無限級数と表すこともできる

超限帰納法とかでなんかできませんか?
0597132人目の素数さん
垢版 |
2023/07/14(金) 00:32:18.25ID:HrRVt/uq
絶対収束の無限和を一般的に定義すれば良い
正数の集合S(無限集合も可)の有限部分集合A⊂S
を考えてAの和
ΣA = Σ_{a∈A} a
の上限をSの和
ΣS = sup{ΣA | A⊂S}
と定義すれば順序関係なく一意の和が定義できるし
どの順序とも同じ結果と証明できる
なお、結果が有限値なら a>0 となる要素は
可算個しかない
0598132人目の素数さん
垢版 |
2023/07/15(土) 12:29:23.51ID:Tm3xQdHF
曲線Cの弧長が連続写像の取り方によらないことを証明したいんですが、
f:[a, b]→Cとg:[c, d]→Cが連続な全単射ならば、f^{-1}◦g:[c, d]→[a, b]って単調ですか?
0599132人目の素数さん
垢版 |
2023/07/15(土) 14:37:52.16ID:5NxpecUb
>>598
じゃないの?同相てことでしょ
0600132人目の素数さん
垢版 |
2023/07/15(土) 16:43:36.67ID:oioNldiz
>>599
単調だとは予想してるんですが、どうやって証明しますか?
0601132人目の素数さん
垢版 |
2023/07/15(土) 16:51:40.72ID:5NxpecUb
>>600
背理法でどうぞ
0602132人目の素数さん
垢版 |
2023/07/15(土) 21:26:08.18ID:Tq95PeGz
内田集合ツォルンの補題の証明、(1)のW_1<y>=W_2<φ(y)>で詰まっています。
W_1<y>⊂W_2<φ(y)>はx<yなるx∈W_1に対して、φ(x)=xとなることから容易に証明出来たのですが、逆側の包含がわからないのです。
0603132人目の素数さん
垢版 |
2023/07/15(土) 21:39:27.95ID:2Qx9XMtf
>>601
h= f^{-1}◦gが単調じゃないと仮定すると、
あるp<qとr<sが存在してh(p)<h(q)とh(r)>h(s)を満たすというところまでは分かったんですが、
その後どうしたらいいか分かりません
0605132人目の素数さん
垢版 |
2023/07/16(日) 09:30:43.62ID:Ja8EtNE9
>>604
示せてろよ
0606132人目の素数さん
垢版 |
2023/07/16(日) 13:40:41.68ID:8pxDvJ+b
>> 603

次のように証明できます:

定理: J を R の区間, f : J → R を連続な単射とする時, f は単調である.

証明. D = {(x, y) ∈ J × J | x>y } と置き, (x, y) ∈ D に対して
g(x, y) = (f(x) - f(y))/(x - y) と置くと, g : D → R は連続である.

そこで, f が単調でないと仮定すると, z_1 = (x_1, y_1) ∈ D と
z_2 = (x_2, y_2) ∈ D が存在し, g(x_1, y_1) > 0 かつ g(x_2, y_2) < 0 となる.
そこで, D が R^2 に於ける凸集合であることに注意すると,
中間値の定理より, z = (x, y) ∈ {t*z_1 + (1-t)*z_2 | t ∈ [0, 1]} (⊆ D) が
存在し, g(x, y) = 0 となる. これは f の単射性に反する.
よって, f は単調. Q.E.D.
0607132人目の素数さん
垢版 |
2023/07/16(日) 13:49:06.54ID:C/ZgYNxI
>>606
f:[a, b]→Cとg:[c, d]→Cが連続な全単射ならば、f^{-1}◦g:[c, d]→[a, b]って連続ですか?
0608132人目の素数さん
垢版 |
2023/07/16(日) 13:50:29.40ID:Ja8EtNE9
連続なんだから上がったり下がったりだったら1対1にならないのはホボホボ自明
0609132人目の素数さん
垢版 |
2023/07/16(日) 13:59:33.47ID:Xpc+Nrax
>>608
定理を使わず3行くらいで示せますか?
0610132人目の素数さん
垢版 |
2023/07/16(日) 14:03:18.59ID:Ja8EtNE9
>>607
f,g:同相はホボホボ自明
0611132人目の素数さん
垢版 |
2023/07/16(日) 14:08:04.86ID:8pxDvJ+b
>>607

C がたとえば R^2 内の曲線の像で, f : [a, b] → C が連続な全単射ならば,
[a, b] がコンパクトハウスドルフ, C がハウスドルフであることより,
f が位相同型であることがわかります. 同様に, g も位相同型です.

従って, f^{-1}◦g:[c, d]→[a, b] は連続です.
0612132人目の素数さん
垢版 |
2023/07/16(日) 16:51:48.18ID:3UdwoJiy
確率論の質問です
以下の画像の問題で(3)以降の解き方がわからないのでご教示頂きたいです
(3)の期待値を∫|XY|×(|XY|の確率密度関数)で求めようとしたのですが、(4)が確率密度関数を求める問題になっているため、恐らく他の定義で期待値を求めるんだと思うんです
その求め方がわかりません
よろしくお願いします

https://i.imgur.com/krfh3BE.jpg
0613132人目の素数さん
垢版 |
2023/07/16(日) 19:48:44.05ID:Ja8EtNE9
>>612
D:x+y<1 |XY|=1-x-y
¬D:x+y>1 |XY|=x+y-1
|XY|=|x+y-1|
E(|XY|)=∬|x+y-1|dxdy=1/3
P(|XY|>t)=(1-t)^2
ρ(t)=(d/dt)P(|XY|<t)=2(1-t)
0614132人目の素数さん
垢版 |
2023/07/16(日) 19:55:17.28ID:Ja8EtNE9
透けてるのも読めそう
0615132人目の素数さん
垢版 |
2023/07/16(日) 20:32:25.00ID:3UdwoJiy
>>613
ありがとうございます!
(4)はP(|XY|≦t)なのですが、場合分けしてそれぞれ微分すれば答えになりますかね?

透けてるのはポアソン分布の問題です
自分で考えてみてわからないところがあったらまた質問させて頂きます
https://i.imgur.com/v2vW34d.jpg

答えて頂きありがとうございました!
0616132人目の素数さん
垢版 |
2023/07/16(日) 21:23:58.70ID:Ja8EtNE9
>>615
>場合分けしてそれぞれ微分すれば
?なんで?
0617132人目の素数さん
垢版 |
2023/07/16(日) 21:38:16.87ID:Ja8EtNE9
>>615
>https://i.imgur.com/v2vW34d.jpg
E(U)=V(U)=λ(公式だが算出もすぐ)
E(X)=Σsin(Uπ/2)p(U)=Σ(-1)^np(2n+1)=Σ(-1)^ne^(-λ)λ^(2n+1)/(2n+1)!=e^(-λ)sinλ
Y+Z〜Po(2λ)(公式だが算出もすぐ)
0618132人目の素数さん
垢版 |
2023/07/16(日) 22:04:25.24ID:3UdwoJiy
>>616
すみません確率密度関数を勘違いしてました
理解できました

>>617
こちらも解いて頂きありがとうございます!
お陰様で理解が深まりました
0619132人目の素数さん
垢版 |
2023/07/17(月) 17:27:41.14ID:94hKTpzP
大学数学(確率)自信ニキ来て✋
sannan.nl/test/read.cgi/livegalileo/1689581669/

・確率空間と測度論
・マルコフ連鎖
の演習問題
外部板の住民は誰も手が出ないらしい
0620132人目の素数さん
垢版 |
2023/07/17(月) 18:16:41.45ID:94hKTpzP
スレ落ちた
kako.sannan.nl/test/read.cgi/livegalileo/1689581669/

問題文
ttps//i.imgur.com/rR0fmQS.jpeg
ttps://i.imgur.com/9Zl8AJP.jpeg

1枚目 問1
(i) 0
(ii) ファトゥの補題より明らか
まで進んで終了

どこの大学だったのか気になる
0621132人目の素数さん
垢版 |
2023/07/17(月) 23:27:57.12ID:WJ77G65/
404
0622132人目の素数さん
垢版 |
2023/07/18(火) 09:36:41.89ID:9tQXc4Vs
質問です

無向グラフが閉路を持たないことと頂点から頂点への道が一通りに定まる事って同値だと思うんだけど証明が思いつかん

単に勉強してて思いついた問題で証明しないと先に進めない訳ではないけど…
0624132人目の素数さん
垢版 |
2023/07/18(火) 12:37:09.16ID:U8+P2LdC
数学的帰納法は?
0625132人目の素数さん
垢版 |
2023/07/18(火) 13:17:31.87ID:uHA8ErRk
連結なら道がある
異なる2つの道があるならループがある
0626132人目の素数さん
垢版 |
2023/07/18(火) 13:21:05.86ID:AQEn33Mt
>>624
やってみた


頂点が(A,B)のみの時自明

Aを出発点として固定する
Bに閉路を作らないよう頂点C〜Cnと結ぶ辺をn個付け加えるとA〜Ckへの道は一意
以下数学的帰納法のアルゴリズムは自明なので略


ある全域木Tの2つの頂点(A,B)を結ぶ道が2通りあったとする
1つ目の道P1に含まれるある頂点Cから分岐したものがP2であるが、P2はP1のCと異なる頂点Dで合流してしまうので閉路を作ってしまう
よって道が1通りだとしたら閉路はない

⇦の証明が自然言語だらけになったのが気に入らん
0628132人目の素数さん
垢版 |
2023/07/18(火) 16:45:43.11ID:jl1x5UAk
関数fをテイラー展開して、ベキ級数で表したときに、収束半径が定まります。
その収束半径の内側で、オリジナルのfが定義されないことってありますか?
0629132人目の素数さん
垢版 |
2023/07/18(火) 16:50:40.23ID:+hWmvjLl
>>622
ループあり→¬(一通りに定まる) 自明
¬(一通りに定まる)→ループあり 自明
0630132人目の素数さん
垢版 |
2023/07/18(火) 20:08:20.26ID:+RztCiP+
>>628
fの定義域を人為的に狭く制限すれば、テイラー級数の収束円より小さくなることはある
0631132人目の素数さん
垢版 |
2023/07/18(火) 21:08:03.90ID:jl1x5UAk
>>630
「人為的」、あるいは「自然な」の数学的な定義を教えてください。

(x + x^2 + x^3 + …)^k を冪級数展開したときに、 n 次の項の係数がいくらになるか?

という問題ですが、神保道夫さんは、 n*(n-1)*…*(n-k+1) / (k-1)! であると書いています。(『複素関数入門』p.142)

ですが、これは、 Binomial(n-1, k-1) が正解だと思います。

(x + x^2 + x^3 + …)^k を冪級数展開したときに、 n 次の項の係数がいくらになるか?

という問題の標準的な解答を教えてください。
0632132人目の素数さん
垢版 |
2023/07/18(火) 21:13:11.20ID:jl1x5UAk
Binomial(n-1, k-1) という答えは、2項定理を利用して得ましたが、組合せ論的にはどうすればいいでしょうか?
0633132人目の素数さん
垢版 |
2023/07/18(火) 21:26:44.80ID:RwN7N92z
>>631
うーん数学以前の問題だな
0635132人目の素数さん
垢版 |
2023/07/18(火) 23:04:26.38ID:ou+IudsB
>>632
xxx…xになるために最初から〜最後から取り出せばいいので
n個のものを順序付きにk組に分類(一組は1個以上含む)する総数
それは
n個のものの間n-1ヶ所のうちk-1ヶ所の境目を選択する総数

x^k(1+x+x^2+…)^kにすると
n-k個のものを順序付きにk組に分類(0個の組も許す)する総数
それは
n-k個と境目k-1個の合計n-1の並べ方の総数だから
n-1個のうち境目k-1個を選択する総数
0636132人目の素数さん
垢版 |
2023/07/19(水) 08:05:36.90ID:yxqIMw1P
関数fのテイラー級数の収束半径が2だったとする
fの定義域を半径1の円に制限する
はい終わり
0637132人目の素数さん
垢版 |
2023/07/19(水) 08:40:20.83ID:fGqyMt+x
>>636
ツマンネ
0638132人目の素数さん
垢版 |
2023/07/19(水) 08:47:23.32ID:fGqyMt+x
>>635
>x^k(1+x+x^2+…)^k
(1+x+x^2+…)^k=(1-x)^(-k)
(∂/∂x)^(n-k)(1-x)^(-k)=k(k+1)…(n-1)(1-x)^(-n)
k(k+1)…(n-1)/(n-k)!=(n-1)!/(k-1)!(n-k)!
0639132人目の素数さん
垢版 |
2023/07/19(水) 09:20:53.41ID:Jvlm+9PB
すいません、微分方程式の級数解法のところで正則点と確定特異点の
違いがわかりません、、y''-y'/x+(x^2)y=0という微分方程式があって
解答にはx=0で確定特異点をもつから〜と書いてあるのですが
x^2は二次関数だからx=0で微分可能だからyの方の係数関数はx=0で正則点を
もっと思いました。確定特異点はy'とyの両方で特異点を持つときのことをいうと思うのですが
自分の認識はあってますでしょうか、どなたか教えていただけないでしょうか
0640132人目の素数さん
垢版 |
2023/07/19(水) 09:53:05.38ID:fGqyMt+x
>>639
x=0は定義域外じゃん
0641132人目の素数さん
垢版 |
2023/07/19(水) 10:00:58.25ID:Jvlm+9PB
>>640
すいません、自分の頭が悪くて理解できません泣
微分方程式に定義域があるとは思いませんでした、式を見てどうやってx=0が
定義域外なのか判断できません、教えていただけないでしょうか
0643132人目の素数さん
垢版 |
2023/07/19(水) 10:45:48.55ID:m653ywGv
>>637
デデンネ、な
0644132人目の素数さん
垢版 |
2023/07/19(水) 12:01:07.57ID:C66hCoSB
>>642
ありがとうございます!
0645132人目の素数さん
垢版 |
2023/07/19(水) 15:56:14.65ID:d+Lqp/4A
>>639
>y''-y'/x+(x^2)y=0
けんど
xy”-y’+x^3y=0
ならば?
0646132人目の素数さん
垢版 |
2023/07/20(木) 02:36:05.18ID:SMbrUUXa
「Rを単位的可換環とする。pを素数とし、Rでp=0であるとき、(x +y)^p=x^p +y^p (x,yはRの元)であることを示せ」
と問題がネット上にあったのですが「Rでp=0」という表現は正しいですか?標数pのことを指しているのは分かるのですが…
0647132人目の素数さん
垢版 |
2023/07/20(木) 03:32:46.77ID:P83gM4TG
それは質問ではなくイチャモンだ
標数pと伝わってる時点で何も問題ない
0649132人目の素数さん
垢版 |
2023/07/20(木) 05:49:57.63ID:PaZqdvMG
いちゃもんとかではなくて、p=0という表記が引っかかった。正しくはどう書くのか?例の人とは別人です。
0651132人目の素数さん
垢版 |
2023/07/20(木) 06:22:55.59ID:Nvi7i1Wd
>>649
>正しく
正しくというのが引っかかるけど
標数pと書くのが通例
でもそれは>>647で指摘されていることからも分かるのでは
0652132人目の素数さん
垢版 |
2023/07/22(土) 06:55:21.37ID:602tYaGt
219 それでも動く名無し 2023/07/17(月) 20:26:51.53 ID:pA5+SQtP0
すっげぇかわいいのにおっぱいも綺麗で大きいし、尻もエロい
https://i.im;gur.com/62eA5KE.jpg
https://i.im;gur.com/St2P7wG.jpg
https://is;.gd/xkUfeT

620 名無しさん@ピンキー sage 2023/07/17(月) 17:36:57.85 ID:AS4vmq4R0
不朽の名作が復活していたので
https://i.im;gur.com/C239AFO.jpg
https://i.im;gur.com/l35c3gZ.jpg
https://is;.gd/Q5ZYzk
0654132人目の素数さん
垢版 |
2023/07/22(土) 19:33:35.61ID:AIJvImFB
工学修士です。要するに数学は専門外です

定義だけ見るとルベーグ積分では微積分学の基本定理が成り立っていないように見えるんだけどこの事はどういうふうに理解すればいいのでしょうか
実際リーマン積分可能だけどルベーグ積分不可能な関数も作れるし

何が言いたいかというとリーマンの意味で積分する限りほぼ自明な逆写像の存在がよく分かんないのも自分が測度論をよく分からない理由の一つだと気づいた
0655132人目の素数さん
垢版 |
2023/07/22(土) 19:57:49.92ID:jX9rDuOG
>>654
>微積分学の基本定理
連続関数の定理ね
0656132人目の素数さん
垢版 |
2023/07/22(土) 20:39:19.84ID:oZ0w8+bJ
> 実際リーマン積分可能だけどルベーグ積分不可能な関数も作れるし

そんなの可能でしたっけ?
0657132人目の素数さん
垢版 |
2023/07/22(土) 20:56:42.42ID:hf8xWuUn
>>656
広義積分じゃね
0658132人目の素数さん
垢版 |
2023/07/22(土) 21:26:54.02ID:ixzkacH3
>>654
こういうことに興味を持つ反面、勉強はしない。質問だけ投下する馬鹿って定期的に出現するよな

こういう馬鹿って教科書の演習問題を解き、分からない問題を質問するという正道から外れちゃってるんだよな
0659132人目の素数さん
垢版 |
2023/07/23(日) 06:57:15.57ID:mZe/OH+8
>>654
まずルベーグの論文を読んだらよい
0660132人目の素数さん
垢版 |
2023/07/23(日) 10:02:42.48ID:JgjlXIj1
n次方程式x^n+(a_1)x^(n-1)+…+(a_(n-1))x+a_nで
係数a_1〜a_(n-1)を固定してa_nをパラメータと思ってn個の実解をR^nにプロットしたいんですが
実解をn個持ち、臨界的にならない良い係数の場合
n=1,2のときは直線
n=3,4のときは円(と同相)
になるようですがn≧5のときはどんな図形になるんでしょうか?
0661132人目の素数さん
垢版 |
2023/07/23(日) 10:14:42.15ID:NphhdSCU
基本対称式からベキ対称式への変換と、1次対称式をx_n=0平面に座標変換することで
n≧3のときは
Σ[k=1,n-1](x_k)^i=C_i (2≦i≦n-1, C_iはある定数)
の共通部分(良い係数のとき、これらが横断的に交わる)になると思うのですが、形が良くわからないです
0663132人目の素数さん
垢版 |
2023/07/23(日) 13:33:50.81ID:SDr0n6x0
n=1〜4の時と同じ議論繰り返すだけじゃないの
例えばx(x-1)(x-4)(x-10)=kなら24個の線分繋ぐだけやろ
この場合円4個やろ
0664132人目の素数さん
垢版 |
2023/07/23(日) 15:56:29.23ID:STuWbT83
>>662
重解を持たないように動かすので適当に順序を決めて考えていました
確かに重解も含めた場合、R^nの中で考えるのは不自然ですね
R^n/S_nみたいな空間で考えるべきなんでしょうか?

>>663
すみません、24個の線分をつなぐというのが理解できてないです
どういうことですか?
0665132人目の素数さん
垢版 |
2023/07/23(日) 16:30:54.66ID:34exI2a5
以下のアルゴリズムで素数を返す関数fがリーマン予想の証明に使えない理由って何?

f(x)
x=1の時、2から順番に素数判定を実行し1番目に素数となった数2を返す
x=2の時、2から順番に素数判定を実行し2番目に素数となった数3を返す

以下略
0666132人目の素数さん
垢版 |
2023/07/23(日) 16:34:14.27ID:z5E5Ut2L
そもそも論としてx^3 -3x = kの場合円になる事示すのに線分6個繋ぐ以外の方法思いつかん
0667132人目の素数さん
垢版 |
2023/07/23(日) 16:47:56.22ID:q48EiD+J
f(x) = k‥①において、これが異なるn個の実数解持つkの範囲を(α,β)であるとする
k ∈ (0,1)に対して①の異なるn個の解を昇順にa₁‥aₙとする
pを{1,..,n}の置換とする時ℝⁿの開曲線Iₚを

Iₚ = {(aₚ₍₁₎(t), ..,aₚ₍ₙ₎(t)) | t∈(α,β)}

とし、I̅ₚをその閉包とする
このとき件の図形Fは

F = ∪_p I̅ₚ

である
0668132人目の素数さん
垢版 |
2023/07/23(日) 17:10:15.30ID:lDY4MDjk
>>665
使えないの?
0669132人目の素数さん
垢版 |
2023/07/23(日) 17:11:51.46ID:lDY4MDjk
>>663
n=2のときって?
0670132人目の素数さん
垢版 |
2023/07/23(日) 22:01:35.77ID:NE/gYr72
>>658
質問スレなんだから大学数学の何を質問しようが自由だろうが。あと、馬鹿とか人格攻撃もやめとけ。
0671132人目の素数さん
垢版 |
2023/07/23(日) 22:06:08.13ID:mZe/OH+8
人格攻撃はよくない
0672132人目の素数さん
垢版 |
2023/07/23(日) 22:31:00.95ID:LD7baZQv
> 質問だけ投下する馬鹿
質問以外に何を期待してるんだ…
0673132人目の素数さん
垢版 |
2023/07/23(日) 23:34:12.71ID:34exI2a5
定義の意味がよく分かりませんってだけの質問にブチ切れるのは謎よね
0674132人目の素数さん
垢版 |
2023/07/24(月) 11:40:26.69ID:hRK7uMtn
>>672
自分で勉強すること
を当然のように期待する

馬鹿だからそれが読み取れないのか本人が悔しがって時間差でレスしてるのか…
0675132人目の素数さん
垢版 |
2023/07/24(月) 12:48:15.30ID:luNr129T
遅age
0676132人目の素数さん
垢版 |
2023/07/24(月) 14:49:52.64ID:5FAW4AqZ
>>670
バカとか他人を攻撃する奴は、1,2年の数学を分かってないから、
5chで必死にマウントを取りたいんだよ
0677132人目の素数さん
垢版 |
2023/07/24(月) 16:11:34.23ID:5dE5zsXI
>>667
クリティカルポイントで重根化するとき
最初n個別別だったんだから
隣同士がくっつくけれど
それが2カ所(2根+2根)とか3カ所とかで
同時に起こったら?
0678132人目の素数さん
垢版 |
2023/07/24(月) 16:51:36.07ID:5dE5zsXI
>>664
>重解を持たないように動かすので適当に順序を決めて考えていました
順序例えば大小順に決めたら
重根の無いx^n+(a_1)x^(n-1)+…+(a_(n-1))x=kに対して
根を座標成分にする点は1つだからただの線分では?
順序は決めず
座標成分のどれもが根になるような点の全体なのでは?
これなら
x^2=k
で各k>0に対して2点(√k,-√k)と(-√k,√k)が対応してk=0で2点が1点になるから
1点から半直線2本が出てることになって全体は直線
x^3-3x=k
で-2<k<2のとき3!=6点対応してk=±2で3点に減るとき
k=2でとk=-2でとで一致の仕方がズレるから全体で閉曲線
けんど
x^4-2x^2=k
なら
-1<k<0で4!=24点対応するうちk=0で12点
k=-1で6点に減るから
k=0では2点が1点にk=-1では4点が1点になるから
閉曲線4つがそれぞれ3点ずつでお互い接してるみたいな
なんて言うか正四面体の各面に内接する円4つでできた図形みたいになるけんど
4根が同時に2重根になるんじゃ無いんなら4つの分離した閉曲線になるけんど
0679132人目の素数さん
垢版 |
2023/07/24(月) 17:00:14.79ID:P5xbJrr7
>>677
もちろん出来上がる図形が“一次元多様体”でなくなるだけ
例えばn=6、a<b<c<d<ex=a,c,eで極小値α、x=b,dで極大値、α=f(a) = f(e) > f(c)、β=f(b) = f(d)なら線分の数は72個
各線分はαの側に対応する端とβの側にある端をもち、6個の解α₁〜α₆がℝ⁶のどの成分に割り当てられているかで決まる
そして下端ではα₁α₂、α₅,α₆が同一の値を取り“つながる”
例えば
(α₁、α₄、α₅、α₂、α₆、α₃)
(α₁、α₄、α₆、α₂、α₅、α₃)
(α₂、α₄、α₅、α₁、α₆、α₃)
(α₂、α₄、α₆、α₁、α₅、α₃)
の4本の線分が“下端”でつながる、よってここが4分岐になる
上端も同様
よっていつぱんには分岐を持つグラフになる
0681132人目の素数さん
垢版 |
2023/07/24(月) 17:20:55.20ID:5dE5zsXI
>>678
>なんて言うか正四面体の各面に内接する円4つでできた図形みたいになるけんど
書き直したら正八面体の辺の集合だった
0683132人目の素数さん
垢版 |
2023/07/24(月) 21:36:29.55ID:YzDdvQbC
はじのところがいい加減やけど巡回行列の固有多項式やろ
wikiにもあるんちゃう?
0684132人目の素数さん
垢版 |
2023/07/25(火) 23:53:59.55ID:X13bmn58
>>682
d0=1
d1=x
d2=xd1-abd0=xx-ab=(x-√ab)(x+√ab)
d3=xd2-abd1=xxx-2abx=x(x-√2ab)(x+√2ab)
d4=xd3-abd2=xxxx-3abxx+aabb=(xx-(3-√5)ab/2)(xx-(3+√5)ab/2)=(x-((√5-1)/2)√ab)(x+((√5-1)/2)√ab)(x-((√5+1)/2)√ab)(x+((√5+1)/2)√ab)
0685132人目の素数さん
垢版 |
2023/07/26(水) 09:19:33.38ID:03Q+r0As
>>678
すごく丁寧にありがとうございます!(返事が遅くなってしまいました)
すみません、重根の箇所を含めないとダメでしたね
なるほど、一般のn!個の解たちは各次数の基本対称式=定数の曲面(これらは次数順に曲率が異なる)の共通部分を順に取ったときにn単体を切頂しn個のn-1単体が、各n-1単体を切頂しn×(n-1)個のn-2単体が…、最終的にはn!の点が現れるようですね
1番大きな曲率のn次対称式=定数(=多項式の定数項に対応)を動かしたとき、n単体上の点が退化していく
これがn=4のときは四面体の各辺の中点を結んだ八面体になると…

しかし不思議なのは(これは>>666への返答にもなりますが)
自分が考えたのは1次からn-1次までの基本対称式を固定するというのは1次からn-1次までの冪対称式を固定することと同値だから
n=4のときの解曲線は
w+x+y+z=k_1
w^2+x^2+y^2+z^2=k_2
w^3+x^3+y^3+z^3=k_3
の共通部分となり3番目の曲面は平面と同相なんだから、結局3次元球面を2枚の超平面で切るので円になると考えたんです
しかしこの説明は変数を1つ落として考えた類推なので4次元空間だと想像以上に1番目と3番目の面が変な交わり方をしてるということなんですかね…
0686132人目の素数さん
垢版 |
2023/07/26(水) 09:41:53.73ID:mmKvSq6l
小林亮、高橋大輔著『ベクトル解析入門』

x(t_n + Δt) - x(t_n) = x'(t_n) * Δt + O(Δt^2)
y(t_n + Δt) - y(t_n) = y'(t_n) * Δt + O(Δt^2)

√((x(t_n + Δt) - x(t_n))^2 + (y(t_n + Δt) - y(t_n))^2) = √(x(t_n)^2 + y(t_n)^2) * Δt + O(Δt^2)


と書いてあります。

√((x(t_n + Δt) - x(t_n))^2 + (y(t_n + Δt) - y(t_n))^2) = √(x(t_n)^2 + y(t_n)^2) * Δt + O(Δt)

が正しいと思いますがどうですか?
0687132人目の素数さん
垢版 |
2023/07/26(水) 09:57:16.93ID:mmKvSq6l
あ、 O(Δt^2) ですね。
0688132人目の素数さん
垢版 |
2023/07/26(水) 14:03:13.86ID:pLjwfvJ3
考えずに書込み その後で自己訂正する
という安直なやり方を繰り返し 馬鹿がより馬鹿になっていく…
大局的に見れば単に「ボケ老人の思いつきの精度が低い」という当たり前の現象
0689132人目の素数さん
垢版 |
2023/07/26(水) 21:25:17.50ID:pSuoNGSP
>>685
あ、いや>>678は重根が同時に発生する特殊な例だからいいのかな
最も一般的な状況は境界を重根1つだけで迎える場合で、このときは3次多項式のときと同じく2つの解が1つになる境界に挟まれてるから、繋ぎ方によっては円になりそうですね
ただ、繋ぎ方によっては4つの円にもなりそうだから、そこはもう少し定性的に見ないとダメか
0690132人目の素数さん
垢版 |
2023/07/26(水) 21:32:13.68ID:pSuoNGSP
いや、対称性から4つの円になるのは当たり前か
というかそのこともちゃんと最後に書いてくれてますね(今気づいた…)
0691132人目の素数さん
垢版 |
2023/07/26(水) 22:40:27.75ID:2/OnRVqO
そもそも位相的性質を調べるだけなら多項式で考える意味はほとんどない
考える空間は
X = ∪[k] { (a₁,...,aₙ) | aₜはすべて相異なる、f(aₜ)=k}
でkは{...}がn!元となる範囲で動く
とした時の閉包X̅
ここでℝの同相p:ℝ→ℝを合成してg = fpにして同様にY,Y̅を作ればX,Yはℝⁿの中の“位置、寸法”が違うだけで物は同相
だから最初からy=f(x)は極値をn個持つ折れ線としても話同じ
そしてその場合Xはℝⁿのn!個の“ホントの線分を繋いだ空間”と思っていい
だからf(x) = |x-1| - |x+1| + x
とかでやればどんな6点繋いだ図形かすぐわかる
0692132人目の素数さん
垢版 |
2023/07/26(水) 22:58:10.42ID:JTFCGyW8
繋がり方を見るだけならそれで十分でしたね
結論から言えばn次方程式の場合は一般にn!/6個の円になる
ただ自分がこれを考えたかったのは定数項が未定の場合の解たちに残る1次元的な対称性が何か、そして出来れば対称的な形でパラメータ表示をしたい、ということだったんです
3次の場合は真円になります

4次の場合
w+x+y+z=k_1
w^2+x^2+y^2+z^2=k_2
w^3+x^3+y^3+z^3=k_3
の共通部分がw+x+y+z=k_1という3次元空間で見たときどういう形の4円になるか依然として気になっています
これをplotできる方いればお願いしたいですm(_ _)m
0693132人目の素数さん
垢版 |
2023/07/26(水) 23:17:31.35ID:cjTTGmdK
>>689
>繋ぎ方によっては4つの円にもなりそう
a<k<bのk=aでxi=xi+1となりk=bでxj=xj+1となり
i,i+1,j,j+1がすべて異なる場合(i+1<jとする)
L1:(x1,…,xi,xi+1,…,xj,xj+1,…,xn)
L2:(x1,…,xi+1,xi,…,xj,xj+1,…,xn)
L3:(x1,…,xi,xi+1,…,xj+1,xj,…,xn)
L4:(x1,…,xi+1,xi,…,xj+1,xj,…,xn)
の4つの軌跡の線分が
k=aでL1とL2, L3とL4がつながり
k=bでL1とL3, L2とL4がつながるので
この場合は4本の線分で閉曲線(円)を形成
しかし(3次の場合と同様な)
k=aでxi=xi+1となりk=bでxi+1=xi+2となるような場合は
6本で閉曲線(円)を形成するから
重根の現れる場所によってn!/4個の円かn!/6個の円かになる
前者は5根以上必要だからn≧5の場合
後者は3根以上で起こりえるからn≧3の場合
0694132人目の素数さん
垢版 |
2023/07/27(木) 01:00:01.98ID:cSCOo7pq
((x+y+z)/2)+((x-y-z)/2)+((-x+y-z)/2)+((-x-y+z)/2)=0.
((x+y+z)/2)^2+((x-y-z)/2)^2+((-x+y-z)/2)^2+((-x-y+z)/2)^2=x^2+y^2+z^2.
((x+y+z)/2)^3+((x-y-z)/2)^3+((-x+y-z)/2)^3+((-x-y+z)/2)^3=3xyz.
0695132人目の素数さん
垢版 |
2023/07/27(木) 01:20:01.78ID:cSCOo7pq
((x+y+z)/2+a)+((x-y-z)/2+a)+((-x+y-z)/2+a)+((-x-y+z)/2+a)=4a.
((x+y+z)/2+a)^2+((x-y-z)/2+a)^2+((-x+y-z)/2+a)^2+((-x-y+z)/2+a)^2=x^2+y^2+z^2+4a^2.
((x+y+z)/2+a)^3+((x-y-z)/2+a)^3+((-x+y-z)/2+a)^3+((-x-y+z)/2+a)^3=3xyz+4a^3+3a(x^2+y^2+z^2).
0696132人目の素数さん
垢版 |
2023/07/27(木) 01:27:03.74ID:sj1moRvk
>>694-695
なるほど!!!
めっちゃ上手い変換だ
この変換座標で見ればxyz=kという4つの象限に浮かぶ双曲面が球面を4つの部分をカットして4円を作るのが直感的にわかりますね
こういう上手い変換は5次元以上でも見つけることができるんでしょうか…?
0697132人目の素数さん
垢版 |
2023/07/27(木) 09:24:31.64ID:u/F7fa58
四面体が立方格子にハマるという特殊な事情を利用してるから一般化は無理か…
0698132人目の素数さん
垢版 |
2023/07/27(木) 11:36:46.99ID:GVbMLvUT
上に書いたように重根の出方によっては
n!/6でなくてn!/4個の閉曲線になるから
たとえ上手く次元下げられたとしても
そこからも結構面倒くさいかと
0699132人目の素数さん
垢版 |
2023/07/28(金) 08:57:40.98ID:CHq1yppE
>>697
あれ、これはあんまり関係ないか
しかしこれだけの項が消えるのは不思議だ
この次元ですでに線形変換の自由度を越えてるから何かが起きてるはずで、もしかしたら5次元以上でも何か「良い係数」みたいなのが存在してたら面白そう
0700132人目の素数さん
垢版 |
2023/07/28(金) 09:02:05.91ID:CHq1yppE
>>693
>>698
たしかに5次以上だと一般的な形が2パターンありますね
4次元が見れたらこの20個の円と30個の円の違いが図形的に理解できるのに見れないのが残念です
0701132人目の素数さん
垢版 |
2023/07/28(金) 13:24:45.45ID:h2+uipFz
図形ったって閉曲線なんだから適当に射影してもある程度の把握はできると思うけど上手い射影探すのは面倒くさそう
0702132人目の素数さん
垢版 |
2023/07/30(日) 15:33:58.89ID:Vz7ZDLXE
(exp(-x)-1)/(exp(x)-1)のx→0はどうなりますか
0703132人目の素数さん
垢版 |
2023/07/30(日) 15:56:08.34ID:cn4mZKrK
ロピタル
0704132人目の素数さん
垢版 |
2023/07/30(日) 20:27:42.95ID:LNgQZH9a
>>703
これにそれ使うか
まあいいけど
0706132人目の素数さん
垢版 |
2023/07/30(日) 20:51:25.85ID:scqfmG/g
まあロピタルの証明やってるも同然だけど
~ = ( -x + o(|x|) ) / ( x + o(|x|) )
= ( -1 + o(|x|)/x ) / ( 1 + o(|x|)/x )
→ -1 / 1
0708132人目の素数さん
垢版 |
2023/07/30(日) 21:29:52.80ID:dUtyWeoH
E=([0,1]×{-1,0,1})∪({0}×[-1,1])と
F=([0,1]×{0,1})∪({0}×[-1,1])
が同相であることを示したいんですけど、同相写像はどう定義すれば良いのでしょうか?
f((a,b))=(b+1,a-1)のような写像は連続になりませんよね?
0709132人目の素数さん
垢版 |
2023/07/30(日) 21:42:13.65ID:LNgQZH9a
>>708
同相と分かるんなら自分の分かっているそのことをそのまま写像にするだけ
0710132人目の素数さん
垢版 |
2023/07/30(日) 22:08:25.06ID:dUtyWeoH
>>709
それが式で表せないんですよね……
イメージ的にはEの下の線を引っ込めれば良いんでしょうけど数式でどう表せばいいのかが思い付かないんです
0711132人目の素数さん
垢版 |
2023/07/30(日) 22:27:23.69ID:LNgQZH9a
>>710
じゃまガンバってね
0712132人目の素数さん
垢版 |
2023/07/30(日) 23:10:52.56ID:cn4mZKrK
LとIの同相写像を考えればいいだけ
0713132人目の素数さん
垢版 |
2023/07/31(月) 00:36:04.75ID:ar0/LG6u
横からなんだけどLじゃなくてトみたいなのを|にしないといけないんじゃないの?
これどうやって同相写像作るんだろ
0714132人目の素数さん
垢版 |
2023/07/31(月) 00:41:57.01ID:xNJrS+2C
>>713
は?トとIは同相じゃないが
0715132人目の素数さん
垢版 |
2023/07/31(月) 00:47:39.26ID:zgNy3QwU
(0, 1]と(0, 2]の同相写像は作り方わかるの?
0716132人目の素数さん
垢版 |
2023/07/31(月) 02:25:09.64ID:XGbwI/r9
>>714
ごめん問題読み間違えてた
これは同相じゃないのか
一点を除いたときの連結成分が違うから、とかでいいのかな
0717132人目の素数さん
垢版 |
2023/07/31(月) 06:51:11.55ID:jznoxopE
それでよい
ポアンカレの本でも
そんな書き方がしてあった
0718132人目の素数さん
垢版 |
2023/07/31(月) 22:24:16.51ID:TFK5uLM3
rが一定の自然数に固定された定数のとき

lim_{n→∞}(P(n,r)/n^r ) は1ですか。
0719132人目の素数さん
垢版 |
2023/07/31(月) 23:00:15.05ID:t2WIcfXm
905:ウィズコロナの名無しさん:[sage]:2023/07/31(月) 22:28:55.07 ID:Zb/rsFU20
123456789+912345678+891234567+789123456+678912345を9で割ったあまりは?

開成中の問題 勿論力技で解いたら時間切れになる。
0720132人目の素数さん
垢版 |
2023/07/31(月) 23:14:39.77ID:xNJrS+2C
>>718
そりゃそうだ
0721132人目の素数さん
垢版 |
2023/08/01(火) 00:43:55.61ID:aVK9/R57
>>720
やはりそうか。ありがと。
0723132人目の素数さん
垢版 |
2023/08/02(水) 16:09:15.01ID:D4EN5//2
>>719

1, 2, 3, 4, 5, 6, 7, 8, 9を横一列に並べて作った9桁の任意の数はすべて9で割り切れますが、
9!個あるそのような数からその問題に出てくる5つの数を選んだ正当な理由はあるのでしょうか?
0724132人目の素数さん
垢版 |
2023/08/02(水) 21:22:42.96ID:QR2024Zy
>>723
あるわけないだ
0727132人目の素数さん
垢版 |
2023/08/03(木) 00:46:22.57ID:sOIptox1
あるわけねっす
0729132人目の素数さん
垢版 |
2023/08/03(木) 11:22:55.84ID:JEJskKzq
フルラニ積分(Frullani's Integral)について教えてください.
普通はf(x)の可微分性を仮定して証明するようなのですが
↓この簡略証明では、それを使わずに済んでいるように見えます.
https://imgur.com/fA9yhFt
これだと何か問題があるのでしょうか?
0731132人目の素数さん
垢版 |
2023/08/03(木) 14:18:54.66ID:JEJskKzq
>>729 (自己レス)
このままで問題無いような気がする
可微分性は より教育的でエレガントな証明(二重積分&フビニの定理)のための御膳立てにすぎないのではないか
0733132人目の素数さん
垢版 |
2023/08/03(木) 16:22:08.93ID:rIuzytb2
積分(f(ax)-f(bx))/xを2つに分けてる時点であかんやん
両方が絶対収束しない限り分けられるわけがない
0734132人目の素数さん
垢版 |
2023/08/04(金) 08:17:42.90ID:+94nWhQs
変なこと聞くけどフェルマーの最終定理の証明だろうが何だろうが無限の時間さえあればどんなバカでも絶対に最後には全部理解出来ちゃうような数学って学問って学術的に難しい部類に入ると思う?

数学を研究するのがクソ難しいのは大前提として
0737132人目の素数さん
垢版 |
2023/08/04(金) 15:07:26.87ID:2kvHxSR0
無限に時間かければどんな学問でも理解できるだろう。
0738132人目の素数さん
垢版 |
2023/08/04(金) 17:27:34.24ID:uJI/i873
んな訳ないだろ
一般人は無限の時間をかけてもリーマン予想は解決出来ない
0740132人目の素数さん
垢版 |
2023/08/04(金) 21:09:16.18ID:aMAbWLfx
>>737
数学と関係ないので困りますよ
0741132人目の素数さん
垢版 |
2023/08/06(日) 19:46:04.11ID:wwoH/Gr9
i.imgur.com/YbrTiGT.jpg

↑は、

A✕(B✕C) = (C・A)B - (A・B)C

というベクトル3重積に関する公式のWikipediaにある証明の一つですが、おかしくないですか?
0743132人目の素数さん
垢版 |
2023/08/06(日) 22:35:17.52ID:wwoH/Gr9
>>742
「a, b, cそれぞれに対して線形なので、λは定数でなければいけない。」が明らかにおかしいです。
0744132人目の素数さん
垢版 |
2023/08/06(日) 22:37:32.37ID:wwoH/Gr9
aを伸縮させても、bを伸縮させても、cを伸縮させても、λは変わらないということが言えるだけです。
0745132人目の素数さん
垢版 |
2023/08/07(月) 02:10:23.43ID:GJsDYCAD
>>744
確かに以下の文献でも厳密には証明になっていないと書かれている。
http://202.243.124.27/~shige/math/lecture/misc/data/exterior1.pdf
0747132人目の素数さん
垢版 |
2023/08/07(月) 08:28:29.45ID:lOniiJpZ
>>745
ありがとうございます。
戸田盛和著『力学』にWikipediaに書いてある

>>741

の証明と同じ証明が書いてありました。
0748132人目の素数さん
垢版 |
2023/08/07(月) 08:43:33.72ID:objDaz3o
>>745
その文献の著者の頭が悪いだけ。

>>744
λは変わらない
ということは任意のa, b, cに対して
関数λ(a, b, c)=一定値。すなわちλは定数関数。
→特別な場合にλ=1となる
→すべてのa, b, c、に対してλ=1

で全く問題は無い。 すなわちその証明は厳密に正しい。

馬鹿(文献著者)+馬鹿(744)→馬鹿
0750132人目の素数さん
垢版 |
2023/08/07(月) 08:59:33.32ID:INayLHqp
>>749

>>馬鹿がまた書き込んでいたのか

こういう馬鹿の顔が見てみたい
0752132人目の素数さん
垢版 |
2023/08/07(月) 09:15:11.51ID:INayLHqp
>>751

>>馬鹿がまた黒歴史を重ねた笑

こんなバカの顔が見てみたい
0753132人目の素数さん
垢版 |
2023/08/07(月) 09:26:22.26ID:kKChvn1y
馬鹿は「馬鹿でいる時間」が長いのでそこから抜け出すのは難しいのだろう。子供の頃から今に至るまで馬鹿であった歴史の積み重ねは思考や直感に非常な悪影響を及ぼし「勉強をしても身につかない」という状態が継続する。

特にこういう所で質問する馬鹿は治らない真正馬鹿の可能性が高い。
0754132人目の素数さん
垢版 |
2023/08/07(月) 11:05:32.51ID:Ot4gLt34
V,Wが線形空間、f,g:V→Wが0でない線形写像、λ:V→K(Kは係数体)が

g = λf

を満たすときλは定数

はもちろん結論としては正しい
が、そのレベルの文書で自明として良いか
パッとあったり前の証明は思いつかんね
0755132人目の素数さん
垢版 |
2023/08/07(月) 11:17:00.04ID:Lj3uUrqP
>>753
おまえのようなバカの顔が見てみたい
0756132人目の素数さん
垢版 |
2023/08/07(月) 16:11:54.28ID:sAhlnq9p
>>743,744
線形性は伸縮だけでなくて分配もよ
だから基底に関して成立していれば
どんな場合にも成立するということに
もっと具体的に言うと
V=R^3として
f:V^3→V:(A,B,C)→A✕(B✕C)-{(C・A)B - (A・B)C}
と定義したら多重線形写像だから
g:V^¥otimes3→V
の線形写像に「拡張」できる(V^¥otimes3=R^27)
基底すべてがker(g)に入るんだから0写像つまり常に0
あるいは
f:V^3→V ⇔ f':V^2→F(V,V):(A,B)→(C→A✕(B✕C)-{(C・A)B - (A・B)C})
でf'(A,B)はCについて線形だから
f':V^2→Hom(V,V)

f':V^2→Hom(V,V) ⇔ f'':V→F(V,Hom(V,V)):A→(B→(C→A✕(B✕C)-{(C・A)B - (A・B)C})))
でf''(A)はBについて線形だから
f'':V→Hom(V,Hom(V,V))

f’’はAについて線形でと順に考えていっても良いけど
0757132人目の素数さん
垢版 |
2023/08/07(月) 16:22:25.10ID:sAhlnq9p
昔はベクトルの割り算というのがあったらしい
50年〜100年ぐらい昔の教科書に定義されてたのを見たときある
見ただけでどんなものか覚えてないけれど
0758132人目の素数さん
垢版 |
2023/08/07(月) 20:03:27.72ID:sAhlnq9p
>>748
>λは変わらない
というのが自明に思えないって書いてる
>上の Step 4. では
>「k2 が A, B, C には無関係である」
>ということを用いて、特殊な A, B, C に対して k2 を決定したが、実はその無関係性 は自明ではない (少なくとも私には容易に示すことはできない)。
0759132人目の素数さん
垢版 |
2023/08/07(月) 20:13:33.70ID:7ziMA96l
>>758
馬鹿(文献の著者及びお前)に自明に思えないことはいいこと。面倒なプロセスを経て納得すればよい。

俺には自明だ。
0760132人目の素数さん
垢版 |
2023/08/07(月) 21:04:05.99ID:ZKmVtEF9
数列空間l^pってなんか役に立つんですか?
0763132人目の素数さん
垢版 |
2023/08/07(月) 21:18:49.28ID:rtuZMILL
なんか変な奴暴れてるんだが
0764132人目の素数さん
垢版 |
2023/08/07(月) 21:20:21.56ID:7ziMA96l
>>761
アスペ(お前)に数学を勉強しづらくさせておくことは文化を維持していく上で有用。

教科書を書く人は今までと同様に「明らか」「証明・解答は省略する」を適宜使って教科書を書いて欲しい。乗り越えた一部のアスペ以外のアスペは排除してよし。
0766132人目の素数さん
垢版 |
2023/08/07(月) 21:35:21.26ID:rtuZMILL
こいついなくなるまで解散
0770132人目の素数さん
垢版 |
2023/08/08(火) 04:39:12.34ID:4Vf/OcYE
>>759
自分も>>754と同意見
0771132人目の素数さん
垢版 |
2023/08/08(火) 05:05:24.45ID:4Vf/OcYE
ア違った
>>759,754
どちらも間違いか
f:R^2→R:(x,y)→x
λ:R^2→R:(x,y)→1 for x≠0, 2 for x=0
g(x,y)=λ(x,y)f(x,y)
も線形
すなわちf,g:線形g=λfとしても必ずしもλ:定数とは限らない
しかし
f,g:線形g=λfのときあるc:定数が存在してg=cf
は成立しそう
>>759,754
の意図もそれだと思うが
>>754
の言うようにどう証明するかな
0772132人目の素数さん
垢版 |
2023/08/08(火) 05:38:30.60ID:4Vf/OcYE
g(x)=λ(x)f(x)
g(kx)=λ(kx)f(kx)
kg(x)=kλ(kx)f(x)
kλ(x)f(x)=kλ(kx)f(x)
k(λ(x)-λ(kx))f(x)=0
kf(x)≠0→λ(kx)=λ(x)
kf(x)≠0⇔k≠0,f(x)≠0
0倍以外のkでker(f)以外のxについて同一方向は一定は簡単か
g(x+y)=λ(x+y)f(x+y)=λ(x+y)(f(x)+f(y))
g(x)+g(y)=λ(x)f(x)+λ(y)f(y)
(λ(x+y)-λ(x))f(x)+(λ(x+y)-λ(y))f(y)=0
f(x),f(y):1次独立→λ(x+y)=λ(x)=λ(y)
ううむ手詰まり感
これはどうかな
f(x)≠0,y∈ker(f)→λ(x+y)=λ(x)
ker(f)以外のxについてker(f)方向(x+ker(f))は一定と
これと上の2つと組み合わせると
ker(f)以外のxでλ:一定が出そうだけど
それが出たらker(f)でもλはその値cにして構わないからg=cfと
0773132人目の素数さん
垢版 |
2023/08/08(火) 06:02:36.75ID:EMDOU0xx
数列空間l^pってなんか役に立つんですか?
0774132人目の素数さん
垢版 |
2023/08/08(火) 06:17:32.25ID:4Vf/OcYE
f:V→Wをp:V→V/ker(f)とf':V/ker(f)→W:単射に分けて
g=λf=λf'pもp:V→V/ker(f)とg'=λf':V/ker(f)→Wに分けて
というわけには行かないか
λ:V→Kがλ':V/ker(f)→Wによってλ=λ'pと表せるのか示さないとg'=λ'f'と出来ないね
あでもそれが
f(x)≠0, y∈ker(f)→λ(x+y)=λ(x)
で言えるのか
いや不十分かf(x)=0の場合も考慮しないとλ'の存在が言えない
でも
f(x)=0, y∈ker(f)のときはf(x+y)=0だからg(x+y)=0であって
λの値は何でも良いから
まず
λに対してλ'’:V→Kを
λ''(x)=λ(x) for f(x)≠0, 0 for f(x)=0
と定義すると
g=λf=λ''f
かつλ'':V→Kは
f(x)≠0, y∈ker(f)→λ''(x+y)=λ(x+y)=λ(x)=λ''(x)
(∵ f(x+y)=f(x)+f(y)=f(x)≠0なので)
f(x)=0, y∈ker(f)→λ''(x+y)=0=λ''(x)
(∵ f(x+y)=f(x)+f(y)=0なので)
であることから
λ':V/ker(f)→K
が存在して
λ’’=λ'p
と分解できる
よってこのλ'によって
g'=λ'f'
と表せると
するとf'の単射性から
x',y':1次独立→λ'(x'+y')=λ'(x')=λ'(y')
となり
x'≠0でλ':一定
が出ると
そこでその値をcとすると
x'≠0→g'(x')=λ'(x')f'(x')=cf'(x')
x'=0→g'(0)=0=cf'(0)
より
目出度くg'=cf'が示せたと
よって
g=g'p=cf'p=cf
でOK
0775132人目の素数さん
垢版 |
2023/08/08(火) 06:28:58.88ID:4Vf/OcYE
>>774
>よってこのλ'によって
>g'=λ'f'
>と表せると
ここは
g=λf=λ''f=(λ'f')p
となるのでg'=λ'f'と定義してやれば
g=g'p
と表せる
に訂正
0776132人目の素数さん
垢版 |
2023/08/08(火) 06:42:00.70ID:4Vf/OcYE
>>774
>そこでその値をcとすると
V=ker(f)のときはx'≠0であるx'∈V/ker(f)は存在しないから
当然ながらcを選べないが
この場合はf=0すなわちg=0なのでcは何でも良い
0777132人目の素数さん
垢版 |
2023/08/08(火) 06:43:17.74ID:4Vf/OcYE
>>774
>するとf'の単射性から
>x',y':1次独立→λ'(x'+y')=λ'(x')=λ'(y')
x',y':1次独立→f'(x'),f'(y'):1次独立→λ'(x'+y')=λ'(x')=λ'(y')
0778132人目の素数さん
垢版 |
2023/08/08(火) 06:46:38.53ID:4Vf/OcYE
>>766
嵐は居なくならないよ
嵐が飽きるまでは
そこまで解散し続けるわけにも行かないから
折り合いをつける他無いと思うけどね
0779132人目の素数さん
垢版 |
2023/08/08(火) 07:53:49.21ID:xil/2MkA
>>771
おい馬鹿。俺は間違っていない。

「馬鹿が自作した問題」とは別の問題だ。よく見ろよ馬鹿。
0780132人目の素数さん
垢版 |
2023/08/08(火) 08:10:02.31ID:4Vf/OcYE
>>779
君のはどれ?
0781132人目の素数さん
垢版 |
2023/08/08(火) 08:26:51.49ID:ffQx2IlH
荒らしにかまわんで
0783132人目の素数さん
垢版 |
2023/08/08(火) 08:32:04.22ID:q3EYdwlh
V,Wが線形空間、f,g:V→Wが0でない線形写像、λ:V→K(Kは係数体)が

g = λf

を満たすとき定数λ'を

g = 'λf

と取り直せる

まぁこの程度は許してよ、と言いたいけどこのレベルの話してるわけやからな、あかんか
0785132人目の素数さん
垢版 |
2023/08/08(火) 08:45:11.20ID:B7V01oJr
V,Wが線形空間、f,g:V→Wが0でない線形写像、λ:V→K(Kは係数体)が
g = λf
を満たすときλは定数
はもちろん結論としては正しい


この命題正しいのか?笑
0786132人目の素数さん
垢版 |
2023/08/08(火) 08:52:38.30ID:4Vf/OcYE
>>783
まあ厳密性を云々しているわけだから
細かいところ指摘させていただきました
けんど>>771に書いたように意図は分かりますよ
0787132人目の素数さん
垢版 |
2023/08/08(火) 09:03:30.68ID:gYApmRdW
イヤ、大丈夫、別に気分を害してる訳でもないので
お気になさらず
0788132人目の素数さん
垢版 |
2023/08/08(火) 09:14:59.03ID:YONd1ImA
なんだこいつら

修正命題(存在定理)も偽なのだが
馬鹿同士で真として納得しあっている笑
0791132人目の素数さん
垢版 |
2023/08/08(火) 09:49:46.99ID:B/NLfL7v
このダラダラした間違った証明は
「馬鹿はいくら勉強しても数学ができるようにならない」ことの実例になる。

あとは「厳密」と称して些末なことにこだわることだけを続けていると問題の「意味」が分からずに
形式的な議論に乗せたがる「数学っぽいが数学ではないもの」をやることにになる。
0792132人目の素数さん
垢版 |
2023/08/08(火) 10:04:57.34ID:Iiw1KUn4
>>791
こんなバカの顔が見てみたい
0794132人目の素数さん
垢版 |
2023/08/08(火) 11:08:53.05ID:Iiw1KUn4
訂正行為は馬鹿でない証拠
0795132人目の素数さん
垢版 |
2023/08/08(火) 11:29:21.36ID:FJe2wShC
松坂くん未満の数学力なのに自分のことを頭がいいと思い込んでるんだから救いようがねえなこの馬鹿
0796132人目の素数さん
垢版 |
2023/08/08(火) 12:01:07.26ID:z8jlYGPe
ID:Iiw1KUn4の他のレスをみたけど第一次反抗期を抜け出してない幼児性丸出しの人だね
岡潔の「数学は数え三つのところで考え数え四つのところで書け」を実践している
0802132人目の素数さん
垢版 |
2023/08/08(火) 14:37:35.60ID:4Vf/OcYE
>>796
>ID:Iiw1KUn4の他のレスをみたけど
自分の予想ではID:Iiw1KUn4とその「他のレス」は別人じゃないかな
0805132人目の素数さん
垢版 |
2023/08/08(火) 16:54:02.98ID:oCNdaIq1
線形性と交代性があれば行列式の定数倍になるって、線形代数の常識じゃないの?
0808132人目の素数さん
垢版 |
2023/08/08(火) 17:10:28.76ID:DpLMz9gd
どのくらい勉強すれば

   線形性と交代性があれば行列式の定数倍になる

ということが分かるのでしょうか?
0812132人目の素数さん
垢版 |
2023/08/08(火) 19:14:30.94ID:74vMO90X
>>808
本によっては行列式の定義のところに符号付き体積とかの話と一緒に書いてある
0814132人目の素数さん
垢版 |
2023/08/08(火) 20:07:13.37ID:CpLcy35y
どのくらい勉強すれば

  線形性と交代性があれば行列式の定数倍になる

    線形性と交代性があれば行列式の定数倍になる

      線形性と交代性があれば行列式の定数倍になる


ということが分かるのでしょうか?


という質問の、どの点が最低なのでしょうか?
0817132人目の素数さん
垢版 |
2023/08/08(火) 21:23:57.67ID:4Vf/OcYE
>>814
答えないからねー
0819132人目の素数さん
垢版 |
2023/08/08(火) 22:07:58.40ID:yR9Hny9I
>>817
この問題には答えがある。

ただし初学者(特にこのスレで質問するような馬鹿)は答えのない「大きな質問」をすべきではなく(馬鹿なんだから)、
思いついちゃった自作問題なんかを問うべきではなく(馬鹿なんだから)、
答えの定まる「小さな質問」のみするべき(馬鹿なんだから)。
0820132人目の素数さん
垢版 |
2023/08/08(火) 22:12:49.77ID:yR9Hny9I
要するにこんな所で質問する馬鹿は先を見ないで「今日の1ページだけ」を真剣に勉強しろということ。
○○は何の役に立ちますかとか最低の質問。意味無し。
0822132人目の素数さん
垢版 |
2023/08/08(火) 22:46:39.71ID:yR9Hny9I
「この記述おかしいですよね)とか
「この著者は本当に理解しているのでしょうか」
などの質問もするな。自分に返ってくるだけ。他人に同意を求めるな。
0823132人目の素数さん
垢版 |
2023/08/08(火) 22:51:36.66ID:4Vf/OcYE
>>814
答えないのを求めても仕方ないよ
0825132人目の素数さん
垢版 |
2023/08/08(火) 23:23:57.28ID:G6B1oOIJ
答えがある問題の結論に至る論証が雑すぎんだろという話
物理学だけどランダウとか読んでても感じる事が多い
こういう理屈で必然的にこういう式になる!みたいな記述、云うほど自明じゃないしもう少し丁寧に説明してくれってなる
0826132人目の素数さん
垢版 |
2023/08/09(水) 00:02:40.13ID:DFD9/WEX
>>825
詳しくは知らないがその本はそういう本として有名なんだから仕方ないんじゃないか笑
「馬鹿を寄せ付けない本」として、読むのをやめれぱいいだけ。

これか

統計物理学
有名なランダウの教程の一冊。熱力学,統計力学ともに基礎的な部分の記述には感心しない。(田崎晴明)
0829132人目の素数さん
垢版 |
2023/08/09(水) 03:18:30.48ID:ekLYtxZ5
>>826
頭に浮かんでたのは「力学」「場の古典論」の方ですがまあ記述スタイルは似たようなもんだと思います

それにしてもランダウ本をよく知らんのにどうして田崎「統計力学 Ⅱ」巻末の参考文献書評に行き当たるのか...
こんなのググっても見つからんでしょうに
0830132人目の素数さん
垢版 |
2023/08/09(水) 06:50:40.08ID:/x3euq4L
>>829

>>どうして田崎「統計力学 U」巻末の参考文献書評に行き当たるのか

こういうところが一番よく読まれるのではないか
0831132人目の素数さん
垢版 |
2023/08/09(水) 07:29:50.18ID:oY2Lttk2
開区間(a, b)上のC^1級関数fに対して、
片側極限f(a+0), f(b-0), f’(a+0), f’(b-0)が存在する場合、
fを閉区間[a, b]上の連続関数に拡張したものは、
aにおいて右側微分可能、bにおいて左側微分可能ですか?
0832132人目の素数さん
垢版 |
2023/08/09(水) 07:43:46.32ID:4kX5/JPp
>>829
熱力学の概念が必要になって物理の人間に聞いたら田崎熱力学、統計力学1, 2を勧められた。
0833132人目の素数さん
垢版 |
2023/08/09(水) 07:56:08.66ID:3gn/MBdZ
厚さ0.1μmの金メッキがされた半径1mの球がある。
この球の表面から無作為(即ち、どの点が選ばれる確率も同じ)に3点を選び球面三角形のメッキを剥がして金を売る。
金の価格は1g9917円、金の比重は19.32とする。
(1)売値の期待値を求めよ
(2)売値の中央値を求めよ
(3)メッキを剥がす費用として50000円請求されるとき、金を売って利益がでる確率を求めよ。
(4)売値の分布を図示せよ。
0835132人目の素数さん
垢版 |
2023/08/09(水) 08:36:11.69ID:/x3euq4L
田崎はポスドク時代に
Elliot Liebの薫陶を受けている
0836132人目の素数さん
垢版 |
2023/08/09(水) 08:39:09.61ID:0puxqnT0
>>831
平均値の定理で解決しました
0837132人目の素数さん
垢版 |
2023/08/09(水) 08:39:19.43ID:UzCvYTp/
>>831
可能
0838132人目の素数さん
垢版 |
2023/08/09(水) 11:45:26.47ID:jmT3hvMa
f(x)を最高次の係数が1の実数係数多項式とする。
lim[n→∞] ∫[0,1] f(x)|sin(nx)| dx
を求めよ。
0839132人目の素数さん
垢版 |
2023/08/09(水) 11:58:55.37ID:jmT3hvMa
Aはn×nの正方行列で、1×nのベクトルX[n]に対して、X[k+1]=AX[k]によりX[k](k=0,1,2,...)を定める。
ある零ベクトルではないX[0]について、3以上の自然数mではじめてX[m]=X[0]がなりたつとき、そのような単位行列でないAが存在するならば、AとX[0]の組を1つ求めよ。
そのようなAが存在しないならばそのことを証明せよ。
0840132人目の素数さん
垢版 |
2023/08/09(水) 12:06:28.34ID:rzyZffkv
とりあえずn=1が必要、以下略
0841132人目の素数さん
垢版 |
2023/08/09(水) 13:35:27.81ID:jUMDxtFN
>>837
答え出てからコメントする奴w
0842132人目の素数さん
垢版 |
2023/08/09(水) 22:11:10.16ID:UzCvYTp/
>>841
まあ書き込むときには>>836見てなかったけど
平均値の定理とか使う必要ないと思うけどね
0843132人目の素数さん
垢版 |
2023/08/09(水) 22:11:44.44ID:bxv4pJ3V
1〜2nの自然数から異なるn個を選んび、それを小さい方からならべて
a[1],a[2],…,a[n] と並べるとき、
すべてのkについてa[k]≧2kが成り立つような
選び方は カタラン数になるのでしょうかなぜですか。
0845132人目の素数さん
垢版 |
2023/08/10(木) 17:12:45.42ID:ENV1Mv9q
数列空間l^pってなんか役に立つんですか?
0846132人目の素数さん
垢版 |
2023/08/10(木) 18:13:54.27ID:AhkxEmcv
はい
0847132人目の素数さん
垢版 |
2023/08/12(土) 07:07:28.66ID:gfpNA3mg
C^1級関数のフーリエ級数が一様収束することってどうやって示すのが簡単ですか?
0849132人目の素数さん
垢版 |
2023/08/14(月) 18:56:20.17ID:D/diOVSY
>>836
どう使うのかな?
0850132人目の素数さん
垢版 |
2023/08/15(火) 10:10:53.70ID:s8YLDwLC
>>849
f’(a+0)が存在するという条件から、
xがaに近いところではf’(x)はf’(a+0)に近い
平均値の定理より、
xに対してあるcが存在して(f(x)-f(a))/(x-a)=f’(c)
よってfはaで右側微分可能
0851132人目の素数さん
垢版 |
2023/08/15(火) 17:36:07.64ID:v8SWOz+7
>>850
>f’(a+0)が存在するという条件から、
>xがaに近いところではf’(x)はf’(a+0)に近い
NG
0852132人目の素数さん
垢版 |
2023/08/15(火) 18:28:19.66ID:x0I2PNUL
>>851
記号の意味を誤解してるかもしれないが
f’(a+0)はf’(x)のx→aの右極限だからな
フーリエ級数でよくある記法
0853132人目の素数さん
垢版 |
2023/08/15(火) 20:23:22.73ID:0tg+1WzS
fを境界に連続拡張しただけであってf'を連続拡張したわけじゃないから、その片側連続性は示さないといけないんじゃね
0854132人目の素数さん
垢版 |
2023/08/16(水) 12:02:59.21ID:AoxmOkF9
平面上の点 P を考える。
座標系 O-x, y とそれを原点を中心として θ だけ回転させた座標系 O-x', y' を考える。
座標系 O-x, y での点 P の座標を (x, y)
座標系 O-x', y' での点 P の座標を (x', y')
とする。

このとき、

x = x' * cos θ - y' * sin θ
y = x' * sin θ + y' * cos θ

が成り立つ。

このことを以下のように説明している本を見かけませんがなぜでしょうか?

x 軸と線分 OP のなす角を φ とする。
このとき、 x' 軸と線分 OP のなす角は θ + φ である。
点 Q を x' 軸と線分 OQ のなす角が φ であるような点とする。
座標系 O-x', y' での点 Q の座標は (x, y) である。
点 P は点 Q を原点を中心として θ だけ回転させた位置にある。

よって、

x = x' * cos θ - y' * sin θ
y = x' * sin θ + y' * cos θ

が成り立つ。
0855132人目の素数さん
垢版 |
2023/08/16(水) 12:41:28.22ID:sR29mFXa
そうですか
0856132人目の素数さん
垢版 |
2023/08/18(金) 22:20:14.34ID:20jBxerI
同値関係で
対称率より 「a〜bならばb~a」で、推移律より「a〜b,b~aならa〜a」だから
a〜aが言えてしまうので、反射率はふようということになるというのですが
どういうことなんでしょう
0858132人目の素数さん
垢版 |
2023/08/18(金) 23:11:34.22ID:20jBxerI
a自信
0859132人目の素数さん
垢版 |
2023/08/18(金) 23:30:36.64ID:UIpPF+tb
>>856
対称率と反射率の意味が分からない。
もしかして対称律、反射律の間違いかな?それとも何か特別な用語として定義されたものなのか?分からない。

論法も用語も滅茶苦茶で意味不明な書き込みですよね笑
まあ馬鹿だということは分かるけど。
0860132人目の素数さん
垢版 |
2023/08/18(金) 23:42:03.16ID:y5IKElDb
>>856
馬鹿が質問してはいけない
ということは言いません。このスレで質問するのは馬鹿ばっかりなので慣れています。馬鹿は成長しないで馬鹿のままだなあ、いやむしろ馬鹿は馬鹿をこじらせてより馬鹿になる道を選ぶのだなあと思います。

馬鹿は昨日より今日、今日より明日、より馬鹿になっていきます。
このスレから馬鹿な質問者が消えることは決してありません。
0861132人目の素数さん
垢版 |
2023/08/18(金) 23:56:26.40ID:8ofuJICt
>>856
どこの大学に行くとこんな馬鹿になれるのか、日本の大学でこんな馬鹿を養成するような大学が実際にあるのか。

馬鹿の方向に振り切れるのはそれはそれで難しいと思います。
0862132人目の素数さん
垢版 |
2023/08/18(金) 23:59:07.97ID:20jBxerI
質問スレでは
質問とそれに対する回答があればいいのでは


それ以外の戯言はチラシの裏にでも書いていればいいのに
0866132人目の素数さん
垢版 |
2023/08/19(土) 07:53:38.40ID:UtQpjDM8
>>858
それを保証するのが反射律だけど要らない?
0867132人目の素数さん
垢版 |
2023/08/19(土) 08:20:24.99ID:IvSTsmOl
まーた馬鹿馬鹿言う人来たよ
0868132人目の素数さん
垢版 |
2023/08/19(土) 09:21:13.12ID:UtQpjDM8
tan(θ1+θ2+θ3+θ4+…)=(σ1-σ3+σ5-σ7+…)/(σ0-σ2+σ4-σ6+…)
σn=nth symmetricsum of tanθ1, tanθ2, tanθ3, tanθ4, …
σ0=1
0870132人目の素数さん
垢版 |
2023/08/19(土) 10:42:00.61ID:UtQpjDM8
sin(θ1+θ2+θ3+θ4+…)=(σ1-σ3+σ5-σ7+…)cosθ1cosθ2cosθ3cosθ4…
cos(θ1+θ2+θ3+θ4+…)=(σ0-σ2+σ4-σ6+…))cosθ1cosθ2cosθ3cosθ4…
0871132人目の素数さん
垢版 |
2023/08/19(土) 10:52:54.89ID:UtQpjDM8
cos(θ1+θ2+θ3+θ4+…)=(σ0-σ2+σ4-σ6+…))cosθ1cosθ2cosθ3cosθ4…
=ω0-ω2+ω4-ω6+…
sin(θ1+θ2+θ3+θ4+…)=(σ1-σ3+σ5-σ7+…)cosθ1cosθ2cosθ3cosθ4…
=ω1-ω3+ω5-ω7+…
ωn=sum of product of n sinθi and other cosθj
0872132人目の素数さん
垢版 |
2023/08/19(土) 12:04:26.42ID:QuB8wVGN
キャンペーンは20日までです、出来ればお早めに
https://i.imgur.com/rgCYY2c.jpg
0873132人目の素数さん
垢版 |
2023/08/19(土) 14:10:53.71ID:sCXSzqyi
>>872
これからやってみる
0874132人目の素数さん
垢版 |
2023/08/19(土) 14:58:14.26ID:LznRjSJp
Jordan標準形は、「表現行列」としてどんな点で一番優れているのでしょうか?
0875132人目の素数さん
垢版 |
2023/08/19(土) 15:29:07.21ID:LznRjSJp
定量的に優れている点、定性的に優れている点を挙げてください。
0876132人目の素数さん
垢版 |
2023/08/19(土) 15:45:26.58ID:W6mDkR8T
小星型十二面体のオイラー数を計算すると-6になるので種数4の閉曲面と同相らしいのですが
図を見ても球面と同相にしか見えません

正十二面体の各面に五角錐を貼り付けただけなのでその錐を正十二面体の方につぶしてやれば
正十二面体に同相になるように思えるのですが
この議論はどこが間違っているのでしょうか
0877132人目の素数さん
垢版 |
2023/08/19(土) 15:56:07.04ID:oUKnUxtT
つまり

小星型十二面体のオイラー数を計算すると-6になるので種数4の閉曲面と同相らしいのですが

こっちがおかしい
0878132人目の素数さん
垢版 |
2023/08/19(土) 16:04:03.61ID:Wb32gLRo
頂点32
辺90
面60
オイラー数は2
0879132人目の素数さん
垢版 |
2023/08/19(土) 16:28:02.48ID:W6mDkR8T
確かに!
ちゃんと自分で数えるべきでした
お二人ともありがとうございます
0880132人目の素数さん
垢版 |
2023/08/19(土) 19:29:11.80ID:LznRjSJp
A を正方行列とします。
P を直交行列とします。
P^{-1} * A * P をできるだけ簡単な形にしたいとします。
どうすればいいのでしょうか?
0881132人目の素数さん
垢版 |
2023/08/19(土) 19:29:50.64ID:LznRjSJp
A を正方行列とします。
P を直交行列とします。
P^{-1} * A * P ができるだけ簡単な形になるような P を求めたいとします。
どうすればいいのでしょうか?
0882132人目の素数さん
垢版 |
2023/08/19(土) 19:31:58.68ID:LznRjSJp
A は対称行列ではないとします。
0883132人目の素数さん
垢版 |
2023/08/19(土) 19:41:07.69ID:LznRjSJp
あ、

P^{-1} * A * P = 直交行列 ✕ 対角行列

とできますね。

これが一番簡単な形ですかね。
0884132人目の素数さん
垢版 |
2023/08/19(土) 20:01:25.84ID:bep17sx1
>>883
(1, 2)
(0, 1)
0887132人目の素数さん
垢版 |
2023/08/22(火) 20:35:15.25ID:/LKo6zo3
一階線形微分方程式を解けという問題が試験で出題されたとします。
解がどのようなものかは分かっているので、直ちに解答を書き始めてもいいんですか?

同次方程式の一般解を求めて、定数変化法で非同次方程式の特殊解を求めて、それらを加えたものを解として書くというようなことはしなくてもいいんですよね?
0888132人目の素数さん
垢版 |
2023/08/22(火) 20:48:38.53ID:/LKo6zo3
定数変化法のアイディアは自然で誰でも思いつく方法だと思いますが、積分因子による解法ってどうなんですか?
微分方程式を眺めて、辻褄を合わせようと努力すれば、パズルを解くように思いつくかもしれません。
ですが、そこに何か深い話はありますか?
はっきり言って、いきなり答えを思いつくのと大差ない方法ですよね。
0889132人目の素数さん
垢版 |
2023/08/22(火) 20:50:12.10ID:ZAgXE9v/
>>887
具体的なモノが無いと何ともカとも
0890132人目の素数さん
垢版 |
2023/08/22(火) 20:51:31.21ID:ZAgXE9v/
>>888
具体的なモノが無いと何ともヵとも
0891132人目の素数さん
垢版 |
2023/08/22(火) 20:52:47.99ID:/LKo6zo3
2次方程式を解くのに、解の公式を使わずに、平方完成というアイディアを使って、解く変わった人はいないと思います。
ですが、定数変化法による解法も積分因子による解法も、平方完成というアイディアを使ってわざわざ2次方程式を解くようなものですよね。
0892132人目の素数さん
垢版 |
2023/08/22(火) 20:55:15.33ID:/LKo6zo3
一度導き出した解を公式として記憶してそれを使って解くというのが正統的な方法だと思います。
定数変化法や積分因子による解法は単なる記憶術にすぎないと思います。
0894132人目の素数さん
垢版 |
2023/08/22(火) 21:57:45.96ID:A8r0KcVW
積分因子は確かなるほど微分方程式が何であんなものが出てくるのか導出してたはず
立ち読みでそのページ見て他で見ない計算だったから買った後本の山に埋もれて行方不明になった
0895132人目の素数さん
垢版 |
2023/08/22(火) 22:41:54.13ID:ZAgXE9v/
>>892
2階から公式無いからね〜
0896132人目の素数さん
垢版 |
2023/08/22(火) 22:43:13.37ID:ZAgXE9v/
定数変化法も積分因子法も他の方法も
公式がないときにも応用できるときがあるからね〜
0897132人目の素数さん
垢版 |
2023/08/22(火) 23:08:36.19ID:F9IhIGpy
>>891
平方完成使うよ
使わないとかいう決めつけしてる奴は大抵無能で進歩ないwww

正統的とか俺ルール押し付けんなよ
0898132人目の素数さん
垢版 |
2023/08/22(火) 23:15:26.62ID:ZAgXE9v/
>>893
a=b=c=0ツマンネ
0899132人目の素数さん
垢版 |
2023/08/24(木) 17:35:57.70ID:z7NvEozZ
京大入試の
「tan1°は有理数か?」
って数学的には一言「いいえ」って書けば満点入るはずだけど実際はどうだったんだろう

書いてて思ったがこれ高校数学の発想に呪われてないかどうか見る意図、つまり土壇場で問題文を正しく理解して証明なしで「いいえ」と書くだけで点が貰えると見抜けるかどうか見てるメタ問題だったって認識であってそうだな

証明欲しいなら無理数である事を証明せよってキチンと出題するはずだし
0900132人目の素数さん
垢版 |
2023/08/24(木) 19:20:18.24ID:Igu3El2/
>>899
円周率が無理数であることを利用するんでしょうね。
0901132人目の素数さん
垢版 |
2023/08/24(木) 19:27:05.35ID:Igu3El2/
あ、勘違いでした。
違うはずですね。
√3が無理数であることを使いそうですね。
0902132人目の素数さん
垢版 |
2023/08/24(木) 19:33:43.92ID:GrIw9G1T
以下の命題を考えているのですがこの命題は成り立ちますかね?成り立たない場合反例を挙げていただけると嬉しいです…
https://i.imgur.com/mOrKYKW.jpg
0903132人目の素数さん
垢版 |
2023/08/24(木) 19:41:08.96ID:Igu3El2/
tan(π/180) ∈ Q であると仮定して矛盾を導く。
tan(a) ∈ Q ならば、 tan(2 * a) = 2*tan(a) / (1 - tan(a)^2) ∈ Q である。
仮定により、 tan(π/180) ∈ Q であるから、 tan((2/180)*π) ∈ Q である。
tan((2/180)*π) ∈ Q であるから、 tan((4/180)*π) ∈ Q である。
tan((4/180)*π) ∈ Q であるから、 tan((8/180)*π) ∈ Q である。
tan((8/180)*π) ∈ Q であるから、 tan((16/180)*π) ∈ Q である。
tan((16/180)*π) ∈ Q であるから、 tan((32/180)*π) ∈ Q である。
tan((32/180)*π) ∈ Q であるから、 tan((64/180)*π) ∈ Q である。

tan(a), tan(b) ∈ Q ならば、 tan(a - b) = (tan(a) - tan(b)) / (1 + tan(a) * tan(b)) ∈ Q である。
tan((4/180)*π), tan((64/180)*π) ∈ Q であるから、 √3 = tan((1/3)*π) = tan((64/180)*π - (4/180)*π) ∈ Q である。
これは、 √3 が無理数であるという事実に反する結果である。
よって、 tan(π/180) ∈ R - Q である。
0904132人目の素数さん
垢版 |
2023/08/24(木) 19:51:49.47ID:Igu3El2/
>>899
例えば、

「2023は素数か?」

という問題に対して、「いいえ」とだけ書いて満点になるということでしょうか?
0905132人目の素数さん
垢版 |
2023/08/24(木) 19:53:35.90ID:Igu3El2/
証明が欲しいなら合成数であることを証明せよと出題するはずでしょうか?
0906132人目の素数さん
垢版 |
2023/08/24(木) 20:15:31.77ID:wdwy4Q0G
>>899
0907132人目の素数さん
垢版 |
2023/08/24(木) 20:17:03.24ID:R3axnSqX
>>899
>数学的には一言「いいえ」って書けば満点入るはずだけど
0点
0909132人目の素数さん
垢版 |
2023/08/24(木) 21:57:36.46ID:MG6mcbcs
>>908
「求めたけど得られませんでした」でいい
0911132人目の素数さん
垢版 |
2023/08/25(金) 00:02:27.05ID:qPl1I7cg
知っているが気にくわないので書かない
0912132人目の素数さん
垢版 |
2023/08/25(金) 00:37:53.12ID:kY/nqcWs
パラメータ付けされた群の族の非自明な例が思いつかない

Xを位相空間(空、1点、離散などではない)
G_x(x∈X)がそれぞれ群(全部同じ以外)
xの変化に対してG_xも適当な意味で連続的に変化する

特殊線形群とか直交群とかはちょっとうごかすの無理そうだし
0913132人目の素数さん
垢版 |
2023/08/25(金) 00:42:30.31ID:m7Ej3QVr
S^nに位相群かリー群の構造が入るのってn = 1, 3だけですか?
0914132人目の素数さん
垢版 |
2023/08/25(金) 00:55:07.76ID:wwxn4A+Q
>>912
X = S^1
すべてのxに対してG_x = S^1でも、R^3に

G_x = {(cosθ, sinθcosx, sinx)| 0≦θ≦2π}

みたいに埋め込んだら、非自明なのでは
0916132人目の素数さん
垢版 |
2023/08/25(金) 01:08:59.86ID:qPl1I7cg
>>912
G_xが全部同型でも全体が直積(自明)でなければいいというなら
x∈S^2にxでの接平面をG_xにしたら?G_x=R^2だけど直積じゃないし
変化しないとやだてなら
S^2の中心を通る平面R^2を1つ固定して
G_xをまず中心に平行移動してからR^2に正射影するπ_xの像をH_xにするとかはどうかな
π_xの逆像にしてG_xの中に持っていくと
x∈S^2∩R^2については直線(部分空間)になる感じで
0917132人目の素数さん
垢版 |
2023/08/25(金) 01:14:13.20ID:qPl1I7cg
>>914
cosx=0のときは?
0918132人目の素数さん
垢版 |
2023/08/25(金) 01:36:19.20ID:qPl1I7cg
>>912
>特殊線形群とか直交群とかはちょっとうごかすの無理そうだし
動かすというのがGLの中でって言う意味?
そうでないならさっきのG_xでSL(G_x)とかO(G_x)とかにしたらいいんじゃないの?
GLの中でもOなら共役P^-1OPでPを適当に動かせばいいよ
SLは同型で動かすのは無理だけどCなら
G⊂S^1⊂C^×でdet^-1(G)考えてGを{1}から連続的に変化させるとか?
たとえばx∈S^1に対してG_x=<x>={x^n|n∈Z}とかでxを1からS^1上をぐるっと一周とか?
0919132人目の素数さん
垢版 |
2023/08/25(金) 01:43:24.07ID:qPl1I7cg
あそうかRでもいいや
いいけど連続的に変化させると
行って戻ってしかないから面白くない
Cでも別にS^1上でなくてイイや
x∈C^×を適当に動かしてやれば
でも0をぐるっと回って1に戻るとかでないと
面白く無さそう
0920132人目の素数さん
垢版 |
2023/08/25(金) 02:14:34.77ID:wUa47Otl
>>917
しらん
計算面倒だから適当に読み替えてくれ
0921132人目の素数さん
垢版 |
2023/08/25(金) 02:29:16.20ID:wUa47Otl
>>917
計算した

G_a
= {R(a)(cosθ, sinθ, 0)} (R(a)は、x軸中心のa回転)
= {(cosθ, sinθcosa, sinθsina)}

こうだ
0922132人目の素数さん
垢版 |
2023/08/25(金) 02:33:01.99ID:wUa47Otl
>>912
というか、

τ∈H = {z∈C | Im(z) > 0}に対して、
G_τ = C/(Z + τZ) (複素トーラス)

でいいのでは?
0923132人目の素数さん
垢版 |
2023/08/25(金) 02:40:30.89ID:wUa47Otl
あ、楕円曲線もC上だと全部S^1 × S^1と同型なのか
0924132人目の素数さん
垢版 |
2023/08/25(金) 07:28:14.02ID:qPl1I7cg
>>922
あそうか
なら
x∈R^3
Gx=R^3/Rx
でいいかホボホボ全部R^2と同型だけどx=0のときだけR^3
x=0除いてもバンドルとしては非自明(R^2/Rxだと自明)
0927132人目の素数さん
垢版 |
2023/08/25(金) 12:40:05.44ID:yi31iKMN
極座標というのがあります。
極座標系の基底ベクトルはなぜあのようなものを選ぶのでしょうか?
同径方向の単位ベクトルを直交基底を構成するベクトルの1つに選ぶのは分かります。
2次元の場合には、直交基底を構成するもう一つのベクトルは、右手系にしたければあのようなベクトルを選ばざるを得ません。
3次元の場合に、直交基底を構成する他の2つのベクトルが、なぜあのベクトルたちでなければならないのかが分かりません。
0928132人目の素数さん
垢版 |
2023/08/25(金) 13:20:45.31ID:XOEsolWl
>>927
極座標(r, Θ, φ) の r だけを増加させた時に動く方向の単位ベクトル、θ だけを~、φ だけを~
この3つが直交してるからそのまま使ってるだけ
というかそういう便利な座標系だから生き残った。円筒座標系なんかもそう
0929132人目の素数さん
垢版 |
2023/08/25(金) 19:47:41.68ID:zUWw7Oz0
岡潔とアンドレヴェイルって
どっちがすごい数学者ですか?
0930132人目の素数さん
垢版 |
2023/08/25(金) 21:09:08.18ID:F7RtDNjY
>>929
森毅先生が「世界一すごい数学者は誰?」
と聞かれたときにうまい答えをしはったけど
今それが思い出せない。
0931132人目の素数さん
垢版 |
2023/08/25(金) 22:32:25.34ID:qPl1I7cg
>>925
xの関数って意味有る?
0932132人目の素数さん
垢版 |
2023/08/25(金) 22:34:45.89ID:qPl1I7cg
ていうかx∈Xを変数として扱うって
xの違いどう反映するの?
0933132人目の素数さん
垢版 |
2023/08/25(金) 22:35:29.57ID:qPl1I7cg
x∈Xによらずただ変数1つ決めるのと同じじゃないの?って疑問
0934132人目の素数さん
垢版 |
2023/08/25(金) 22:50:09.92ID:mvyQpBuc
元の疑問はパラメトライズされた群の例なわけだから
xの関数は普通にRからRへの関数とでも思ってくれていい
J(x)の成分が変わるとG_xは群として変わっていくから
0935132人目の素数さん
垢版 |
2023/08/25(金) 23:39:45.71ID:qPl1I7cg
つまり
J:X→GL(n,R[x])
0936132人目の素数さん
垢版 |
2023/08/25(金) 23:41:40.87ID:qPl1I7cg
あ違うか
J:X→GL(n,C(R))
0937132人目の素数さん
垢版 |
2023/08/26(土) 02:14:50.75ID:MMl6MI4X
JはGLじゃなくてただのMでもok
だから関数をn×n個並べてるだけ
0938132人目の素数さん
垢版 |
2023/08/27(日) 19:38:05.05ID:D6KY9zT7
J(x) = ((0, x), (-x, 0))にしたら、G_xはxと無関係になった

J(x)でいい感じのない?
0939132人目の素数さん
垢版 |
2023/08/27(日) 23:02:52.79ID:h3MGlSb5
x=0のときは少なくともx≠0のときと違う群になるけどね
たしかに群の同型類で見れば行列の階数や標準形のような感じで実質離散的になってしまう可能性はあるか
0940132人目の素数さん
垢版 |
2023/08/28(月) 01:38:01.08ID:+PlLO6+o
アフィン代数多様体の無限小変形は自明なものしかないので、行列群では連続的に変化する族を作るのは無理では?
0941132人目の素数さん
垢版 |
2023/08/28(月) 02:30:56.66ID:JoHdLIPj
な、なるほど
マフィンなら知ってる…
0942132人目の素数さん
垢版 |
2023/08/28(月) 15:18:09.06ID:lTiy2Dkd
>>940
なんで?
{(cosθ -tsinθ)
((1/t)sinθ cosθ)|t>0}
は?
0943132人目の素数さん
垢版 |
2023/08/28(月) 15:20:18.66ID:lTiy2Dkd
ア違った
{{(cosθ -tsinθ)
((1/t)sinθ cosθ)|θ∈R}|t>0}
0944132人目の素数さん
垢版 |
2023/08/28(月) 20:20:28.70ID:iOzGlv58
思い付かんし、行列群の互いに同型でない族が無数にあるなら、すでにみんな知ってそう
0945132人目の素数さん
垢版 |
2023/08/29(火) 00:23:17.65ID:APBsTlo0
>>944
>行列群の互いに同型でない族
とは?
0947132人目の素数さん
垢版 |
2023/08/29(火) 08:41:26.77ID:JDHoIQCt
互いにSL-共役だからがんばっても示せない
0948132人目の素数さん
垢版 |
2023/08/29(火) 08:51:24.52ID:KzCpg1Vs
GLじゃないだめか
0950132人目の素数さん
垢版 |
2023/08/29(火) 09:10:23.11ID:ZWoCrYYV
代数群の変形ならその関数環の変形を考える
抽象群としての非同型性は知らん
0951132人目の素数さん
垢版 |
2023/08/29(火) 10:02:34.66ID:mhUDh+Tj
可換群を連族変形して非可換化するくらいは
楽にできるのではないか
0953132人目の素数さん
垢版 |
2023/08/29(火) 10:42:10.57ID:APBsTlo0
>>946
は?同型だけど?
>>940
>行列群では連続的に変化する族を作るのは無理では?
の反例てだけ
もともと条件として>>912
>適当な意味で連続的に変化する
てだけだし
ところで
x∈Rx⊂R^2
は連続的に変化してるとは見做さない?ならもともとの「適当な意味で連続的に変化する」をもう少しハッキリさせて欲しいね>>912
0955132人目の素数さん
垢版 |
2023/08/29(火) 13:49:45.97ID:uLBYu+md
量子群とかがおそらくそう
0956132人目の素数さん
垢版 |
2023/08/29(火) 13:50:54.24ID:s6avn+3Q
>>953
このアスペって何年数学板に粘着してるんだ
0957132人目の素数さん
垢版 |
2023/08/29(火) 13:59:56.71ID:bQ9ne+62
>>912
>パラメータ付けされた群の族の非自明な例が思いつかない
あと非自明性の定義もハッキリしてほしいのと
何らかの群の中での話かただの群の族の話かもか
0958132人目の素数さん
垢版 |
2023/08/29(火) 14:58:07.47ID:ktjdjM3O
高校範囲で解けるけど数学科の大学生ですら手こずる整数問題を教えてください
0959132人目の素数さん
垢版 |
2023/08/29(火) 15:47:33.34ID:dwUOnF2H
素数定理の初等的証明
0960132人目の素数さん
垢版 |
2023/08/29(火) 16:56:33.92ID:Xz1mFSEJ
答え知ってれば一瞬だけど知らなきゃまず解けないタイプの問題は全部そうでしょ
0961132人目の素数さん
垢版 |
2023/08/29(火) 17:27:59.89ID:1VSuGews
>>958
数オリ問題集
0962132人目の素数さん
垢版 |
2023/08/29(火) 20:22:16.37ID:2PU6wOJI
コンパクトリー群からはみ出して量子群に移行すれば変形できるらしい
0963132人目の素数さん
垢版 |
2023/08/29(火) 20:58:00.76ID:nI7SUyvw
>>962
そこで言う変形とは何かちゃんと定義して言ってね
0964132人目の素数さん
垢版 |
2023/08/29(火) 21:25:04.79ID:ukqPSbMq
そもそも量子群で非可換化する演算は群じゃないでしょ
それにq変形のようなものも結局q=0とq≠0で類別されてしまいそう
0965132人目の素数さん
垢版 |
2023/08/29(火) 21:58:37.82ID:rOkmjxrE
物理学徒だけど非可換な構造が入ってる体Xについて
A,B ∈ X, AB-BA=εとして
X(ε)を考えてX(0)とX(ε)を同類(?言葉知らない)にしてX(0)が解析力学、x(ε)が量子力学みたいなこと出来れば古典と量子の狭間が解明出来る気がするけど多分すごく変な系が出てくるだけで物理的に無意味になりそうなのでモチベーションはゼロ
0966132人目の素数さん
垢版 |
2023/08/29(火) 22:14:11.32ID:gR2YEly7
e^iπ を求める方法
0967132人目の素数さん
垢版 |
2023/08/29(火) 22:40:58.09ID:6A3ntVHd
e^xのテイラー展開にixを代入して、cosx, sinxのテイラー展開と比較する

f(x) = e^(-ix)(cosx + isinx) を微分する

二階線形微分方程式 d^2y/dx = -y の解であることからわかる

逆関数の∫ dx/x を 1→1+i→-1+i→-1→-1-i→1-i→1 の積分路で留数定理を使って計算するとわかる


指数法則だけからわかったりするんだろうか
0970132人目の素数さん
垢版 |
2023/08/29(火) 23:56:46.90ID:nI7SUyvw
>>969
exp(iπ)=lim(1+iπ/n)^n
=lim(√(1+π^2/n^2)(cosarctanπ/n+isinarctanπ/n))^n
=lim(1+π^2/n^2)^(n/2)(cosnarctanπ/n+isinnarctanπ/n)
=lim((1+π^2/n^2)^(n^2))^(1/2n)(cosnarctanπ/n+isinnarctanπ/n)
=cosπ+isinπ
=-1
0971132人目の素数さん
垢版 |
2023/08/30(水) 00:18:40.48ID:xGw/1Y9k
lim(1+iπ/n)^n
=lim(1-C[n,2](π/n)^2+C[n,4](π/2)^4+…)+ilim(C{n,1](π/n)-C[n,3](π/2)^3+…)
=lim(1-π^2/2!(1-1/n)+π^4/4!(1-1/n)(1-2/n)(1-3/n)+…)+ilim(π-π^3/3!(1-1/n)(1-2/n)+π^5(1-1/n)(1-2/n)(1-3/n)(1-4/n)+…)
=cosπ+isinπ
=-1
0972132人目の素数さん
垢版 |
2023/08/30(水) 07:55:18.50ID:N/VTqnO1
>>912
H⊂GLを部分群として、x∈GLに対して

G_x = xHx^(-1)

はどうか?
0973132人目の素数さん
垢版 |
2023/08/31(木) 15:15:27.40ID:7y5nXJ+5
幾何学的な話を重視した線形代数の本ってないですか?

例えば、↓のような定理が沢山載っているような本です。

中岡稔・服部晶夫著『線型代数入門』に以下の定理があります。

f は R^3 上の回転で、対称変換でないとする。右手系の標準基底に関する f の行列表示を A として、

B = (1/2) * (A - A^T),
B = {{0, -b3, b2}, {b3, 0, -b1}, {-b2, b1, 0}},
b = {b1, b2, b3}

とおけば、 f は b のまわりの回転を表し、回転角 θ (0 < θ < π)は

sin θ = ||b||,
cos θ = (1/2) * (tr A - 1)

をみたす。
0974132人目の素数さん
垢版 |
2023/08/31(木) 15:37:24.38ID:wKBynqp2
法をpとする時、a^x≡1となる最小のxを簡単に求めるにはどのようにすればよいのでしょうか?x-1の正の約数になるのはわかっているのですが…
0975132人目の素数さん
垢版 |
2023/08/31(木) 19:53:35.65ID:7y5nXJ+5
>>973

f は R^3 上の回転で、対称変換である場合に、回転の軸を表す簡単な式はないんですか?
もちろん、 θ = 0 である場合、すなわち恒等変換である場合には、回転しないので軸自体がありませんが。
θ = π の場合にはどうなんでしょうか?
0976132人目の素数さん
垢版 |
2023/08/31(木) 20:00:16.04ID:651o0dmf
>>975
固有値1の固有ベクトルを求めたら?
0977132人目の素数さん
垢版 |
2023/08/31(木) 20:03:14.24ID:7y5nXJ+5
あ、勘違いしていました。回転の軸を表すベクトルは求まりますね。
例えば、恒等変換の場合には、任意のベクトルが軸になりますね。
0978132人目の素数さん
垢版 |
2023/08/31(木) 20:04:58.03ID:7y5nXJ+5
>>976
ありがとうございます。
そうですね。
0979132人目の素数さん
垢版 |
2023/08/31(木) 22:16:06.11ID:LHP8euCC
nを正の整数とする。C[[t]]の部分環Aと極大イデアルmの組(A, m)で以下の条件をみたすものをひとつ求めなさい。

(1) AはCを含む
(2) C[[t]]/Aの、Cベクトル空間としての次元は有限
(3) Aの商体における整閉包はC[[t]]
(4) m/m^2 のCベクトル空間としての次元はn
0980132人目の素数さん
垢版 |
2023/09/01(金) 02:15:39.10ID:CI/qYquy
原点に、余接空間がn次元になる特異点をもつ曲線をつくればいいと思う
0981132人目の素数さん
垢版 |
2023/09/01(金) 15:43:21.53ID:YAnAZXqd
n = 2なら、C[[t^2, t^3]]がその例か
0982132人目の素数さん
垢版 |
2023/09/02(土) 00:38:01.15ID:EPjjJmNy
y^n - x^(n+1) - x^n
= y^n - x^n(1 + x)
= Π[k=0, n-1](y - ζ^k x (1 + x)^(1/n)) (ζ = exp(2πi/n))

これどうよ
0983132人目の素数さん
垢版 |
2023/09/02(土) 11:10:18.28ID:7Gu9ZxtL
f:ℝ^n→ℝ^n
f_1(x)=f(x)=Ax+b
(Aはn次正方行列,bはゼロベクトルでないn次定ベクトル)
f_{m+1}(x)=f(f_m(x)) (m=1,2,…)
としたとき、
A,A-E(Eはn次単位行列)が正則ならば、
任意のmに対して(E-A(A-E)^(-1))b∈Im f_mである

ことを示したいのですがどのようにすれば良いでしょうか…
0985132人目の素数さん
垢版 |
2023/09/02(土) 15:36:41.32ID:7Gu9ZxtL
たしかに!
実際に手を動かしてみたら当たり前でした!
ありがとうございました!
0986132人目の素数さん
垢版 |
2023/09/03(日) 10:26:23.42ID:HPrPZntd
James R. Munkres著『Analysis on Manifolds』

テンソル積が、

結合法則
(c・f) * g = c・(f * g) = f * (c・g)
分配法則

などを満たすことを定理として述べています。

これは実数の集合上での積の性質から明らかです。

同じく明らかな性質として、テンソル積は交換法則を満たします。
ですが、そのことは書いてありません。

なぜでしょうか?
0987132人目の素数さん
垢版 |
2023/09/03(日) 10:54:34.96ID:n49vj0nO
満たさないから
厳密に言えば結合則も満たさない
(厳密には直積が結合則を満たさないから)
0988132人目の素数さん
垢版 |
2023/09/03(日) 13:08:31.76ID:atSo7mtU
テンソル積は結合法則も交換法則も満たさないが
0989132人目の素数さん
垢版 |
2023/09/03(日) 13:54:11.69ID:AkoYJnwT
テンソル積とはモノイダル圏が持つ双関手のことであり、モノイダル圏の定義から結合律(と左右の単位律)は満たす
結合律を満たさないものはモノイダル圏ではないのでa fortioriにその演算もテンソル積ではない

結合律を満たさないというのは、
結合律を「集合として等しい」とすれば確かに成り立たないが、
これをモノイダル圏の定義がそうしているように自然同型とすれば成り立つ
実際、集合の圏は直積をテンソル積としてモノイダル圏になる
0991132人目の素数さん
垢版 |
2023/09/03(日) 16:54:38.27ID:HPrPZntd
地球についての情報を得ようとしたのではなく、宇宙空間についての幾何学的な情報を得ようとしたんですよね。
完全に意味不明です。
0992132人目の素数さん
垢版 |
2023/09/03(日) 17:49:56.46ID:HPrPZntd
https://youtu.be/yQZzforso6k?si=WTDyrGWSxZ2fDZ05&t=1350

↑この話もおかしくないですか?
メビウスの帯を1週してきた人が逆さになっていますが、逆さにはならないですよね。
0993132人目の素数さん
垢版 |
2023/09/03(日) 17:57:24.55ID:HPrPZntd
>>992
面の表と裏という概念はないんですか?
スタート時に面の片方の側に張り付いていた平面人は帰ってきたときに、面の他方の側にいるとは考えないんですか?
0994132人目の素数さん
垢版 |
2023/09/03(日) 18:29:21.75ID:n49vj0nO
まあ一生疑問に思ってればいいと思うよ
0995132人目の素数さん
垢版 |
2023/09/03(日) 20:12:32.40ID:HPrPZntd
>>990

↑これやばくないですか?

こんなバカな目的でガウスが測量するわけないですよね?
0997132人目の素数さん
垢版 |
2023/09/03(日) 23:21:59.43ID:n49vj0nO
>>983
A=B+Eとしたら
f(x)=Bx+x+b
この不動点は
x=Bx+x+b
より
x=-B^-1b
ところで
E-A(A-E)^-1=E-(B+E)B^-1=-B^-1
なので
x=(E-A(A-E)^-1)b
とは冗長な記述に過ぎない
0998132人目の素数さん
垢版 |
2023/09/04(月) 17:08:14.39ID:aq+GzwFf
数学の教科書や専門書はそんなに多く所持する必要は無いと思う。
僕自身は4万~5万冊ぐらいしか数学の専門書を持ってないが研究や学生の教育に困ったことは無い。一応日本で出版された全ての数学書とSPRINGERの数学書は全部買って持っているが何よりも読むことが大事。僕は1日5冊ずつ精読している。
10011001
垢版 |
Over 1000Thread
このスレッドは1000を超えました。
新しいスレッドを立ててください。
life time: 117日 23時間 8分 54秒
10021002
垢版 |
Over 1000Thread
5ちゃんねるの運営はプレミアム会員の皆さまに支えられています。
運営にご協力お願いいたします。


───────────────────
《プレミアム会員の主な特典》
★ 5ちゃんねる専用ブラウザからの広告除去
★ 5ちゃんねるの過去ログを取得
★ 書き込み規制の緩和
───────────────────

会員登録には個人情報は一切必要ありません。
月300円から匿名でご購入いただけます。

▼ プレミアム会員登録はこちら ▼
https://premium.5ch.net/

▼ 浪人ログインはこちら ▼
https://login.5ch.net/login.php
レス数が1000を超えています。これ以上書き込みはできません。

ニューススポーツなんでも実況